Tải bản đầy đủ (.pdf) (112 trang)

Tải 30 ĐỀ THI HỌC SINH GIỎI TOÁN LỚP 6 Cấp Huyện – Thành Phố - Tỉnh

Bạn đang xem bản rút gọn của tài liệu. Xem và tải ngay bản đầy đủ của tài liệu tại đây (1.81 MB, 112 trang )

<span class='text_page_counter'>(1)</span><div class='page_container' data-page=1>

<b>PHÒNG GIÁO DỤC VÀ ĐÀO TẠO GIA LAI </b>


<b>TRƯỜNG THCS TRẦN PHÚ </b> <b>ĐỀ THI HỌC SINH GIỎI CẤP TRƯỜNG MƠN TỐN LỚP 6 </b>
<b>NĂM HỌC 2018-2019 </b>


<b>Bài 1. </b>(3 điểm)


Hãy viết số lớn nhất bằng cách dùng 3 chữ số 1;2;3với điều kiện mỗi chữ số
dùng một lần và chỉ một lần


<b>Bài 2. </b>(4 điểm) Tìm <i>x x</i>



2


2 3 2 2


)5 125
)3 81


)5 2.5 5 .3
<i>x</i>


<i>x</i>
<i>x</i>


<i>a</i>
<i>b</i>


<i>c</i> 






 


<b>Bài 3. </b>(4 điểm) Cho <i><sub>M</sub></i>    <sub>2 2</sub>2 <sub>2</sub>3 <sub>2</sub>4 <sub>... 2</sub> 2017<sub>2</sub>2018


a) Tính M


b) Chứng tỏ rằng M chia hết cho 3


<b>Bài 4.</b>(3 điểm) Tìm một số tự nhiên có 6 chữ số tận cùng là chữ số 4. Biết rằng khi


chuyển chữ số 4 đó lên đầu cịn các chữ số khác giữ ngun thì ta được số mới gấp
4 lần số cũ


<b>Bài 5. </b>(6 điểm)


a) Cho 40 điểm trong đó khơng có ba điểm nào thẳng hàng. Cứ qua hai điểm ta
vẽ được một đường thẳng. Hỏi vẽ được bao nhiêu đường thẳng ?


b) Cho 40 điểm trong đó có đúng 10 điểm thẳng hàng, ngồi ra khơng có ba
điểm nào thẳng hàng. Cứ qua hai điểm ta vẽ được một đường thẳng. Hỏi vẽ
được bao nhiêu đường thẳng.


c) Cho n điểm

<i>n</i>

. Trong đó khơng có ba điểm nào thẳng hàng, cứ qua hai


</div>
<span class='text_page_counter'>(2)</span><div class='page_container' data-page=2>

<b>ĐÁP ÁN HỌC SINH GIỎI 6 GIA LAI 2018-2019 </b>
<b>Bài 1. </b>


Trường hợp không dùng lũy thừa, số lớn nhất có thể viết được là 321


*Trường hợp dùng lũy thừa: (Ta bỏ qua lũy thừa có cơ số và số mũ là 1)
- Xét các lũy thừa mà số mũ có một chữ số: 2 2 3 3


13 ;31 ;12 ; 21
So sánh 3 2


21 à 31<i>v</i> ta có 3 2


21 31 (vì 3 2


21 9261; 31 961)
- Xét các lũy thừa mà số mũ có hai chữ số: <sub>2 ; 2 ;3 ;3</sub>13 31 12 21


So sánh 21


3 với 31


2 ta có


 


 



10


21 20 2 10


10


31 30 3 10



3 3.3 3. 3 3.9
2 2.2 2. 2 2.8


  


  


Từ đó suy ra 21 31


3 2 . So sánh 21


3 với <sub>21</sub>3<sub>ta có : </sub><sub>3</sub>21<sub>3</sub>9 

 

<sub>3</sub>3 3 <sub>27</sub>3<sub>21</sub>3


Vậy số lớn nhất là : 21


3
<b>Bài 2. </b>


3
2
2 4


) 5 125
5 5


3
) 3 81
3 3


2 4 2



<i>x</i>
<i>x</i>
<i>x</i>
<i>x</i>
<i>a</i>
<i>x</i>
<i>b</i>
<i>x</i> <i>x</i>


 


   


2 3 2 2


2 3 2 2


2 3 3


) 5 2.5 5 .3
5 5 .3 2.5


5 5


2 3 3
3
<i>x</i>


<i>x</i>
<i>x</i>
<i>c</i>
<i>x</i>
<i>x</i>



 
 

 
 
<b>Bài 3. </b>


a) Ta có <sub>2</sub><i><sub>M</sub></i>   <sub>2</sub>2 <sub>2</sub>3 <sub>2</sub>4 <sub>... 2</sub> 2018<sub>2</sub>2019


Lấy <sub>2</sub><i><sub>M</sub></i><i><sub>M</sub></i> <sub>2</sub>2019<sub>2</sub><sub>. Vậy </sub><i><sub>M</sub></i> <sub>2</sub>2019<sub>2</sub>


b)


 

 







2 3 4 5 6 2017 2018


3 5 2017



3 5 2017


2 2 2 2 2 2 ... 2 2
2 1 2 2 . 1 2 2 .(1 2) ...2 . 1 2
3. 2 2 2 ... 2


<i>M</i>
<i>M</i>
<i>M</i>
        
       
    


Vậy <i>M</i> 3


<b>Bài 4. </b>


</div>
<span class='text_page_counter'>(3)</span><div class='page_container' data-page=3>

Ta có:




4.4 400000
10 4 .4 400000
40 16 400000
39 399984


10256


<i>x</i> <i>x</i>



<i>x</i> <i>x</i>


<i>x</i> <i>x</i>


<i>x</i>
<i>x</i>


 


  


  





Vậy số cần tìm là 10256.


<b>Bài 5. </b>


a) Kẻ từ 1 điểm bất kỳ với các điểm còn lại được : 39 đường thẳng
Làm như vậy với 40 điểm ta được 39.40 1560 (đường thẳng)


Nhưng mỗi đường thẳng được tính hai lần


Do vậy số đường thẳng thực sự là : 1560: 2 780 (đường thẳng)


b) Nếu 40 điểm khơng có ba điểm nào thẳng hàng thì sẽ vẽ được 780 đường thẳng.
*Với 10 điểm, khơng có ba điểm nào thẳng hàng thì vẽ được:



10.9: 2 45 (đường thẳng)


Số đường thẳng cần tìm là : 780 44 736  (đường thẳng)


c) Ta có:




. 1 : 2 105
( 1) 210
( 1) 15.14


<i>n n</i>
<i>n n</i>
<i>n n</i>


 
 
 


Vậy n = 15


<b>PHÒNG GIÁO DỤC VÀ ĐÀO TẠO </b>
<b>HUYỆN TRỰC NINH </b>


<b>ĐỀ CHÍNH THỨC </b>


<b>ĐỀ THI CHỌN HỌC SINH GIỎI </b>
<b>NĂM HỌC 2017-2018 </b>



MƠN TỐN LỚP 6
Thi ngày 04 tháng 4 năm 2018


<i>Thời gian làm bài: 120 phút, không kể thời gian giao đề </i>


<b>Bài 1. </b><i>(5,0 điểm) </i>Tính hợp lý


       

   



2


2 3 4 99 100


) 2018 2017.2018


) 1 . 1 . 1 . 1 ... 1 . 1


1 2 3 88


88 ...


6 7 8 93


)


1 1 1 1


...
12 14 16 186



<i>a A</i>
<i>b B</i>


<i>c C</i>


 


      


    


</div>
<span class='text_page_counter'>(4)</span><div class='page_container' data-page=4>

<b>Bài 2. </b><i>(5,0 điểm) </i>


a. Tìm <i>x y</i>,  biết

2<i>y</i>1



<i>x</i> 4

10


b. Cho <i>x y</i>,  thỏa mãn

3<i>x</i>5<i>y x</i>



4<i>y</i>

7. Chứng tỏ rằng

3<i>x</i>5<i>y x</i>



4<i>y</i>

49
c. Tìm số tự nhiên n trong khoảng 290 đến 360 để phân số 5 2



2 7


<i>n</i>


<i>n</i>
<i>n</i>


 <sub></sub>


 rút



gọn được


<b>Bài 3. </b><i>(4,0 điểm) </i>


a. Tìm số nguyên dương n nhỏ nhất sao cho <i>n</i>1;2<i>n</i>1;5<i>n</i>1đều là số chính
phương?


b. Cho <i><sub>A</sub></i><sub>2017 2017</sub> 2<sub>2017</sub>3<sub>... 2017</sub> 18


Chứng tỏ rằng <i>A</i> 2018. Tìm chữ số tận cùng của A


<b>Bài 4. </b><i>(4,0 điểm) </i>


a. Cho đoạn thẳng AB có độ dài bằng 2 cm. Lấy điểm C thuộc đường thẳng
AB sao cho <i>BC</i>5<i>cm</i>.Tính độ dài đoạn thẳng AC


b. Cho <i><sub>xOy</sub></i><sub>160</sub>0<sub>. Vẽ tia phân giác </sub>
1


<i>Ox</i> của <i>xOy</i>. Tính số đo góc <i>xOx</i>1


Giả sử <i>Ox</i><sub>2</sub>là tia phân giác của <i>xOx</i><sub>1</sub>, <i>Ox</i><sub>3</sub>là tia phân giác của <i>xOx</i><sub>2</sub>,……<i>Ox</i><sub>42</sub>là
tia phân giác của <i>xOx</i><sub>41</sub>. Tính số đo góc <i>xOx</i><sub>42</sub>


<b>Bài 5. </b><i>(2,0 điểm) </i>


a. Chứng minh rằng với mọi số nguyên n ta có 3


6



<i>n</i> <i>n</i>


b. Viết số 1234


4321 dưới dạng tổng của một số số nguyên dương. Gọi T là tổng
các lập phương của tất cả các số đó. Tìm số dư của <i>T</i> trong phép chia cho 6


<b>----hết--- </b>


<b>ĐÁP ÁN HSG 6 TRỰC NINH_2017-2018 </b>
<b>Bài 1. </b>




) 2018. 2018 2017 2018.1 2018


<i>a A</i>   


</div>
<span class='text_page_counter'>(5)</span><div class='page_container' data-page=5>

1 2 3 88


1 1 1 ... 1


6 7 8 93


)


1 1 1 1


...


12 14 16 186


1 1 1 1


5 5 5 5 <sub>5.</sub> <sub>...</sub>


...


6 7 8 93


6 7 8 93


1 1 1 1 1 1 1 1 1


... . ...
12 14 16 186 2 6 7 8 93
10


<i>c C</i>


<i>C</i>


<i>C</i>


 <sub></sub>  <sub> </sub>  <sub> </sub> <sub></sub> <sub> </sub> 
       
       


   



 <sub>  </sub> <sub></sub> 


    <sub></sub> <sub></sub>


 


 


     <sub></sub>     <sub></sub>


 


 
<b>Bài 2. </b>








) 2 8 14
(2 1) 8 4 14 4


2 1 4(2 1) 10


2 1 4 10


<i>a</i> <i>xy</i> <i>x</i> <i>y</i>


<i>x</i> <i>y</i> <i>y</i>



<i>x</i> <i>y</i> <i>y</i>


<i>y</i> <i>x</i>


  


    
   
  


Vì <i>x y</i>,  nên 2<i>y</i> 1 ,<i>x</i> 4 , suy ra 2<i>y</i>1,<i>x</i>4 là ước nguyên của 10 và 2<i>y</i>1lẻ
Lập bảng


2<i>y</i>1 <sub>1 </sub> - 1 5 -5


4


<i>x</i> 10 -10 2 -2


<i>x</i> 14 -6 6 2


<i>y</i> <sub>0 </sub> <sub>-1 </sub> <sub>2 </sub> <sub>-3 </sub>


Vậy 14; 6; 6; 2


0 1 2 3


<i>x</i> <i>x</i> <i>x</i> <i>x</i>



<i>y</i> <i>y</i> <i>y</i> <i>y</i>


    


   


 <sub></sub>  <sub> </sub>  <sub></sub>  <sub> </sub>


   


b) Phải chứng minh 3<i>x</i>5 7<i>y</i>  <i>x</i> 4<i>y</i> 7


Đặt <i>A</i>3<i>x</i>5 ,<i>y B</i> <i>x</i> 4 .<i>y</i> Xét tổng <i>A</i>4<i>B</i>7<i>x</i>21 7


Nếu <i>A</i>74<i>B</i> 7, mà

 

4,7  1 <i>B</i> 7


Nếu <i>B</i> 74<i>B</i> 7<i>A</i> 7.Chứng tỏ 3<i>x</i>5 7<i>y</i>  <i>x</i> 4<i>y</i> 7


3 5



4

7 3 5 7
4 7


<i>x</i> <i>y</i>


<i>x</i> <i>y</i> <i>x</i> <i>y</i>


<i>x</i> <i>y</i>




  <sub>  </sub>





</div>
<span class='text_page_counter'>(6)</span><div class='page_container' data-page=6>

Nếu

<i>x</i>4<i>y</i>

7

3<i>x</i>5<i>y</i>

7

3<i>x</i>5<i>y x</i>



4<i>y</i>

49
c) Gọi d là ước nguyên tố chung của 5<i>n</i>2và 2<i>n</i>7


Ta có: 5 2 2. 5

2

10 35

 

10 4


2 7 5.(2 7)


<i>n</i> <i>d</i>


<i>n</i> <i>d</i>


<i>n</i> <i>n</i> <i>d</i>


<i>n</i> <i>d</i> <i>n</i> <i>d</i>







 <sub></sub> <sub></sub> <sub></sub> <sub></sub> <sub></sub>


 <sub></sub>  <sub></sub>




 



Vì d nguyên tố nên <i>d</i>31


Khi đó 5 2 31 5 2 62 31 5 60 31 5( 12) 31
2 7 31 2 7 31 31 2 24 31 2( 12) 31


<i>n</i> <i>n</i> <i>n</i> <i>n</i>


<i>n</i> <i>n</i> <i>n</i> <i>n</i>


    


 <sub></sub> <sub></sub> <sub></sub>


 <sub></sub>  <sub> </sub>  <sub></sub>  <sub></sub>


   


5,31

 

1; 2;31

1 suy ra <i>n</i>12 31 <i>n</i> 31<i>k</i>12

<i>k</i>



Do 290 <i>n</i> 360290 31 <i>k</i> 12 360  9 <i>k</i> 11, mà k là số tự nhiên nên


9;10;11



<i>k</i>


Từ đó tìm được <i>n</i>

291;322;353



<b>Bài 3. </b>


a) Do <i>n</i>1là số chính phương nên khi chia cho 3 chỉ có thể dư 0 hoặc 1.


Nếu <i>n</i>1 3thì n chia cho 3 dư 2  2<i>n</i> 1chia cho 3 dư 2, vơ lý.


Do đó <i>n</i>1chia cho 3 sẽ dư 1<i>n</i> 3


Do 2<i>n</i>1là số chính phương lẻ nên 2<i>n</i>1chia cho 8 dư 1, suy ra 2 8<i>n</i> , từ đó
4


<i>n</i>


Do đó <i>n</i>1 là số chính phương lẻ nên <i>n</i>1chia cho 8 dư 1, suy ra <i>n</i> 8
Ta thấy <i>n</i> 3,<i>n</i> 8 mà

 

3,8 1 nên <i>n</i> 24mà n là số nguyên dương


Với <i>n</i>24thì <i><sub>n</sub></i> <sub>1 25</sub><sub>5 ; 2</sub>2 <i><sub>n</sub></i> <sub>1 49</sub><sub>7 ; 5</sub>2 <i><sub>n</sub></i> <sub>1 121 11</sub> 2


Vậy <i>n</i>24là số nguyên dương nhỏ nhất thỏa mãn đề bài


b) Ta có <i><sub>A</sub></i><sub>2017 2017</sub> 2<sub>2017</sub>3<sub>... 2017</sub> 2018<sub>(tổng A có 2018 số hạng, </sub><sub>2018 2)</sub>


 





2 3 4 2017 2018


3 2017


3 2017


2017 2017 2017 2017 ... 2017 2017
2017.(1 2017) 2017 .(1 2017) ...2017 .(1 2017)


2018. 2017 2017 ... 2017 2018


<i>A</i>
<i>A</i>
<i>A</i>


      


     


</div>
<span class='text_page_counter'>(7)</span><div class='page_container' data-page=7>



 

 

 



2 3 4 5 6 2015 2016 2017 2018


3 2015


2017 2017 2017 2017 2017 2017 ... 2017 2017 2017 2017
...6 2017 . ....0 ... 2017 . ...0 ...6


<i>A</i>
<i>A</i>


          


    


<b>Bài 4. </b>



a) Trường hợp điểm C thuộc tia đối của tia BA


Điểm C thuộc tia đối của tia BA nên hai tia BA và BC đối nhau, suy ra
điểm B nằm giữa hai điểm A và C


Ta có: <i>AB BC</i> <i>AC</i> thay số tính được <i>AC</i>7<i>cm</i>


Trường hợp điểm C thuộc tia BA


Trên tia BA, <i>BA</i><i>BC</i>

2<i>cm</i>5<i>cm</i>

nên điểm A nằm giữa hai điểm B và C


Ta có: <i>AB AC</i> <i>BC</i> Thay số tính được <i>AC</i>3<i>cm</i>


b)


Tia <i>Ox</i>1là tia phân giác của <i>xOy</i>nên


0
0
1


160
80


2 2


<i>xOy</i>


<i>xOx</i>   



Tia <i>Ox</i>2là tia phân giác của <i>xOx</i>1nên


0
1


2 2


160


2 2


<i>xOx</i>


<i>xOx</i>  


<i><b>A</b></i>

<i><b>B</b></i>

<i><b>C</b></i>



<i><b>C</b></i>

<i><b>A</b></i>

<i><b>B</b></i>



<b>y</b>



<b>x1</b>



<b>x2</b>



<b>x3</b>



<b>x</b>



</div>
<span class='text_page_counter'>(8)</span><div class='page_container' data-page=8>

Tương tự như trên, tia <i>Ox</i><sub>42</sub> là tia phân giác của <i>xOx</i><sub>41</sub> nên


0
41
42 42
160
2 2
<i>xOx</i>


<i>xOx</i>  


<b>Bài 5 </b>


a) Ta có

3

 

2

 

2

 





1 1 1 1 1 1


<i>n</i>  <i>n</i> <i>n n</i>  <i>n n</i>    <i>n n</i> <i>n n n</i><sub></sub>   <i>n</i> <sub></sub><i>n n</i> <i>n</i>


Với mọi số nguyên dương n thì

<i>n</i>1

 

<i>n n</i>1

là tích của 3 số tự nhiên liên tiếp sẽ
chia hết cho 2 và 3 mà

 

2,3 1 nên <i>n n</i>

1



<i>n</i>1 6



b) Ta có


1234


1 2 3


3 3 3 3


1 2 3



4321 ...


...


<i>n</i>
<i>n</i>


<i>a</i> <i>a</i> <i>a</i> <i>a</i>


<i>T</i> <i>a</i> <i>a</i> <i>a</i> <i>a</i>


    
    


Xét hiệu 1234

3 3 3 3



1 2 3 1 2 3


4321 ... <i><sub>n</sub></i> ... <i><sub>n</sub></i>


<i>T</i>  <i>a</i>   <i>a</i> <i>a</i> <i>a</i>  <i>a</i>   <i>a</i> <i>a</i> <i>a</i>


 

 



1234 3 3 3 3


1 1 2 2 3 3


4321 ... <i><sub>n</sub></i> <i><sub>n</sub></i>



<i>T</i>  <i>a</i> <i>a</i>  <i>a</i> <i>a</i>  <i>a</i> <i>a</i>   <i>a</i> <i>a</i>


Theo câu a ta có 3 3 3 3


1 1 6, 2 2 6, 3 3 6,... <i>n</i> <i>n</i> 6,


<i>a</i> <i>a</i> <i>a</i> <i>a</i> <i>a</i> <i>a</i> <i>a</i> <i>a</i> nên <i><sub>T</sub></i><sub>4321</sub>1234 <sub>6</sub>


Suy ra T và 1234


4321 cùng dư khi chia cho 6
Mặt khác 4321 chi 6 dư 1 nên 1234


4321 chia cho 6 cũng dư 1. Vậy T chia 6 dư 1


<b>PHÒNG GD&ĐT NGA SƠN </b>
<b>TRƯỜNG THCS NGA THẮNG </b>


<b>ĐỀ THI CHÍNH THỨC </b>


<b>KỲ THI CHỌN HỌC SINH GIỎI </b>
<b>NĂM HỌC 2017-2018 </b>
<b>Mơn thi: TỐN – Lớp 6 </b>


<b>Thời gian: 120 phút </b><i>(không kể thời gian giao đề) </i>


<b>Ngày thi: -3-2018 </b>


<i>(Đề thi gồm 1 trang) </i>



<b>Câu 1. </b><i>(4,0 điểm) </i>Thực hiện phép tính bằng cách hợp lý:




) 2013 .2014 1007.26
1313 10 130 1515
)


1414 160 140 1616


<i>a</i>
<i>b</i>
 
 <sub></sub>  <sub></sub> <sub></sub> 
   
   


<b>Câu 2. </b><i>(6,0 điểm) </i>


a) Tìm <i>x y z</i>, , biết <i>x</i> <i>y</i> 2011 ; <i>y z</i>  2012; <i>z</i> <i>x</i> 2013


b) Tìm hai số tự nhiên a và b biết <i>BCNN a b</i>( , ) 180 ; <i>UCLN a b</i>( , ) 12
c) Tìm <i>n</i> để phân số 4 1


2 3
<i>n</i>
<i>A</i>
<i>n</i>




</div>
<span class='text_page_counter'>(9)</span><div class='page_container' data-page=9>

Một hiệu sách có năm hộp bít bi và bút chì. Mỗi hộp chỉ đựng một loại bút. Hộp 1:
78 chiếc; Hộp 2: 80 chiếc; Hộp 3: 82 chiếc; Hộp 4: 114 chiếc; Hộp 5: 128 chiếc.
Sau khi bán một hộp bút chì thì số bút bi gấp bốn lần số bút chì cịn lại. Hãy cho
biết lúc đầu hộp nào đựng bút bi, hộp nào đựng bút chì ?


<b>Câu 4. </b><i>(4,0 điểm) </i>Trên tia Ox cho 4 điểm A, B, C, D. Biết rằng A nằm giữa B và


C; B nằm giữa C và D; <i>OA</i>7<i>cm OD</i>; 3<i>cm BC</i>; 8<i>cm</i>và <i>AC</i>3<i>BD</i>
a) Tính độ dài AC


b) Chứng tỏ rằng: Điểm B là trung điểm của đoạn thẳng AD


<b>Câu 5 </b><i>(2,0 điểm) </i> Tìm số nguyên dương n nhỏ nhất sao cho sau khi viết tiếp số đó
sau số 2014 ta được số chia hết cho 101


</div>
<span class='text_page_counter'>(10)</span><div class='page_container' data-page=10>

<b>ĐÁP ÁN </b>
<b>Câu 1 </b>










) 2013 .2014 1007.26
2013 .2014 2014.13
2014. 2013 13



2014. 2000 4028000
1313 10 130 1515
)


1414 160 140 1616
13 1 13 15


14 16 14 16
13 13 15 1


1
14 14 16 16


<i>a</i>


<i>b</i>


 


  


  


   


 <sub></sub>  <sub></sub> <sub></sub> 


   



   


   


<sub></sub>  <sub> </sub>  <sub></sub>


   


   


<sub></sub>  <sub> </sub>  <sub></sub>


   


<b>Câu 2. </b>


a) Từ đề bài ta có:


 

 

2011

2012

2013
2 2012 1006


<i>x</i> <i>y</i> <i>y</i> <i>z</i> <i>z</i> <i>x</i>


<i>x</i> <i>x</i>


        


   


Vì <i>x y</i> 2011  <i>y</i> <i>x</i> 2011 1006 2011   1005


Vì <i>x z</i> 2013 <i>z</i> 2013 <i>x</i> 2013 1006 1007 


Vậy <i>x</i>1006 ;<i>y</i> 1005 ; <i>z</i>1007
b) Ta có <i>ab</i>180.12 2160


Giả sử <i>a b</i> .Vì <i>UCLN a b</i>( , ) 12 nên <i>a</i>12 ,<i>m b</i>12<i>n</i>với

<i>m n</i>,

1và <i>m</i><i>n</i>
Suy ra 12 .12<i>m</i> <i>n</i>2160<i>mn</i>15. Ta có bảng sau:


m n <i>a</i> <i>b</i>


1 15 12 180


3 5 36 60


c) 4 1 2 2

3

7 2 7


2 3 2 3 2 3 2 3


<i>n</i>
<i>n</i>


<i>A</i>


<i>n</i> <i>n</i> <i>n</i> <i>n</i>





    



   


A có giá trị nguyên 2<i>n</i> 3 <i>U</i>

  

7   1; 7


Ta có bảng sau


2<i>n</i>3 1 -1 7 -7


<i>n</i> -1 -2 2 -5


<b>Câu 3. </b>


</div>
<span class='text_page_counter'>(11)</span><div class='page_container' data-page=11>

Vì số bút bi cịn lại gấp bốn lần số bút chì cịn lại nên tổng số bút bi và số
bút chì còn lại là số chia hết cho 5, mà 482 chia cho 5 dư 2 nên hộp bút chì
bán đi có số lượng chia cho 5 dư 2.


Trong các số 78; 80; 82; 114; 128 chỉ có 82 chia cho 5 dư 2.
Vậy hộp bút chì bán đi là hộp 3: 82 chiếc


Số bút bi và bút chì cịn lại là : 482 82 400  (chiếc)
Số bút chìn cịn lại : 400: 5 80 (chiếc)


Vậy , các hộp đựng bút chì là: hộp 2, hộp 3
Các hộp đựng bút bi là: hộp 1, hộp 4, hộp 5


<b>Câu 4. </b>


a) Đặt <i>BD</i><i>x cm</i>( )<i>AC</i>3 (<i>x cm</i>)


Vì D nằm giữa O và A (Do OD < OA) nên : <i>OD DA OA</i>  <i>DA</i>4



4 4 (1)


<i>DB BA</i> <i>hay x BA</i>


    


Vì A nằm giữa B và C nên : <i>BA AC</i> <i>BC</i> hay 3<i>x BA</i> 8(2)


Từ (1) và (2) ta có

3<i>x BA</i>

 

 <i>x BA</i>

  8 4 2<i>x</i>  4 <i>x</i> 2<i>AC</i>3.2 6( <i>cm</i>)
b) Theo (1) ta có: <i>x BA</i> 4mà <i>x</i> 2 <i>BA</i>2


Mà <i>BD</i>  <i>x</i> 2 <i>BD</i><i>BA</i>( 2) <i>B</i>là trung điểm của đoạn thẳng AD


<b>Câu 5. </b>


Giả sử n có k chữ số

<i>k</i>1



Ta có : 2014 19.101 95  , do đó:


2014<i><sub>n</sub></i>2014.10<i>k</i> <i><sub>n</sub></i> 19.101.10<i>k</i> 95.10<i>k</i><i><sub>n</sub></i>


Suy ra 2014 101<i>n</i> khi và chỉ khi 95.10<i>k</i><i>n</i>101


Với <i>k</i>1thì 95.10<i>k</i>  <i>n</i> 950 <i>n</i> 101.9 (41 <i>n</i>) 101khi và chỉ khi 41<i>n</i>101 nhưng n
có một chữ số nên 41   <i>n</i> 41 9 101, nên khơng có số n thỏa mãn đầu bài.


Với <i>k</i>2thì 95.10<i>k</i> <i><sub>n</sub></i> 9500 <i><sub>n</sub></i> 101.94 

6 <i><sub>n</sub></i>

101<sub> suy ra </sub>


6<i>n</i>101, và số n nhỏ
nhất được xác định bởi 6 <i>n</i> 101 <i>n</i> 95



Vậy n = 95 thỏa mãn đề bài


</div>
<span class='text_page_counter'>(12)</span><div class='page_container' data-page=12>

UBND HUYỆN KINH MƠN


<b>PHỊNG GIÁO DỤC VÀ ĐÀO TẠO </b>


<b>ĐỀ THI OLYMPIC NĂM HỌC 2017-2018 </b>
<b>MƠN: TỐN – LỚP 6 </b>


Thời gian làm bài: 150 phút


<b>Câu 1. (2,0 điểm) </b>


1) Rút gọn biểu thức 1 1 . 1 1 . 1 1 ... 1 1


3 8 15 2499


<i>A</i> <sub></sub>  <sub> </sub>  <sub> </sub>   <sub></sub> <sub></sub>  <sub></sub>


       


2) Tính nhanh


1 1 1 4 4 4


1 4


3 9 27 <sub>:</sub> 7 49 343



2 2 2 1 1 1


2 1


3 9 27 7 49 343


<i>B</i>


     


     
<b>Câu 2. (2,0 điểm) </b>


1) Tìm x, biết 1 1 1 ... 1 23
1.2 2.3 3.4 8.9 <i>x</i> 45


 <sub></sub> <sub></sub> <sub></sub> <sub></sub>  <sub></sub>


 


 


2) So sánh:


99
100


2018 1
2018 1



<i>E</i> 


 và


98
99


2018 1
2018 1


<i>F</i>  



<b>Câu 3: (2,0 điểm) </b>


1) Tìm số tự nhiên <i>x y</i>, biết 5<i>x</i>11<i>y</i> 26


2) Tìm số nguyên tố <i>ab</i>

<i>a b</i> 0

biết <i>ab ba</i> là số chính phương


<b>Câu 4: (3,0 điểm) </b>


1) Trên tia Ox lấy 2 điểm A, B sao cho <i>OA</i>6<i>cm OB</i>, 10<i>cm</i>.Gọi E, F lần lượt là
trung điểm của OA, AB. Tính độ dài đoạn thẳng EF.


2) Trên cùng nửa mặt phẳng bờ chứa tia Ox vẽ hai tia Oy, OZ sao cho


0 0


50 ; 100



<i>xOy</i> <i>xOz</i> . Vẽ tia Oy’ là tia đối của tia Oy. Tính số đo <i>y Oz</i>'


3) Cho 2018 điểm phân biệt trong đó có đúng 3 điểm thẳng hàng. Qua hai điểm
ta kẻ được một đường thẳng. Tính số đường thẳng kẻ được.


<b>Câu 5 (1,0 điểm) </b>


Cho <i>abc</i> là số tự nhiên có ba chữ số. Tìm giá trị lớn nhất của <i>A</i> <i>abc</i> 1918


<i>a b c</i>


 


 


</div>
<span class='text_page_counter'>(13)</span><div class='page_container' data-page=13>

<b>ĐÁP ÁN HỌC SINH GIỎI TỐN 6 KINH MƠN 2017-2018 </b>
<b>Câu 1. </b>


1 1 1 1


1) 1 . 1 . 1 ... 1


3 8 15 2499


4 9 16 2500 2.2 3.3 4.4 50.50
. . ... . . ...
3 8 15 2499 1.3 2.4 3.5 49.51
2.3.4...50 2.3.4...50 50 2 1



. .


1.2.3...49 3.4.5...51 1 51


<i>A</i>
<i>A</i>
       
 <sub></sub> <sub> </sub>  <sub> </sub>  <sub></sub> <sub></sub>  <sub></sub>
       
 


   00


51
Vậy 100


51


<i>A</i>


1 1 1 4 4 4


1 4


3 9 27 7 49 343


2) :


2 2 2 1 1 1



2 1


3 9 27 7 49 343


1 1 1 1 1 1


1. 1 4. 1


3 9 27 7 49 343
:


1 1 1 1 1 1


2. 1 1. 1


3 9 27 7 49 343


1 1
: 4
2 8
<i>B</i>
<i>B</i>
<i>B</i>
     

     
 <sub>  </sub>   <sub> </sub> <sub></sub> 
   
   


 <sub>  </sub>   <sub> </sub> <sub></sub> 
   
   
 


Vậy 1
8


<i>B</i>


<b>Câu 2. </b>


1 1 1 1 23


1) ...


1.2 2.3 3.4 8.9 45


1 1 1 1 1 1 1 1 23


...


1 2 2 3 3 4 8 9 45


1 1 23
1 9 45


8 23 23 8 23
:



9 45 45 9 40


<i>x</i>
<i>x</i>
<i>x</i>
<i>x</i> <i>x</i>
 <sub></sub> <sub></sub> <sub></sub> <sub></sub>  <sub></sub>
 
 
 <sub>     </sub> <sub> </sub>  <sub></sub>
 
 
 <sub></sub>  <sub></sub>
 
 
   


Vậy 23
40


<i>x</i>


2)Ta có:


99 100


100 100 100


2018 1 2018 2018 2017



2018 2018. 1


2018 1 2018 1 2018 1


<i>E</i>   <i>E</i>   <i>E</i> 


  


98 99


99 99 99


2018 1 2018 2018 2017


2018. 2018. 1


2018 1 2018 1 2018 1


<i>F</i>    <i>F</i>    <i>F</i>  


</div>
<span class='text_page_counter'>(14)</span><div class='page_container' data-page=14>

Vì 2017<sub>100</sub> 2017<sub>99</sub> 1 2017<sub>100</sub> 1 2017<sub>99</sub>
2018 12018 1 2018 1 2018 1
Hay 2018<i>E</i>2018<i>F</i> <i>E</i> <i>F</i>


Vậy E > F


<b>Bài 3 </b>


1. +Với <i>y</i>2, ta có 2



11 121 26  <i>y</i> 2 không thỏa mãn
Do y là số tự nhiên nên <i>y</i>

 

0;1


+) Với y = 1, ta có: 5<i>x</i> 11 26 5<i>x</i> 15vì x là số tự nhiên khơng có giá trị
nào của x thỏa mãn 5<i>x</i> 15 <i><sub>y</sub></i> 1<sub> không thỏa mãn </sub>


+)Với <i>y</i>0 ta có 2


5<i>x</i> 1 26 5<i>x</i> 25 5 nên x=2 (thỏa mãn)
Vậy <i>x</i>2;<i>y</i>0


2. Ta có: <i>ab ba</i> 9

<i>a b</i>



Do a, b là các chữ số, <i>ab</i>là số nguyên tố, nên 3 <i>b</i> 9.

<i>a b</i>

là số chính
phương khi <i>a b</i> 

 

1;4


+) Với <i>a b</i> 1mà <i>ab</i>là số nguyên tố ta được số <i>ab</i>43


+)Với <i>a b</i> 4 mà <i>ab</i>là số nguyên tố ta được số <i>ab</i>73


Vậy <i>ab</i>

43;73



<b>Bài 4. </b>


Câu 1


Vì hai điểm A, B cùng nằm trên tia Ox mà OA < OB

6<i>cm</i>10<i>cm</i>

nên điểm
A nằm giữa hai điểm O và B<i>OA AB OB</i> 


Thay số 6<i>AB</i> 10 <i>AB</i>4<i>cm</i>. Vậy <i>AB</i>4<i>cm</i>


Vì E là trung điểm OA nên


2


<i>OA</i>


<i>EA</i> , thay số <i>EA</i>6: 2 3 <i>cm</i>


F là trung điểm của <i>AB</i>nên


2


<i>AB</i>
<i>AF</i>


Thay số: <i>AF</i>4: 2 2 <i>cm</i>


Do A nằm giữa O và B. Mà E là trung điểm của OA, F là trung điểm của AB
nên điểm A nằm giữa hai điểm E và F


</div>
<span class='text_page_counter'>(15)</span><div class='page_container' data-page=15>

3 2 5( )


<i>EF</i> <i>EA AF</i> <i>cm</i>


      Vậy <i>EF</i>5<i>cm</i>.
Câu 2


Vì hai tia <i>Oz Oy</i>, cùng nằm trên nửa mặt phẳng bờ chứa tia <i>Ox</i>, mà <i>xOy</i><i>xOz</i>


nên tia Oy nằm giữa hai tia Ox vfa Oz



<i>xOy</i> <i>yOz</i> <i>xOz</i>


   . Thay số 0 0 0


50 <i>yOz</i>100 <i>yOz</i>50
Do tia <i>Oy</i>'là tia đối của tia <i>Oy</i><i>y Oz yOz</i>' , là hai góc kề bù


0


' 180


<i>y Oz</i> <i>yOz</i>


   . Thay số : 0 0 0


' 50 180 ' 130


<i>y Oz</i>  <i>y Oz</i>


Vậy 0


' 130


<i>y Oz</i>


Câu 3.


Giả sử trong 2018 điểm khơng có ba điểm nào thẳng hàng



Từ 1 điểm ta nối với 2017 điểm còn lại ta được 2017 đường thẳng. Làm như
vậy với 2018 điểm ta được 2018.20174070306đường thẳng


Vì mỗi đường thẳng được tính hai lần, do đó số đường thẳng kẻ được là :
2035153đường thẳng.


Số đường thẳng đi qua 3 điểm không thẳng hàng là 3; Số đường thẳng đi qua
3 điểm phân biệt thẳng hàng là 1; Khi thay 3 điểm phân biệt không thẳng
hàng thành 3 điểm phân biệt thẳng hàng thì số đường thẳng giảm đi là :


3 1 2 


Do trong 2018 điểm phân biệt trên có đúng ba điểm thẳng hàng nên số
đường thẳng thực tế kẻ được là : 2035153 2 2035151 


<i><b>y'</b></i>



<i><b>y</b></i>



<i><b>x</b></i>


<i><b>z</b></i>



</div>
<span class='text_page_counter'>(16)</span><div class='page_container' data-page=16>

Vậy ta kẻ được tất cả là 2 035 151 đường thẳng.


<b>Câu 5 </b>


100 10


1918 1918



<i>abc</i> <i>a</i> <i>b c</i>


<i>A</i>


<i>a b c</i> <i>a b c</i>


 


   


   


<b>+)</b>Nếu <i>b c</i> 0thì <i>A</i>100 1918 2018 


<b>+)</b>Nếu b hoặc c khác 0thì
100 100 100


1918 100 1918 2018


<i>a</i> <i>b</i> <i>c</i>


<i>A</i>


<i>a b c</i>


 


    


 



Nên <i>A</i>2018


Giá trị lớn nhất của A là 2018<i>khi a</i>

1;2;...;9 ;

<i>b</i> <i>c</i> 0


<b>PHÒNG GIÁO DỤC ĐÀO TẠO </b>
<b>BÁ THƯỚC </b>


<b>ĐỀ THI CHỌN HỌC SINH GIỎI CẤP TRƯỜNG </b>
<b>Năm học 2018-2019 </b>


<b>Mơn : Tốn lớp 6 </b>
<b>Câu 1. (3 điểm) Tính </b>


2 5 5


2


5 2 5


1 2 .7 2


)4.5 3. 24 9 )7 6. )


2 2 .5 2 .3


<i>a</i>   <i>b</i>  <sub></sub> <sub></sub> <i>c</i> 





 


<b>Câu 2. (3 điểm). </b>Tìm <i>x</i>biết:


 

1 1 5 5


) 15 : 5 22 24 ) 7 15 4 ) : 9


2 3 7 7


<i>a x</i>   <i>b x</i>    <i>c</i> <sub></sub><i>x</i> <sub></sub>  


 


<b>Câu 3. (5 điểm) </b>


1) Cho <i>A</i>     1 2 3 4 ... 99 100
a) Tính <i>A</i>


b) A có chia hết cho 2, cho 3, cho 5 khơng ?


c) <i>A</i>có bao nhiêu ước tự nhiên ? Bao nhiêu ước nguyên ?
2) Thay <i>a b</i>, bằng các chữ số thích hợp sao cho 24 68 45<i>a</i> <i>b</i>


3) Cho <i>a</i>là một số nguyên có dạng <i>a</i>3<i>b</i>7

<i>b</i>

.Hỏi <i>a</i>có thể nhận những
giá trị nào trong các giá trị sau:


11; 2002; 2003; 11570; 22789; 29563; 299537


<i>a</i> <i>a</i> <i>a</i> <i>a</i> <i>a</i> <i>a</i> <i>a</i>


<b>Câu 4. (3 điểm) </b>


</div>
<span class='text_page_counter'>(17)</span><div class='page_container' data-page=17>

b) Cho <i>A</i> 1 2012 2012 2 2012320124 ... 2012 71201272và


73


2012 1.


<i>B</i>  So sánh A và B


<b>Câu 5. (6 điểm) </b>


Cho góc bẹt <i>xOy</i>,trên tia <i>Ox</i>lấy điểm A sao cho <i>OA</i>2<i>cm</i>,trên tia <i>Oy</i>lấy
hai điểm M và B sao cho <i>OM</i> 1<i>cm OB</i>, 4<i>cm</i>.


a) Chứng tỏ: Điểm M nằm giữa hai điểm O và B; Điểm M là trung điểm của
đoạn thẳng <i>AB</i>


b) Từ O kẻ hai tia <i>Ot Oz</i>, sao cho <i>tOy</i>130 ,0 <i>zOy</i>30 .0 Tính số đo <i>tOz</i>


<b>ĐÁP ÁN </b>
<b>Câu 1. </b>






5
5



2 . 7 1


17 8 4


)55 ) )


2 2 . 25 3 22 11


<i>a</i> <i>b</i> <i>c</i>   




<b>Câu 2. </b>


12 <sub>7</sub>


) 25 ) )


26 2


<i>x</i>


<i>a x</i> <i>b</i> <i>c x</i>


<i>x</i>





 <sub>  </sub> 





<b>Câu 3. </b>


1) a) <i>A</i> 50


b) <i>A cho A</i>2 5, không chia hết cho 3


c) A có 6 ước tự nhiên và có 12 ước nguyên.
2) Ta có: 45 9.5 mà

 

5,9 1


Do 24 68 45<i>a</i> <i>b</i> suy ra 24 68 5 0
5


<i>b</i>
<i>a</i> <i>b</i>


<i>b</i>





  <sub></sub>


Th1: <i>b</i>0ta có số 24 680<i>a</i>


Để 24 680 9<i>a</i> thì

2 4    <i>a</i> 6 8 0 9

 <i>a</i> 20 9 <i>a</i> 7
Th2: <i>b</i>5ta có số 24 685<i>a</i>



</div>
<span class='text_page_counter'>(18)</span><div class='page_container' data-page=18>

Vậy 7, 0
2, 5


<i>a</i> <i>b</i>


<i>a</i> <i>b</i>


 




  


3) Số nguyên có dạng <i>a</i>3<i>b</i>7

<i>b</i>

hay a là số chia 3 dư 1
Vậy a có thể nhận những giá trị là <i>a</i>2002;<i>a</i>22789;<i>a</i>29563


<b>Câu 4. </b>


a) Gọi số cần tìm là <i>a</i>


Ta có <i>a</i>chia cho 9 dư 5 <i>a</i> 9<i>k</i>5

<i>k</i>

2<i>a</i>9<i>k</i><sub>1</sub> 1

2<i>a</i>1 9


Ta có <i>a</i>chia cho 7 dư 4 <i>a</i> 7<i>m</i>4

<i>m</i>

2<i>a</i>7<i>m</i><sub>1</sub> 1

2<i>a</i>1 7


Ta có <i>a</i>chia cho 5 dư 3  <i>a</i> 5<i>t</i> 3

<i>t</i>

2<i>a</i>5<i>t</i><sub>1</sub> 1

2<i>a</i>1 5



2<i>a</i> 1 9,7,5



  , mà

9;7;5

1và a là số tự nhiên nhỏ nhất
2<i>a</i> 1 <i>BCNN</i>(9,7,5) 315



    . Vậy <i>a</i>158


b) Ta có: 2012<i>A</i>2012 2012 2 2012320124 .... 201272 201273
Lấy 2012<i>A</i> <i>A</i> 2012731, Vậy


73


73


2012 1


2012 1
2011


<i>A</i>   <i>B</i> 


</div>
<span class='text_page_counter'>(19)</span><div class='page_container' data-page=19>

a) Trên tia <i>Oy</i>ta có: <i>OM</i> 1<i>cm OB</i> 4<i>cm</i><i>M</i>là điểm nằm giữa O và B


Do M nằm giữa O và B ta có:


 



4 1 3


<i>OM</i> <i>MB</i><i>OB</i><i>MB</i><i>OB OM</i>    <i>cm</i>


Do A thuộc tia Ox, M thuộc tia Oy nên O nằm giữa hai điểm A và M suy ra:
2 1 3( )


<i>OM</i> <i>OA</i><i>MA</i>   <i>cm</i>



Mặt khác do A, B nằm trên hai tia đối nhau , M lại nằm giữa O và B nên suy ra M
nằm giữa A và B, Vậy <i>M</i>là trung điểm của <i>AB</i>.


b) TH1: Tia <i>Ot Oz</i>, trên cùng một nửa mặt phẳng


Do <i>yOt</i> 130 ,0 <i>yOz</i>300 tia Oz nằm giữa hai tia <i>Ot Oy</i>, .
Ta có: <i>tOz</i><i>tOy</i> <i>yOz</i>1300 300 1000


TH2: Tia Ot và tia Oz không nằm trên cùng một nửa mặt phẳng bờ là <i>xy</i>


Suy ra tia Oy nằm giữa 2 tia <i>Ot Oz</i>,


Ta có: <i>tOz</i><i>tOy</i> <i>yOz</i>1300 300 1600


<i><b>x</b></i>

<i><b>y</b></i>



<i><b>t</b></i>



<i><b>z</b></i>



<i><b>z'</b></i>



<i><b>O</b></i>

<i><b>B</b></i>



</div>
<span class='text_page_counter'>(20)</span><div class='page_container' data-page=20>

<b>ĐỀ THI HỌC SINH GIỎI TOÁN 6 </b>
<b>Năm học 2019-2020 </b>


<b>Bài 1. (2 điểm) </b>



a) Tìm <i>x</i>biết:


2


1 1


0


3 4


<i>x</i>


 <sub></sub>  <sub> </sub>


 


 


b) Tìm ,<i>x y</i> biết 2<i>x</i> 624 5 <i>y</i>


<b>Bài 2. (2 điểm) </b>


a) So sánh : 22
45




và 51
103





b) So sánh :


2009
2010


2009 1


2009 1


<i>A</i> 


 và


2010
2011


2009 2


2009 2


<i>B</i> 




<b>Bài 3. (2 điểm) </b>


Tìm số tự nhiên có 3 chữ số, biết rằng khi chia số đó cho các số 25;28;35thì được
các số dư lần lượt là 5;8;15



<b>Bài 4. (2 điểm) </b>


Ba máy bơm cùng bơm vào một bể lớn, nếu dùng cả máy một và máy hai
thì sau 1 giờ 20 phút sẽ đầy bể, dùng máy hai và máy ba thì sau 1 giờ 30 phút sẽ
đầy bể còn nếu dùng máy một và máy ba thì sẽ đầy bể sau 2 giờ 24 phút. Hỏi mỗi
máy bơm được dùng một mình thì bể sẽ đầy sau bao lâu ?


<b>Bài 5. (2 điểm) </b>Cho góc tù <i>xOy</i>.Bên trong góc <i>xOy</i>,vẽ tia Om sao cho góc <i>xOm</i>


bằng 90 và vẽ tia 0 <i>On</i>sao cho góc <i>yOn</i>bằng 90 0


a) Chứng minh <i>xOn</i> <i>yOm</i>


</div>
<span class='text_page_counter'>(21)</span><div class='page_container' data-page=21>

<b>ĐÁP ÁN </b>
<b>Bài 1. </b>


a) Từ giả thiết ta có:


2


1 1 5


1 1 <sub>3</sub> <sub>2</sub> <sub>6</sub>


1 1 1


3 4


3 2 6



<i>x</i> <i>x</i>


<i>x</i>


<i>x</i> <i>x</i>


 <sub> </sub>  <sub></sub>


 


 <sub></sub>  <sub> </sub><sub></sub> <sub></sub><sub></sub>


  <sub></sub>


  <sub></sub> <sub>  </sub> <sub></sub> <sub></sub>


 


 


b) Nếu <i>x</i>0thì 5<i>y</i> 20 624625 5  4 <i>y</i> 4


Nếu <i>x</i>0thì vế trái là số chẵn, vế phải là số lẻ với mọi ,<i>x y</i> : vô lý
Vậy <i>x</i>0,<i>y</i>4


<b>Bài 2. </b>


22 22 1 51 51 22 51 22 51



)


45 44 2 102 101 45 101 45 101


<i>a</i>          






2010
2011


2010 2010 2010
2011 2011 2011


2009 <sub>2009</sub>
2010
2010


2009 2


) 1


2009 2


2009 2 2009 2 2011 2009 2009


2009 2 2009 2 2011 2009 2009



2009 2009 1 <sub>2009</sub> <sub>1</sub>


2009 1


2009 2009 1


<i>b B</i>


<i>B</i>


<i>A</i>




 




   


   


   


 <sub></sub>


  






Vậy <i>A B</i>
<b>Bài 3. </b>


Gọi số tự nhiên phải tìm là <i>x</i>


Từ giả thiết suy ra

<i>x</i>20 25

<i>x</i>20 28

<i>x</i>20 35





20 25;28;35


<i>x</i> <i>BC</i>


  


Tìm được <i>BCNN</i>

25;28;35

700 <i>x</i> 20700.<i>k k</i>



Vì <i>x</i> *và x có ba chữ số suy ra <i>x</i>999 <i>x</i> 20 1019  <i>k</i> 1


20 700 680


<i>x</i> <i>x</i>


    


<b>Bài 4. </b>


Máy 1 và máy 2 bơm 1 giờ 20 phút hay 4



3giờ đầy bể nên một giờ máy một và hai
bơm được 3


</div>
<span class='text_page_counter'>(22)</span><div class='page_container' data-page=22>

Máy 2 và máy 3 bơm 1 giờ 30 phút hay 3


2giờ đầy bể nên một giờ máy hai và ba
bơm được 2


3bể


Máy một và máy ba bơm 2 giờ 24 phút hay 12


5 giờ đầy bể nên một giờ máy 1 và
máy 3 bơm là 5


12bể.


Một giờ cả ba máy bơm được: 3 2 5 : 2 11


4 3 12 12


 <sub> </sub>  <sub></sub>


 


  (bể)


Một giờ:


Máy 3 bơm được 11 3 1



12  4 6bể Máy ba bơm một mình 6 giờ đầy bể
Máy 1 bơm được 11 2 1


12  3 4bểMáy 1 bơm 1 mình 4 giờ đầy bể
Máy 2 bơm được 11 5 1


12 12  2bểMáy 2 bơm một mình 2 giờ đầy bể


<b>Bài 5. </b>


a) Lập luận được: <i>xOm mOy</i> <i>xOy</i>hay 900 <i>mOy</i><i>xOy</i>
<i>yOn nOx</i> <i>xOy</i>hay 900 <i>nOx</i><i>xOy</i><i>xOn</i> <i>yOm</i>
b) Lập luận được:


<i>xOt</i><i>tOy</i><i>xOt</i><i>xOn nOt</i> <i>tOy</i> <i>yOm mOt</i> <i>nOt</i><i>mOt</i>


<i>Ot</i>


 là tia phân giác của <i>mOn</i>


<i><b>y</b></i>


<i><b>x</b></i>
<i><b>O</b></i>


<i><b>m</b></i>


</div>
<span class='text_page_counter'>(23)</span><div class='page_container' data-page=23>

<b>UBND HUYỆN BA VÌ </b>
<b>PHỊNG GD&ĐT BA VÌ </b>



<b>KỲ KHẢO SÁT CHẤT LƯỢNG HỌC SINH GIỎI </b>
<b>Năm học 2017-2018 </b>


<b>Mơn Tốn lớp 6 </b>


Ngày thi : 25/4/2018


<b>Câu 1. </b><i>(6 điểm) </i>Tính:


 



 



22 7 15
2
14


2 2 2 2


)1152 374 1152 374 65


7 5 1 3 5


)


12 6 4 7 12


11.3 .3 9
)



2.3


3 8 15 899
) . . ....


2 3 4 30


<i>a</i>
<i>b</i>


<i>c</i>


<i>d</i>


   


   


<b>Câu 2. </b><i>(3 điểm) </i>


a) Tìm <i>x</i>biết: 2 2 .... 2 221 4


11.13 13.15 19.21 <i>x</i> 231 3


 <sub></sub> <sub></sub> <sub></sub> <sub> </sub> <sub></sub>


 



 


b) Tìm các số nguyên <i>x</i>biết: 1 3 4 3 1


3 35 210 7 5 3


<i>x</i>


    


<b>Câu 3. </b><i>(3 điểm) </i>


Tìm số tự nhiên <i>a</i>nhỏ nhất sao cho <i>a</i>chia cho 3, cho 5, cho 7 được số dư thứ tự là


2;4;6.


<b>Câu 4. </b><i>(6 điểm) </i>


a) Cho <i>xOy</i>1000và <i>xOz</i>60 .0 Tính số đo <i>xOm</i>,biết <i>Om</i>là tia phân giác của
<i>yOz</i>


b) Cho tam giác <i>ABC</i>và một đường thẳng <i>d</i>không đi qua bất kỳ đỉnh nào của


tam giác và cắt cạnh <i>BC</i>của tam giác. Hãy chứng tỏ rằng đường thẳng <i>d</i>cắt
một và chỉ một trong hai cạnh <i>AB</i>và <i>AC</i>của tam giác <i>ABC</i>


<b>Câu 5. </b><i>(2 điểm) </i>


Cho 1 1 1 ... 1 1



31 32 33 59 60


<i>A</i>     


Chứng tỏ rằng: 4
5


</div>
<span class='text_page_counter'>(24)</span><div class='page_container' data-page=24>

<b>ĐÁP ÁN </b>
<b>Bài 1. </b>


 



 

  



)1152 374 1152 374 65
1152 374 1152 374 65


1152 1152 374 374 65


65


7 5 1 3 5


)


12 6 4 7 12


7 5 5 1 3


12 12 6 4 7



1 5 1 3


6 6 4 7


1 3 5 3 23


1


4 7 4 7 28


<i>a</i>
<i>b</i>
   
    
     
 
   
    
   
     

 




22 7 15 29 30
2 2 28
14


29 29 3
2 28 2 28



2 2 2 2


11.3 .3 9 11.3 3


)


2 .3
2.3


3 . 11 3 3 .2
6


2 .3 2 .3


3 8 15 899
) . . ...


2 3 4 30


1.3 2.4 3.5 29.31
. . ...


2.2 3.3 4.4 30.30
1.2.3....29 3.4.5....31


.


2.3.4...30 2.3.4...30
1 31 31



.


30 2 60


<i>c</i>
<i>d</i>
 <sub></sub> 

  


 
<b>Bài 2. </b>


2 2 2 221 4


) ....


11.13 13.15 19.21 231 3


1 1 1 1 1 1 221 4


...


11 13 13 15 19 21 231 3


1 1 221 4


11 21 231 3



10 221 4


231 231 3


4 1
1
3 3
<i>a</i> <i>x</i>
<i>x</i>
<i>x</i>
<i>x</i>
<i>x</i> <i>x</i>
 <sub></sub> <sub> </sub> <sub> </sub> <sub></sub>
 
 
 
<sub></sub>       <sub></sub>  
 
 
<sub></sub>  <sub></sub>  
 
   
     


1 3 44 88


)


3 35 105 210



4 3 1 158 316


7 5 3 105 210


88 316


210 210 210


<i>b</i>


<i>x</i>


  


   


 


88 <i>x</i> 316


   <b>, </b>mà <i>x</i>


89;90;....;315



<i>x</i>


 


<b>Bài 3. </b>



<i>a</i>chia cho 3 dư 2 <i>a</i> 3<i>k</i>   2 <i>a</i> 1 <i>B</i>

 

3


<i>a</i>chia cho 5 dư 4 <i>a</i> 5<i>p</i>   4 <i>a</i> 1 <i>B</i>

 

5


</div>
<span class='text_page_counter'>(25)</span><div class='page_container' data-page=25>



1 3;5;7 ,


<i>a</i> <i>BC</i>


   mà <i>a</i>nhỏ nhất nên <i>a</i>1là


3,5,7

105 1 105 104


<i>BCNN</i>    <i>a</i>  <i>a</i>


<b>Bài 4. </b>


*Học sinh vẽ hình đúng 2 trường hợp cho 0,5 điểm


a) Trường hợp 1: Tia <i>Oy</i>,tia <i>Oz</i>cùng nằm trên một nửa mặt phẳng bờ chứa tia <i>Ox</i>


+) <i>xOz</i><i>xOy</i>

600 1000

Tia <i>Oz</i>nằm giữa hai tia <i>Ox Oy</i>,


0 0 0


60 100 40


<i>xOz</i> <i>zOy</i> <i>xOy</i> <i>zOy</i> <i>yOz</i>



       


+)Tia <i>Om</i>là tia phân giác của


0


0


40


20


2 2


<i>yOz</i>


<i>yOz</i> <i>yOm</i><i>mOz</i>  


+)<i>yOm</i> <i>yOx</i>

200 1000

Tia <i>Om</i>nằm giữa hai tia <i>Oy Ox</i>,


0 0 0


20 100 80


<i>yOm mOx</i> <i>xOy</i> <i>mOx</i> <i>mOx</i>


       


-Trường hợp 2: Tia <i>Oz Oy</i>, nằm trên hai nửa mặt phẳng đối nhau bờ chứa tia <i>Ox</i>



<i><b>x</b></i>


<i><b>m</b></i>



<i><b>z</b></i>


<i><b>y</b></i>



</div>
<span class='text_page_counter'>(26)</span><div class='page_container' data-page=26>

+Ta có : <i>xOy</i><i>xOz</i>1600 1800nên tia <i>Ox</i>nằm giữa hai tia <i>Oy Oz</i>,


0 0 0


100 60 160


<i>yOz</i> <i>zOx</i> <i>xOy</i>


     


Tia <i>Om</i>là tia phân giác <i>yOz</i>nên


0


0


160


80
2


<i>yOm</i> 



0 0



80 100


<i>yOm</i> <i>yOx</i>


   nên tia <i>Om</i>nằm giữa hai tia <i>Oy Ox</i>,


0 0 0


80 100 20


<i>yOm mOx</i> <i>yOx</i> <i>mOx</i> <i>mOx</i>


       


<i><b>m</b></i>



<i><b>x</b></i>



<i><b>z</b></i>


<i><b>y</b></i>



</div>
<span class='text_page_counter'>(27)</span><div class='page_container' data-page=27>

b) Đường thẳng <i>d</i> cắt cạnh <i>BC</i>và <i>B C d</i>,  nên <i>B</i>và <i>C</i>nằm trên hai nửa mặt phẳng
đối nhau bờ là đường thẳng <i>d</i>


TH1: Nếu <i>A</i>thuộc nửa mặt phẳng chứa điểm B thì <i>d</i>cắt cạnh <i>AC</i>mà khơng cắt
cạnh <i>AB</i>


Th2: Nếu <i>A</i>thuộc nửa mặt phẳng chứa C thì <i>d</i>cắt cạnh <i>AB</i>mà không cắt cạnh AC



<b>Bài 5. </b>


1 1 1 1 1 1 1 1


... .... ....


31 32 40 41 42 50 51 60


1 1 1 1 1 1 10 10 10


... .... ...


30 30 40 40 50 50 30 40 50


1 1 1 47 48 4


3 4 5 60 60 5


<i>A</i><sub></sub>     <sub> </sub>     <sub> </sub>   <sub></sub>


     


     


<sub></sub>   <sub> </sub>   <sub> </sub>   <sub></sub>  


     


     



<b>PHỊNG GD&ĐT LÝ NHÂN </b>
<b>ĐỀ CHÍNH THỨC </b>


<b>ĐỀ KIỂM TRA CHẤT LƯỢNG HỌC SINH GIỎI </b>
<b>MƠN TỐN 6 </b>


<i><b>d</b></i>


<i><b>A</b></i>



<i><b>B</b></i>

<i><b>C</b></i>



<i><b>d</b></i>



<i><b>A</b></i>



</div>
<span class='text_page_counter'>(28)</span><div class='page_container' data-page=28>

<b>Năm học 2018-2019 </b>


<i>(Thời gian làm bài: 150 phút, không kể giao đề) </i>


<b>Bài 1. </b><i>(4,0 điểm) </i>


a) Tính:


7 7 1


2012 9 4


5 3 1



9 2012 2


<i>M</i>


 


 


b) So sánh <i>A</i>và <i>B</i>biết: 2010 2011 2012
2011 2012 2010


<i>A</i>   và 1 1 1 .... 1


3 4 5 17


<i>B</i>    


<b>Bài 2. </b><i>(4,0 điểm) </i>


a) Tìm <i>x</i>biết 1 25 2,75 7 3 0,65 7 : 0,07


8 4 <i>x</i> 2 200


 <sub></sub> <sub></sub>  <sub> </sub> <sub></sub> <sub></sub> 


   


   



b) Tìm các số tự nhiên <i>x y</i>, sao cho

 

<i>x y</i>, 1và <sub>2</sub> <sub>2</sub> 7
25


<i>x</i> <i>y</i>


<i>x</i> <i>y</i>


 <sub></sub>




<b>Bài 3. </b><i>(4,0 điểm) </i>


a) Tìm chữ số tận cùng của số <i>P</i>141414 999 234


b) Tìm ba số nguyên dương biết rằng tổng của ba số ấy bằng nửa tích của
chúng


<b>Bài 4.</b><i>(2,0 điểm) </i>


Cho các số nguyên dương <i>a b c d</i>, , , thỏa mãn <i>ab</i><i>cd</i>.Chứng minh rằng


<i>n</i> <i>n</i> <i>n</i> <i>n</i>


<i>A a</i> <i>b</i>  <i>c</i> <i>d</i> là một hợp số với mọi số tự nhiên <i>n</i>


<b>Bài 5.</b><i>(6,0 điểm) </i>


Cho đoạn thẳng <i>AB</i>,điểm <i>O</i>thuộc tia đối của tia <i>AB</i>.Gọi <i>M N</i>, thứ tự là
trung điểm của <i>OA OB</i>, .



a) Chứng tỏ rằng <i>OA OB</i>


b) Chứng tỏ rằng độ dài đoạn thẳng <i>MN</i>không phụ thuộc vào vị trí điểm .<i>O</i>


c) Lấy điểm <i>P</i>nằm ngồi đường thẳng <i>AB</i>.Cho <i>H</i>là điểm nằm trong tam giác
.


<i>ONP</i> Chứng tỏ rằng tia <i>OH</i>cắt đoạn thẳng <i>NP</i>tại một điểm <i>E</i>nằm giữa <i>N</i>


</div>
<span class='text_page_counter'>(29)</span><div class='page_container' data-page=29>

<b>ĐÁP ÁN </b>
<b>Bài 1. </b>


7 7 1


.2012.9.2


7.9.2 7.2012.2 1006.9
2012 9 4


)


5 3 1 5.2012.2 3.9.2 2012.9


.2012.9.2
9 2012 2


7.2021 503.9 9620
5.2012 3.9 1006.9 979



<i>a N</i>
 <sub> </sub> 
  <sub></sub> <sub></sub>
 
 
 
 <sub></sub> <sub></sub> 
 
 

 
 


b) Câu b


1 1 2


1 1 1


2011 2012 2010


1 1 1 1


3


2010 2011 2010 2012


3


1 1 1 1 1 1



... ...


3 4 5 9 10 17


1 1 1


.2 .5 .8 3


2 5 8


<i>A</i>
<i>A</i>
<i>A</i>
<i>B</i>
<i>B</i> <i>B</i>
     
 <sub></sub> <sub> </sub>  <sub> </sub>  <sub></sub>
     
   
 <sub></sub>  <sub> </sub>  <sub></sub>
   
 
     
<sub></sub>  <sub> </sub>   <sub> </sub>   <sub></sub>
     
    


Từ đó suy ra <i>A B</i>
<b>Bài 2.</b>a) câu a.



5 437 7


7 :


8 200 100


5 437 100


7 .


8 200 7


5 437
7
8 14
5 535
8 14
535 5
:
14 8
1
61 .
7
<i>x</i>
<i>x</i>
<i>x</i>
<i>x</i>
<i>x</i>
<i>x</i>


 
 
 




</div>
<span class='text_page_counter'>(30)</span><div class='page_container' data-page=30>

2 2



2 2


7


7 25


25


<i>x</i> <i>y</i>


<i>x</i> <i>y</i> <i>x</i> <i>y</i>


<i>x</i> <i>y</i>


 <sub></sub> <sub></sub> <sub></sub> <sub></sub> <sub></sub>




2 2


7<i>x</i> 25<i>x</i>25<i>y</i>7<i>y</i>



7 25

 

25 7



<i>x</i> <i>x</i>  <i>y</i>  <i>y</i>


Suy ra 7<i>x</i>25và 25 7 <i>y</i>cùng dấu vì <i>x y</i>, là các số tự nhiên


a) Nếu 7<i>x</i>25 0 thì 25 7 <i>y</i>  0 <i>x</i> 4,<i>y</i>4(trái với điều giả sử)
b) Nếu 7<i>x</i>250thì 25 7 <i>y</i>0, Vậy <i>x</i>4,<i>y</i>4


Thử các số tự nhiên <i>y</i>từ 0,1,2,3ta được <i>x</i>4


Cặp số

   

<i>x y</i>,  4,3 ; vai trò của <i>x y</i>, như nhau nên

   

<i>x y</i>,  3,4


<b>Bài 3. </b>


a) <i>P</i>141414 999 234


Chữ số tận cùng của 141414là 6
Chữ số tận cùng của 999 là 9
Chữ số tận cùng của 2 là 34 2


Chữ số tận cùng của <i>P</i>là chữ số tận cùng của tổng

6 9 2 

là 7
b) Gọi 3 số nguyên dương cần tìm là <i>a b c</i>, ,


Ta có:


2


<i>abc</i>


<i>a b c</i>  


Giả sử <i>a</i> <i>b</i> <i>c</i>thì <i>a b c</i>  3<i>c</i>, do đó: 3 6
2


<i>abc</i>


<i>c</i> <i>ab</i>


  


Có các trường hợp sau:
*)<i>ab</i>  6 <i>c</i> 3,5(loại)


*)<i>ab</i>  5 <i>a</i> 1,<i>b</i>5,<i>c</i>4(<i>ktm</i>)


*) 4 1, 4, 5( )


2, 2, 4( )


<i>a</i> <i>b</i> <i>c</i> <i>tm</i>


<i>ab</i>


<i>a</i> <i>b</i> <i>c</i> <i>tm</i>


  





     <sub></sub>


*)<i>ab</i>2(<i>ktm</i>)


*)<i>ab</i>  3 <i>a</i> 1,<i>b</i>3,<i>c</i>8(<i>tm</i>)
*)<i>ab</i> 1 (<i>ktm</i>)


Vậy bộ ba số cần tìm 1,4,5hoặc 2,2,4boặc1,3,8


<b>Bài 4. </b>


Giả sử <i>t</i> 

 

<i>a c</i>, .Đặt <i>a</i><i>a t c</i><sub>1</sub>; <i>c t</i><sub>1</sub> với

<i>a c</i><sub>1</sub>, <sub>1</sub>

1


1 1 1 1


<i>ab cd</i> <i>a bt</i><i>c dt</i><i>a b c d</i>


<i>a c</i><sub>1</sub>, <sub>1</sub>

 1 <i>b c</i><sub>1</sub>,đặt <i>b c k</i> <sub>1</sub> , do đó: <i>d</i> <i>a k</i><sub>1</sub>


</div>
<span class='text_page_counter'>(31)</span><div class='page_container' data-page=31>

11 1



1 1

1


. . . .


<i>n</i> <i>n</i> <i>n</i> <i>n</i> <i>n</i> <i>n</i> <i>n</i> <i>n</i>


<i>n</i> <i>n</i> <i>n</i> <i>n</i>


<i>A a t</i> <i>c k</i> <i>c t</i> <i>a k</i>


<i>A</i> <i>a</i> <i>c</i> <i>k</i> <i>t</i>



   


  


Vì <i>a c t k</i><sub>1</sub>, , ,<sub>1</sub> <sub>1</sub> nguyên dương nên <i>A</i>là hợp số.


<b>Bài 5. </b>


a) Hai tia <i>AO</i>và <i>AB</i>là hai tia đối nhau


Suy ra điểm <i>A</i>nằm giữa điểm O và điểm B
Vậy <i>OA OB</i>


b) Vì <i>M N</i>, lần lượt là trung điểm của <i>OA OB</i>,


Suy ra <i>OM</i> <i>OM</i> <i>MN</i> <i>ON</i> <i>MN</i> <i>ON</i><i>OM</i>




1 1 1 1


2 2 2 2


<i>MN</i>  <i>OB</i> <i>OA</i> <i>OB OA</i>  <i>AB</i>


<i>AB</i>có độ dài không đổi nên <i>MN</i>không đổi


c) Điểm <i>H</i>nằm trong tam giác <i>ONP</i>suy ra <i>H</i>nằm trong góc <i>O</i>



Suy ra tia <i>OH</i>nằm giữa hai tia <i>ON</i>và OP
,


<i>P N</i>là các điểm không trùng O và thuộc các tia <i>ON OP</i>,
Suy ra tia <i>OH</i>cắt đoạn <i>NP</i>tại điểm E nằm giữa <i>N</i>và P.


<b>PHÒNG GIÁO DỤC VÀ ĐÀO TẠO </b>
<b>HUYỆN HƯƠNG SƠN </b>


<b>ĐỀ THI HỌC SINH GIỎI LỚP 6 </b>
<b>NĂM HỌC 2018-2019 </b>


<b>MÔN THI: TỐN </b>
<b>Câu 1. (4,5 điểm) </b>


1) Tính giá tri của các biểu thức sau:


2



)2. 6 24 : 4 2014


<i>a</i> <sub></sub>  <sub></sub>


<i><b>E</b></i>



<i><b>M</b></i>

<i><b>N</b></i>



<i><b>O</b></i>

<i><b>B</b></i>



<i><b>P</b></i>




<i><b>H</b></i>



</div>
<span class='text_page_counter'>(32)</span><div class='page_container' data-page=32>

1 1 7 1


) 1 2 3 : 1 3 4


3 4 12 2


<i>b</i> <sub></sub>    <sub> </sub>   <sub></sub>


   


2) Tìm <i>x</i>,biết: 5 2


6 3


<i>x</i><sub></sub> <i>x</i><sub></sub> <i>x</i>


 


<b>Câu 2. (4,5 điểm) </b>


1) Tìm <i>x</i> ,biết: <i>x</i>

<i>x</i><sub></sub><i>x</i>  

<i>x</i> 1

<sub></sub>

1
2) Tìm các chữ số <i>x y</i>, sao cho 2014<i>xy</i> 42
3) Tìm các số nguyên <i>a b</i>, biết rằng: 1 1


7 2 1


<i>a</i>



<i>b</i>


 


<b>Câu 3. (4,0 điểm) </b>


1) Tìm số tự nhiên <i>n</i>để

<i>n</i>3



<i>n</i>1

là số nguyên tố


2) Cho <i>n</i>7 5 8 4.<i>a</i>  <i>a</i> Biết <i>a b</i> 6và n chia hết cho 9. Tìm <i>a b</i>,
3) Tìm phân số tối giản <i>a</i>


<i>b</i>lớn nhất

<i>a b</i>,  *

sao cho khi chia mỗi phân số


4 6


;


75 165cho


<i>a</i>


<i>b</i>ta được kết quả là số tự nhiên.


<b>Câu 4. (5,0 điểm) </b>


1) Trên tia Ox lấy hai điểm M, N sao cho <i>OM</i> 3<i>cm ON</i>, 7<i>cm</i>
a) Tính độ dài đoạn thẳng <i>MN</i>.



b) Lấy điểm P trên tia Ox, sao cho <i>MP</i>2<i>cm</i>.Tính độ dài đoạn thẳng OP.
c) Trong trường hợp M nằm giữa O và P. Chứng tỏ rằng P là trung điểm của


đoạn thẳng MN


2) Cho 2014 điểm, trong đó khơng có ba điểm nào thẳng hàng. Có bao nhiêu
tam giác mà các đỉnh là 3 trong 2014 đỉnh đó


<b>Câu 5. (2,0 điểm) </b>


1) Cho tổng gồm 2014 số hạng, 1 2<sub>2</sub> 3<sub>3</sub> 4<sub>4</sub> ... 2014<sub>2014</sub>.


4 4 4 4 4


<i>S</i>       Chứng minh


1
2


<i>S</i> 


</div>
<span class='text_page_counter'>(33)</span><div class='page_container' data-page=33>

<b>ĐÁP ÁN </b>
<b>Câu 1. </b>


2



1) )2. 6 24 : 4 2014 2. 36 24 : 4 2014 2020


1 1 7 1 1 1 7 1



) 1 2 3 : 1 3 4 : 1


3 4 12 2 3 4 12 2


5 2 5 2


2)


6 3 6 3


5 2 1


6 3 6


<i>a</i>


<i>b</i>


<i>x</i> <i>x</i> <i>x</i> <i>x</i> <i>x</i> <i>x</i>


<i>x</i>


 <sub></sub> <sub></sub> <sub></sub> <sub></sub><sub></sub> <sub></sub> <sub></sub><sub></sub><sub></sub> <sub></sub>


 


 <sub></sub> <sub></sub>   <sub></sub> <sub></sub>  <sub></sub> <sub></sub>   <sub></sub> <sub></sub>


       



       




 


<sub></sub>  <sub></sub>      


 


   


<b>Câu 2. </b>








1) 1 1 1 1


2 1 1 2 1 1 2 2 1


<i>x</i> <i>x</i> <i>x</i> <i>x</i> <i>x</i> <i>x</i> <i>x</i> <i>x</i>


<i>x</i> <i>x</i> <i>x</i> <i>x</i> <i>x</i> <i>x</i> <i>x</i> <i>x</i>


 <sub></sub>    <sub></sub>       



             


2)2014<i>xy</i>201400<i>xy</i>42.4795 10 <i>xy</i> 42 10 <i>xy</i> 42
Do 0<i>xy</i>100 <i>xy</i>

32;74

. Vậy

     

<i>x y</i>;  3;2 ; 7;4






1 1 2 7 1


3) 2 7 1 14


7 2 1 14 1


<i>a</i> <i>a</i>


<i>a</i> <i>b</i>


<i>b</i> <i>b</i>




       


 


Do <i>a b</i>,  2<i>a</i> 7 <i>U</i>(14)    

1; 2; 7; 14


Vì 2<i>a</i>7lẻ nên 2<i>a</i>   7

7; 1;1;7

 <i>a</i>

0;3;4;7


Từ đó tính được

  

<i>a b</i>, 

0; 3 ; 3; 15 ; 4;13 ; 7;1

 

 

  



<b>Câu 3. </b>



1) Để

<i>n</i>3



<i>n</i>1

là số nguyên tố thì một trong hai thừa số <i>n</i>3;<i>n</i>1phải
bằng 1


Mà <i>n</i>        3 <i>n</i> 1 1 <i>n</i> 1 1 <i>n</i> 0. Khi đó <i>n</i> 3 3là số nguyên tố.
Vậy <i>n</i>0thì

<i>n</i>3



<i>n</i>1

là số ngun tố.


2) Ta có: <i>n</i>7 5 8 4 9<i>a</i>  <i>b</i>      7 <i>a</i> 5 8 <i>b</i> 4 9


 



24 <i>a b</i> 9 <i>a b</i> 3;12


      (vì <i>a</i> <i>b</i> 19)


Mà <i>a b</i>       6 <i>a b</i> 3 <i>a b</i> 12


Kết hợp với <i>a b</i>   6 <i>a</i> 9,<i>b</i>3


3) Ta có: 14: 14 14 , 75


75 75


<i>a</i> <i>b</i>


</div>
<span class='text_page_counter'>(34)</span><div class='page_container' data-page=34>

Tương tự : 16 : 16 16
175


165 165



<i>a</i>


<i>a</i> <i>b</i>


<i>b</i>


<i>b</i> <i>a</i>




 <sub>  </sub>




Để <i>a</i>


<i>b</i>là số lớn nhất thì <i>a UCLN</i> (14,16)2;<i>b</i><i>BCNN</i>(75;165) 825


Vậy 2


825


<i>a</i>
<i>b</i> 


<b>Câu 4. </b>


1)


a) Do M, N cùng thuộc tia Ox mà <i>OM</i> <i>ON</i>nên M nằm giữa hai điểm O và N



3 7 4


<i>OM</i> <i>MN</i> <i>ON</i> <i>MN</i> <i>MN</i> <i>cm</i>


       


b) Th1: Nếu P nằm giữa M và N thì M nằm giữa O và P
3 2 5


<i>OP</i> <i>OM</i> <i>MP</i> <i>cm</i>


     


Th2: Nếu P nằm giữa O và M


3 2 1


<i>OM</i> <i>OP</i> <i>PM</i> <i>OP</i> <i>OM</i> <i>PM</i> <i>cm</i>


        


c) M nằm giữa O và P <i>OP</i>5<i>cm ON</i> 7<i>cm</i>nên P nằm giữa O và N
Suy ra : <i>OP</i><i>PN</i><i>ON</i> 5 <i>PN</i>  7 <i>PN</i> 2<i>cm</i>


Do đó <i>MP</i><i>PN</i>,mà P nằm giữa M và N nên P là trung điểm của MN.
2) Với n điểm, trong đó khơng có ba điểm nào thẳng hàng. Nối các điểm với


nhau cho ta

1


2


<i>n n</i>


đoạn thẳng


Chọn một đoạn thẳng trong

1


2


<i>n n</i>


đoạn thẳng này và từng <i>n</i>2điểm cịn
lại, ta được <i>n</i>2tam giác. Có

1



2


<i>n n</i>


đoạn thẳng nên có


1

<sub>     </sub>

1 2


. 2


2 2


<i>n n</i> <i>n n</i> <i>n</i>


<i>n</i>


  



  tam giác. Tuy nhiên mỗi tam giác được
tính 3 lần (ABC,ACB,BAC)


Do đó số tam giác được tạo thành là:

1



2

: 3

1



2



2 6


<i>n n</i> <i>n</i> <i>n n</i> <i>n</i>




<i><b>O</b></i>

<i><b>M</b></i>

<i><b>P</b></i>

<i><b>N</b></i>

<i><b>x</b></i>



</div>
<span class='text_page_counter'>(35)</span><div class='page_container' data-page=35>

Áp dụng với <i>n</i>2014ta được số tam giác tạo thành:
2014.2013.2012


1359502364


2 


<b>Câu 5. </b>


1) Ta có: 4 1 2 3<sub>2</sub> 4<sub>3</sub> ... 2014<sub>2013</sub> .


4 4 4 4


<i>S</i>      


2 3 2013 2014



2 3 2013 2 3 2013


2 3 2012


1 1 1 1 2014


3 4 1 ...


4 4 4 4 4


1 1 1 1 1 1 1 1


3 1 ... . 1 ... .


4 4 4 4 4 4 4 4


1 1 1 1


4 4 1 ...


4 4 4 4


<i>S</i> <i>S</i> <i>S</i>


<i>S</i> <i>Dat M</i>


<i>M</i>


         



            


       


Ta có: 3 4 4 <sub>2013</sub>1 4 4


4 3


<i>M</i>  <i>M</i> <i>M</i>    <i>M</i> 


Do đó 3 4 4 4 1


3 9 8 2


<i>S</i>    <i>S</i>


2) Nếu n là số có ít hơn 4 chữ số thì <i>n</i>999và <i>S n</i>

 

27
Suy ra <i>n</i><i>S n</i>

 

999 27 1026  2014(<i>ktm</i>)


Mặt khác <i>n</i> <i>n</i> <i>S n</i>( )2014nên n là số có ít hơn 5 chữ số. Vậy n là số có 4 chữ
số, suy ra <i>S n</i>

 

9.436.Do vậy <i>n</i>2014 36 1978 


Vì 1978 2014 19


20


<i>n</i> <i>ab</i>


<i>n</i>



<i>n</i> <i>cd</i>


 


   






*Nếu <i>n</i>19 .<i>ab</i> Ta có: 19<i>ab</i>    

1 9 <i>a b</i>

2014
1910 11<i>a</i> 2<i>b</i> 2014 11<i>a</i> 2<i>b</i> 104 <i>a</i> 2


       




11<i>a</i>104 2 <i>b</i>104 2.9 86    8 10 <i>a a</i>, 2     <i>a</i> 8 <i>b</i> 8 <i>n</i> 1988(<i>tm</i>)
*Nếu <i>n</i>20<i>cd</i>20<i>cd</i>   

2 0 <i>c d</i>

2014


2002 11<i>c</i> 2<i>d</i> 2014 11<i>c</i> 2<i>d</i> 12 <i>c</i> 2


</div>
<span class='text_page_counter'>(36)</span><div class='page_container' data-page=36>

Và 11 12 0 6, 2006( )


1 2 1( )


<i>c</i> <i>d</i> <i>n</i> <i>tm</i>


<i>c</i>



<i>c</i> <i>d</i> <i>ktm</i>


   




 <sub>    </sub>




Vậy <i>n</i>

1988;2006



<b>PHÒNG GIÁO DỤC VÀ ĐÀO TẠO </b>
<b>BẠCH THƠNG </b>


<b>ĐỀ CHÍNH THỨC </b>


<b>KỲ THI CHỌN HỌC SINH GIỎI THCS CẤP HUYỆN </b>
<b>NĂM HỌC 2018-2019 </b>


<b>MƠN THI: TỐN – LỚP 6 </b>
<b>Câu 1. (4,0 điểm) </b>


a) Thực hiện phép tính




10 10
8



540 : 23,7 19,7 42. 132 75 36 7317
2 .13 2 .65


2 .104


<i>A</i>


<i>B</i>


<sub></sub>  <sub></sub>   





b) Chứng minh rằng tổng của 5 số tự nhiên chẵn liên tiếp thì chia hết cho 10,
còn tổng của 5 số tự nhiên lẻ liên tiếp chia cho 10 dư 5


<b>Câu 2. (4,0 điểm) </b>


a) Tổng của hai số nguyên tố có thể bằng 2015hay khơng ? Vì sao ?
b) Tìm tất cả các số nguyên tố <i>p</i> sao cho <i>p</i>11cũng là số nguyên tố.


<b>Câu 3. (4,0 điểm) </b>


a) Tìm <i>x</i>biết:

<i>x</i> 1

 

<i>x</i> 3

 

<i>x</i>  5

....

<i>x</i>99

0
b) Tìm <i>n</i> biết:

3<i>n</i>8

 

<i>n</i>1



<b>Câu 4. (4,0 điểm) </b>


a) Tìm tích 1 1 1 1 1 1 ... 1 1



2 3 4 100


 <sub></sub>  <sub></sub>  <sub></sub>   <sub></sub> 


     


     


b) So sánh <i>A</i>và <i>B</i>biết: 2013.2014 1
2013.2014


<i>A</i>  và 2014.2015 1


2014.2015


<i>B</i> 


<b>Câu 5. (4,0 điểm) </b>


Cho đoạn thẳng <i>AB</i>;điểm O thuộc tia đối của tia <i>AB</i>.Gọi <i>M N</i>, thứ tự là
trung điểm của <i>OA OB</i>,


a) Chứng tỏ <i>OA OB</i>


</div>
<span class='text_page_counter'>(37)</span><div class='page_container' data-page=37></div>
<span class='text_page_counter'>(38)</span><div class='page_container' data-page=38>

<b>ĐÁP ÁN </b>
<b>Câu 1. </b>







10


10 10 10


8 8 8 3


) 540 : 4 42.171 7317
135 7182 7317 0


2 .13. 1 5


2 .13 2 .65 2 .13.6


3
2 .104 2 .8.13 2 .2 .13


<i>a A</i>
<i>A</i>
<i>B</i>


  


   





   



b) Gọi 5 số chẵn liên tiếp là:2 ;2<i>n n</i>2;2<i>n</i>4;2<i>n</i>6;2<i>n</i>8
Tính tổng ta được: 10<i>n</i>20 10


Gọi 5 số lẻ liên tiếp là: 2<i>n</i>1;2<i>n</i>3;2<i>n</i>5;2<i>n</i>7;2<i>n</i>9
Tính tổng được: 10<i>n</i>25 10

<i>n</i> 2

5chia cho 10 dư 5


<b>Câu 2. </b>


a) Tổng của hai số nguyên tố bằng 2015 là số lẻ, nên một trong hai số nguyên
tố phải là 2


Khi đó số kia là 2013, số này là hợp số


Vậy không tồn tại hai số nguyên tố có tổng bằng 2015


b) Nếu <i>p</i>lẻ <i>p</i> 11là số chẵn lớn hơn 11 nên không là số nguyên tố
Suy ra <i>p</i>chẵn  <i>p</i> 2


<b>Câu 3. </b>


a) Ta có:


 

 



 





1 3 5 .... 99 0



1 99 .50


0
2


50 .50 0


50 0 50


<i>x</i> <i>x</i> <i>x</i> <i>x</i>


<i>x</i> <i>x</i>


<i>x</i>


<i>x</i> <i>x</i>


        


  


 


  <sub></sub>


 


    



b) Ta có: 3<i>n</i> 8 3<i>n</i>  3 5 3

<i>n</i> 1

5


Suy ra :

3<i>n</i>8

 

<i>n</i>1

khi

<i>n</i> 1

<i>U</i>(5)  

1; 5


Tìm được: <i>n</i>  

6; 2;0;4



<b>Câu 4. </b>


a) Ta có:




1 1 1 1


1 1 1 ... 1


2 3 4 100


1.2.3.4....99


1 2 3 99 1


. . ...


2 3 4 100 2.3.4....100 100


 <sub></sub>  <sub></sub>  <sub></sub>   <sub></sub> 


     


     





    


</div>
<span class='text_page_counter'>(39)</span><div class='page_container' data-page=39>

b) Ta có:


2013.2014 1 1


1


2013.2014 2013.2014


2014.2015 1 1


1


2014.2015 2014.2015


<i>A</i>


<i>B</i>




  




  



Vì 1 1


2013.2014 2014.2015nên <i>A B</i>


<b>Câu 5. </b>


a) Hai tia <i>OA OB</i>, đối nhau nên điểm <i>A</i>nằm giữa hai điểm <i>O</i>và B,
suy ra <i>OA OB</i>


b) Ta có <i>M</i>và N thứ tự là trung điểm của <i>OA OB</i>, nên ;


2 2


<i>OA</i> <i>OB</i>


<i>OM</i>  <i>ON</i> 


Vì <i>OA OB</i> <i>OM</i> <i>ON</i>


Hai điểm <i>M</i>và N thuộc tia OB mà <i>OM</i> <i>ON</i>nên điểm M nằm giữa hai điểm <i>O</i>và
N


c) Ta có: <i>OM</i> <i>MN</i> <i>ON</i><i>MN</i> <i>ON</i><i>OM</i>


Hay


2 2


<i>OB OA</i> <i>AB</i>



<i>MN</i>   


Vì <i>AB</i>có độ dài khơng đổi nên <i>MN</i>có độ dài khơng đổi.


<b>PHỊNG GIÁO DỤC VÀ ĐÀO TẠO </b>
<b>PHÚC THỌ </b>


<b>ĐỀ THI HỌC SINH GIỎI </b>
<b>MƠN TỐN 6 </b>


<b>NĂM HỌC 2018-2019 </b>
<b>Bài 1. </b>Thực hiện phép tính (tính nhanh nếu có thể)


2 99 100 100


5 5 10 2 6 10


) : :


7 11 3 7 11 3


1 1 1 1 1


) 1 ...


2 2 2 2 2


<i>a M</i>



<i>b P</i>


   


       


<sub></sub>  <sub> </sub> <sub> </sub>  <sub> </sub> <sub></sub>


       


      


<b>Bài 2. </b>Tìm <i>y</i>biết:


<i><b>M</b></i>

<i><b><sub>N</sub></b></i>



</div>
<span class='text_page_counter'>(40)</span><div class='page_container' data-page=40>

1 2 1


) 3 2 .2 5


2 3 3


1 2 2


) . : : 255


3 9 7


<i>a</i> <i>y</i>



<i>b y</i> <i>y</i>


 <sub></sub>  <sub></sub>


 


 


 


<b>Bài 3. </b>Xe máy thứ nhất đi từ A đến B mất 4 giờ, xe thứ hai đi từ B đến A mất 3
giờ. Nếu hai x khởi hành cùng một lúc từ A và B thì sao 1,5 giờ hai xe sẽ còn cách
nhau 15km (hai xe chưa gặp nhau). Tính quãng đường AB


<b>Bài 4. </b>Trên một nửa mặt phẳng bờ chứa tia <i>Ox</i>,vẽ hai tia <i>Oy Oz</i>, sao cho


0 0


100 , 20


<i>xOy</i> <i>xOz</i>


a) Trong 3 tia <i>Ox Oy Oz</i>, , tia nào nằm giữa hai tia cịn lại ? Vì sao ?
b) Vẽ <i>Om</i>là tia phân giác của <i>yOz</i>.Tính góc <i>xOm</i>?


<b>Bài 5. </b>


a) Cho <i>A</i>    1 3 32 33 ... 3 201132012.Chứng minh rằng

4<i>A</i>1

là lũy
thừa của 3



b) Chứng minh rằng: 11...1222....2


<i>n</i> <i>n</i>


</div>
<span class='text_page_counter'>(41)</span><div class='page_container' data-page=41>

<b>ĐÁP ÁN </b>
<b>Bài 1. </b>




99 100 100 99 99


5 5 10 2 6 10


) : :


7 11 3 7 11 3


5 5 3 2 6 3


. .


7 11 10 7 11 10


3 5 5 2 6 3


. . 1 1 0


10 7 11 7 11 10


1 1 1 1 1



) 2 2 2 2


2 2 2 2 2


<i>a M</i>


<i>b P</i> <i>P</i> <i>P</i>


   
       
<sub></sub>  <sub> </sub> <sub> </sub>  <sub> </sub> <sub></sub>
       
 
   
<sub></sub>  <sub></sub> <sub></sub>  <sub></sub>
 
   

 
 <sub></sub>     <sub></sub>   
   
     
  <sub></sub>  <sub> </sub>  <sub> </sub>  <sub></sub> 
     
<b>Bài 2. </b>
3
)
4



<i>a y</i> 


1 2 2


) . : : 255


3 9 7


1 9 7


. . . 255


3 2 2


3 7


. . 255


2 2


3 7


. 255


2 2


.5 255 255: 5 51


.... 51



<i>b y</i> <i>y</i>


<i>y</i> <i>y</i>
<i>y</i> <i>y</i>
<i>y</i>
<i>y</i> <i>y</i>
<i>Vay</i> <i>y</i>
 
 
 
 <sub></sub> <sub></sub>
 
 
   

<b>Bài 3. </b>


Mỗi giờ xe thứ nhất đi được: 1: 4 1
4


 (quãng đường AB)
Mỗi giờ xe thứ hai đi được: 1: 3 1


3


 (quãng đường AB)
Sau 1,5giờ cả hai xe đi được 1 1 .1,5 7


4 3 8



 <sub></sub>  <sub></sub>


 


  (quãng đường AB)


Phân số chỉ 15<i>km</i>là: 1 7 1


8 8


</div>
<span class='text_page_counter'>(42)</span><div class='page_container' data-page=42>

Quãng đường AB là: 15 :1 120( )


8 <i>km</i>


<b>Bài 4. </b>


a) <i>Oz</i>là tia nằm giữa hai tia <i>Ox Oy</i>, (vì trên cùng nửa mặt phẳng bờ chứa tia


<i>Ox</i>ta thấy <i>xOz</i><i>xOy</i>

200 800



b) Tính được : <i>zOy</i>80 ,0 tính được <i>zOm</i>400


Tính được: 0 0 0


40 20 60


<i>xOm</i>  
<b>Bài 5. </b>


2 3 2011 2012



2 3 4 2011 2012 2013


2 3 2011 2012


2013


) 1 3 3 3 ... 3 3


4 1 3 1 3 3 3 3 ... 3 3 3 1


3 3 3 ... 3 3 1


3


<i>a A</i>


<i>A</i> <i>A</i> <i>A</i>


      


            


      




)111....1.222....2 111....1.0000.00 222....2 111....1. 1000.00 2


111..11.10...2 111..11.3.33....4 33...3.333..34



<i>n</i> <i>n</i> <i>n</i> <i>n</i> <i>n</i> <i>n</i> <i>n</i>


<i>n</i> <i>n</i>


<i>n</i> <i>n</i> <i>n</i> <i>n</i>


<i>b</i>      


 


  


<i><b>y</b></i>



<i><b>x</b></i>


<i><b>z</b></i>


<i><b>m</b></i>



20



</div>
<span class='text_page_counter'>(43)</span><div class='page_container' data-page=43>

Vậy số trên là tích của hai số tự nhiên liên tiếp


<b>ĐỀ THI HỌC SINH GIỎI TOÁN 6 </b>
<b>Năm học 2019-2020 </b>


<b>Bài 1. (2 điểm) </b>


c) Tìm <i>x</i>biết:



2


1 1


0


3 4


<i>x</i>


 <sub></sub>  <sub> </sub>


 


 


d) Tìm ,<i>x y</i> biết 2<i>x</i> 624 5 <i>y</i>


<b>Bài 2. (2 điểm) </b>


c) So sánh : 22
45




và 51
103





d) So sánh :


2009
2010


2009 1


2009 1


<i>A</i> 


 và


2010
2011


2009 2


2009 2


<i>B</i> 




<b>Bài 3. (2 điểm) </b>


Tìm số tự nhiên có 3 chữ số, biết rằng khi chia số đó cho các số 25;28;35thì được
các số dư lần lượt là 5;8;15


<b>Bài 4. (2 điểm) </b>



Ba máy bơm cùng bơm vào một bể lớn, nếu dùng cả máy một và máy hai
thì sau 1 giờ 20 phút sẽ đầy bể, dùng máy hai và máy ba thì sau 1 giờ 30 phút sẽ
đầy bể cịn nếu dùng máy một và máy ba thì sẽ đầy bể sau 2 giờ 24 phút. Hỏi mỗi
máy bơm được dùng một mình thì bể sẽ đầy sau bao lâu ?


<b>Bài 5. (2 điểm) </b>Cho góc tù <i>xOy</i>.Bên trong góc <i>xOy</i>,vẽ tia Om sao cho góc <i>xOm</i>


bằng 90 và vẽ tia 0 <i>On</i>sao cho góc <i>yOn</i>bằng 90 0


c) Chứng minh <i>xOn</i> <i>yOm</i>


</div>
<span class='text_page_counter'>(44)</span><div class='page_container' data-page=44></div>
<span class='text_page_counter'>(45)</span><div class='page_container' data-page=45>

<b>ĐÁP ÁN </b>
<b>Bài 1. </b>


c) Từ giả thiết ta có:


2


1 1 5


1 1 <sub>3</sub> <sub>2</sub> <sub>6</sub>


1 1 1


3 4


3 2 6


<i>x</i> <i>x</i>



<i>x</i>


<i>x</i> <i>x</i>


 <sub> </sub>  <sub></sub>


 


 <sub></sub>  <sub> </sub><sub></sub> <sub></sub><sub></sub>


  <sub></sub>


  <sub></sub> <sub>  </sub> <sub></sub> <sub></sub>


 


 


d) Nếu <i>x</i>0thì 5<i>y</i> 20 624625 5  4 <i>y</i> 4


Nếu <i>x</i>0thì vế trái là số chẵn, vế phải là số lẻ với mọi ,<i>x y</i> : vô lý
Vậy <i>x</i>0,<i>y</i>4


<b>Bài 2. </b>


22 22 1 51 51 22 51 22 51


)



45 44 2 102 101 45 101 45 101


<i>a</i>          






2010
2011


2010 2010 2010
2011 2011 2011


2009 <sub>2009</sub>
2010
2010


2009 2


) 1


2009 2


2009 2 2009 2 2011 2009 2009


2009 2 2009 2 2011 2009 2009


2009 2009 1 <sub>2009</sub> <sub>1</sub>



2009 1


2009 2009 1


<i>b B</i>


<i>B</i>


<i>A</i>




 




   


   


   


 <sub></sub>


  





Vậy <i>A B</i>


<b>Bài 3. </b>


Gọi số tự nhiên phải tìm là <i>x</i>


Từ giả thiết suy ra

<i>x</i>20 25

<i>x</i>20 28

<i>x</i>20 35





20 25;28;35


<i>x</i> <i>BC</i>


  


Tìm được <i>BCNN</i>

25;28;35

700 <i>x</i> 20700.<i>k k</i>



Vì <i>x</i> *và x có ba chữ số suy ra <i>x</i>999 <i>x</i> 20 1019  <i>k</i> 1


20 700 680


<i>x</i> <i>x</i>


    


<b>Bài 4. </b>


Máy 1 và máy 2 bơm 1 giờ 20 phút hay 4


3giờ đầy bể nên một giờ máy một và hai
bơm được 3



</div>
<span class='text_page_counter'>(46)</span><div class='page_container' data-page=46>

Máy 2 và máy 3 bơm 1 giờ 30 phút hay 3


2giờ đầy bể nên một giờ máy hai và ba
bơm được 2


3bể


Máy một và máy ba bơm 2 giờ 24 phút hay 12


5 giờ đầy bể nên một giờ máy 1 và
máy 3 bơm là 5


12bể.


Một giờ cả ba máy bơm được: 3 2 5 : 2 11


4 3 12 12


 <sub> </sub>  <sub></sub>


 


  (bể)


Một giờ:


Máy 3 bơm được 11 3 1


12  4 6bể Máy ba bơm một mình 6 giờ đầy bể


Máy 1 bơm được 11 2 1


12  3 4bểMáy 1 bơm 1 mình 4 giờ đầy bể
Máy 2 bơm được 11 5 1


12 12  2bểMáy 2 bơm một mình 2 giờ đầy bể


<b>Bài 5. </b>


c) Lập luận được: <i>xOm mOy</i> <i>xOy</i>hay 900 <i>mOy</i><i>xOy</i>
<i>yOn nOx</i> <i>xOy</i>hay 900 <i>nOx</i><i>xOy</i><i>xOn</i> <i>yOm</i>
d) Lập luận được:


<i>xOt</i><i>tOy</i><i>xOt</i><i>xOn nOt</i> <i>tOy</i> <i>yOm mOt</i> <i>nOt</i><i>mOt</i>


<i>Ot</i>


 là tia phân giác của <i>mOn</i>


<i><b>y</b></i>


<i><b>x</b></i>
<i><b>O</b></i>


<i><b>m</b></i>


</div>
<span class='text_page_counter'>(47)</span><div class='page_container' data-page=47>

<b>ĐỀ THI HỌC SINH GIỎI CẤP TRƯỜNG </b>
<b>NĂM HỌC 2018-2019 </b>


<b>Môn: Toán 6 </b>


<b>Câu 1. (2,0 điểm) </b>


Cho <i>A</i> 2 22 23 24 ... 2 20. Tìm chữ số tận cùng của A


<b>Câu 3. (1,5 điểm) </b>


Chứng minh rằng: <i>n n</i>

1 2



<i>n</i>1 3



<i>n</i>1 4



<i>n</i>1

chia hết cho 5 với mọi số tự
nhiên n


<b>Câu 4. (1,0 điểm) </b>


Tìm tất cả các số nguyên tố <i>p</i>và q sao cho các số 7<i>p</i><i>q</i>và <i>pq</i>11cũng là
các số nguyên tố.


<b>Câu 5. (1,5 điểm) </b>


a) Tìm <i>UCLN</i>(7<i>n</i>3,8<i>n</i>1)

<i>n</i> * .

Tìm điều kiện của n để hai số đó ngun
tố cùng nhau.


b) Tìm hai số tự nhiên biết: Hiệu của chúng bằng 84, UCLN của chúng bằng 28
và các số đó khoảng từ 300 đến 400


<b>Câu 6. (1,0 điểm) </b>


Tìm các số nguyên <i>x y</i>, sao cho : <i>xy</i>2<i>x</i>  <i>y</i> 6


<b>Câu 7. (2,0 điểm) </b>


Cho <i>xAy</i>, trên tia <i>Ax</i>lấy điểm B sao cho <i>AB</i>5<i>cm</i>.Trên tia đối của tia <i>Ax</i>



lấy điểm D sao cho <i>AD</i>3<i>cm C</i>, là một điểm trên tia Ay
a) Tính BD


</div>
<span class='text_page_counter'>(48)</span><div class='page_container' data-page=48>

<b>ĐÁP ÁN </b>
<b>Câu 1. </b>


2 3 4 20

2 3 21


21 21


.2 2 2 2 2 ... 2 .2 2 2 .... 2


2 2 2 2 2


<i>A</i>


<i>A</i> <i>A</i> <i>A</i>


         


      


Ta có: 22124.5 1 

 

24 5.2 16 .2 5


5


...16 có tận cùng là 6 nên 16 .2 có tận cùng là 2 nên 5 <i>A</i>2212có tận cùng là 0


<b>Câu 3. </b>



Với mọi số tự nhiên <i>n</i>ta có các trường hợp sau:


Th1: <i>n</i> 5thì tích chia hết cho 5
Th2:<i>n</i>chia cho 5 dư 1 thì <i>n</i>5<i>k</i>1


4<i>n</i> 1 20<i>k</i> 5


    chia hết cho 5 tích chia hết cho 5
Th3: n chia cho 5 dư 2 thì <i>n</i>5<i>k</i>2


2<i>n</i> 1 10<i>k</i> 5


    chia hết cho 5tích chia hết cho 5
Th4: n chia cho 5 dư 3 thì <i>n</i>5<i>k</i>3


3<i>n</i> 1 15<i>k</i> 10


    chia hết cho 5tích chia hết cho 5
Th5: n chia cho 5 dư 4 thì <i>n</i>5<i>k</i>4


1 5 5


<i>n</i> <i>k</i>


    chia hết cho 5tích chia hết cho 5


Vậy <i>n n</i>

1 2



<i>n</i>1 3



<i>n</i>1 4



<i>n</i>1

chia hết cho 5 với mọi số tự nhiên n


<b>Câu 4. </b>Nếu <i>pq</i>11là số ngun tố thì nó phải là số lẻ (vì <i>pq</i> 11 2)



<i>pq</i>


 là số chẵn ít nhất 1 trong 2 số phải chẵn, tức là bằng 2
+giả sử <i>p</i>2.Khi đó 7<i>p</i> <i>q</i> 14<i>q pq</i>;  11 2<i>q</i>11


</div>
<span class='text_page_counter'>(49)</span><div class='page_container' data-page=49>

Vậy 2, 3
3, 2


<i>p</i> <i>q</i>


<i>p</i> <i>q</i>


 




  


<b>Câu 5. </b>


a) Gọi <i>UCLN</i>(7<i>n</i>3,8<i>n</i> 1) <i>d</i>với <i>n</i> *
Ta có: 7<i>n</i>3 ,8<i>d n</i>1 <i>d</i>


 



8. 7<i>n</i> 3 7. 8<i>n</i> 1 <i>d</i> 31<i>d</i> <i>d</i> 1;31


      



Để hai số đó nguyên tố cùng nhau thì <i>d</i> 31
Mà 7<i>n</i>3 317<i>n</i> 3 31 317

<i>n</i>4 31



4 31


<i>n</i>


  (vì 7 và 31 nguyên tố cùng nhau) <i>n</i> 31<i>k</i>4

<i>k</i>



Do đó <i>d</i> 31 <i>n</i> 31<i>k</i>4


Vậy hai số 7<i>n</i>3,8<i>n</i>1nguyên tố cùng nhau khi <i>n</i>31<i>k</i>4

<i>k</i>



b) Gọi hai số phải tìm là <i>a b a b</i>,

,  *,<i>a</i><i>b</i>



Ta có: ( , ) 28 28

, *,

 

, 1



28


<i>a</i> <i>k</i>


<i>UCLN a b</i> <i>k q</i> <i>k q</i>


<i>b</i> <i>q</i>





 <sub> </sub>  





Ta có: <i>a b</i> 84  <i>k</i> <i>q</i> 3


Theo bài ra : 300  <i>b</i> <i>a</i> 440   10 <i>q</i> <i>k</i> 16


Chỉ có 2 số 11, 14 nguyên tố cùng nhau và có hiệu là 3 <i>q</i> 11,<i>k</i>14
28.11 308


28.14 392


<i>a</i>
<i>b</i>


 




  <sub></sub> <sub></sub>


 . Vậy hai số phải tìm là 308,392.


<b>Câu 6. </b>






6 1 2 4( , )


<i>xy</i>    <i>x</i> <i>y</i> <i>x</i> <i>y</i>   <i>x y</i>



1 1 1 2 2 4 4


2 4 4 2 2 1 1


0 2 1 3 3 5


6 2 4 0 3 1


<i>x</i>
<i>y</i>
<i>x</i>
<i>y</i>


   


   


 




</div>
<span class='text_page_counter'>(50)</span><div class='page_container' data-page=50>

a) Vì <i>B</i><i>Ax D</i>, tia đối tia Ax<i>A</i>nằm giữa D và B
5 3 8


<i>BD</i> <i>BA</i> <i>AD</i> <i>cm</i>


     


b) Vì A nằm giữa D và B nên tia CA nằm giữa hai tia <i>CB CD</i>,



0 0 0


85 50 35


<i>ACD</i> <i>ACB</i> <i>BCD</i> <i>ACD</i> <i>BCD</i> <i>ACB</i>


        


c) *Trường hợp 1: K thuộc tia Ax


Chứng minh được K nằm giữa A và B


5 1 4( )


<i>AK</i> <i>KB</i> <i>AB</i> <i>KB</i> <i>AB</i> <i>AK</i> <i>cm</i>


        


*Trường hợp 2: K thuộc tia đối của tia <i>Ax</i>


-Lập luận chỉ ra được A nằm giữa K và B
Suy ra : <i>KB</i><i>KA</i><i>AB</i><i>KB</i>  5 1 6<i>cm</i>


Vậy <i>KB</i>4<i>cm</i>hoặc <i>KB</i>6<i>cm</i>


<b>PHÒNG GIÁO DỤC VÀ ĐÀO TẠO </b>
<b>HUYỆN HOẰNG HÓA </b>


<b>ĐỀ THI HỌC SINH GIỎI LỚP 6 </b>
<b>NĂM HỌC 2018-2019 </b>



<i><b>y</b></i>


<i><b>C</b></i>



<i><b>D</b></i>

<i><b>A</b></i>

<i><b>B</b></i>



<i><b>x</b></i>


<i><b>B</b></i>



<i><b>D</b></i>

<i><b>A K</b></i>



<i><b>x</b></i>


<i><b>B</b></i>



</div>
<span class='text_page_counter'>(51)</span><div class='page_container' data-page=51>

<b>MÔN TỐN 6 </b>
<b>Bài 1. </b>Tính giá trị các biểu thức sau:


 







2


2 3


2 5 1


) : 5 . 3



3 6 18


) 3. 5. 5 2 :11 16 2015


1 1 1 1


) 1 1 1 ... 1


1.3 2.4 3.5 2014.2016


<i>a A</i>


<i>b B</i>


<i>c C</i>


   


 


 <sub></sub>  <sub></sub> 


     


 <sub></sub> <sub></sub>  <sub></sub>  <sub></sub> <sub></sub>  <sub></sub>


     


<b>Bài 2. </b>



a) Tìm số tự nhiên <i>x</i>biết: 8.6 288 :

<i>x</i>3

2 50


b) Tìm các chữ số <i>x y</i>, để <i>A</i><i>x</i>183<i>y</i>chia cho 2;5 và 9 đều dư 1


c) Chứng tỏ rằng nếu <i>p</i>là số nguyên tố lớn hơn 3 thì <i>p</i>2 1chia hết cho 3


<b>Bài 3. </b>


a) Cho biểu thức : 5

, 3


3


<i>B</i> <i>n</i> <i>n</i>


<i>n</i>


  




Tìm tất cả các giá trị nguyên của n để B nguyên
b) Tìm các số nguyên tố <i>x y</i>, sao cho <i>x</i>2 117 <i>y</i>2


c) Số 2100viết trong hệ thập phân có bao nhiêu chữ số.


<b>Bài 4. </b>


Cho góc <i>xBy</i>55 .0 Trên các tia <i>Bx By</i>, lần lượt lấy các điểm <i>A C A</i>,

<i>B C</i>; <i>B</i>

.
Trên đoạn thẳng <i>AC</i>lấy diểm D sao cho <i>ABD</i>300



a) Tính độ dài AC, biết: <i>AD</i>4<i>cm CD</i>, 3<i>cm</i>
b) Tính số đo của <i>DBC</i>


c) Từ B vẽ tia Bz sao cho <i>DBz</i>90 .0 Tính số đo <i>ABz</i>


<b>Bài 5. </b>


a) Tìm các chữ số <i>a b c</i>, , khác 0 thỏa mãn <i>abbc</i><i>ab ac</i> 7
b) Cho 1

720122015 39294



2


</div>
<span class='text_page_counter'>(52)</span><div class='page_container' data-page=52>

<b>ĐÁP ÁN</b>
<b>Bài 1. </b>


 









2


2 3


2 2 2


2 5 1 2 1 1 1



) : 5 . 3


3 6 18 3 6 2 3


) 3. 5. 5 2 :11 16 2015 3. 5. 33:11 16 2015 2012


1 1 1 1


) 1 1 1 ... 1


1.3 2.4 3.5 2014.2016


2.3.4...2015 . 2.3.4..


2 3 2015


. ...


1.3 2.4 2014.2016


<i>a A</i>


<i>b B</i>


<i>c C</i>


       


 



 <sub></sub>  <sub></sub>     


     


 <sub></sub> <sub></sub>  <sub></sub>  <sub></sub> <sub></sub>  <sub></sub>


     


 



1.2.3....2014 . 3.4.5...2016

 

..2015

10082015


<b>Bài 2. </b>


a) Biến đổi được:

3

2 144 122

12

2 3 12 15( )


3 12 9( )


<i>x</i> <i>x</i> <i>tm</i>


<i>x</i>


<i>x</i> <i>x</i> <i>ktm</i>


  


 


     <sub></sub> <sub></sub>



    


 


b) Do <i>A</i><i>x</i>1831chia cho 9 dư 1     

<i>x</i> 1 8 3 1 1 9

 <i>x</i> 6
Vậy <i>x</i>6;<i>y</i>1


c) Xét số nguyên tố p khi chia cho 3. Ta có: <i>p</i>3<i>k</i>1hoặc




3 2 *


<i>p</i> <i>k</i> <i>k</i>


Nếu <i>p</i>3<i>k</i> 1 <i>p</i>2  1

3<i>k</i>1

2  1 9<i>k</i>26 3<i>k</i>


Nếu <i>p</i>3<i>k</i>2thì <i>p</i>2  1

3<i>k</i>2

2  1 9<i>k</i>2 12<i>k</i>3 3
Vậy <i>p</i>2 1 3


<b>Bài 3. </b>


a) Để B nhận giá trị nguyên thì <i>n</i> 3 <i>U</i>(5)     

1; 3

<i>n</i>

2;2;4;8



b) Với <i>x</i>  2 22 117 121  <i>y</i>2 121 <i>y</i> 11(là số nguyên tố)


Với <i>x</i>2,<i>x</i>là số nguyên tố nên x lẻ  <i>y</i>2 <i>x</i>2 117là số chẵn
Nên y chẵn, kết hợp với y nguyên tố nên <i>y</i>2(<i>ktm</i>)



Vậy <i>x</i>2;<i>y</i>11


c) Ta có: 1030 100010và 2100 1024101030 2 (1)100


</div>
<span class='text_page_counter'>(53)</span><div class='page_container' data-page=53>

Từ (1) và (2) suy ra số 2100viết trong hệ thập phân có 31 chữ số.


<b>Bài 4. </b>


a) Vì D thuộc đoạn thẳng AC nên D nằm giữa A và C
4 3 7


<i>AC</i> <i>AD CD</i> <i>cm</i>


     


b) Chứng minh tia BD nằm giữa hai tia BA, BC nên ta có:


0 0 0


55 30 25


<i>ABC</i><i>ABD DBC</i> <i>DBC</i> <i>ABC</i><i>ABD</i>  
c) Xét hai trường hợp:


- Trường hợp 1: Tia Bz và BD nằm về hai phía nửa mặt phẳng có bờ là AB
nên tia BA nằm giữa 2 tia <i>Bz BD</i>,


Tính được: 0 0 0 0


90 90 30 60



<i>ABz</i>  <i>ABD</i>  


- Trường hợp 2: Tia Bz và BD nằm về cùn nửa mặt phẳng có bờ là AB nên
tia BD nằm giữa hai tia Bz, BA


Tính được: 0 0 0 0


90 90 30 120


<i>ABz</i>  <i>ABD</i>  
<b>Bài 5. </b>


a) Ta có: <i>abbc</i><i>ab ac</i>. .7 (1)




100 7. . . 7 100


7. 100 . 0 10 0 7. 100 10


100 110


100 7. 110 14 16 15


7 7


<i>ab bc</i> <i>ab ac</i> <i>ab</i> <i>ac</i> <i>bc</i>


<i>bc</i> <i>bc</i>



<i>ac</i> <i>Do</i> <i>ac</i>


<i>ab</i> <i>ab</i>


<i>ac</i> <i>ac</i> <i>ac</i>


     


        


         


<i><b>x</b></i>



y


z'
z


<i><b>B</b></i>



<i><b>A</b></i>



</div>
<span class='text_page_counter'>(54)</span><div class='page_container' data-page=54>

Thay vào (1) được: 1 5 1 .15.7<i>bb</i>  <i>b</i> 1005 110 <i>b</i>1050 105 <i>b</i> <i>b</i> 9
Vậy <i>a</i>1,<i>b</i>9,<i>c</i>5


b) Vì 2012, 92 đều là bội của 4 nên 2015


2012 và 92 cũng là bội của 4 94





2015


2012 4 (<i>m m</i> *);92 4<i>n n</i> *


    


Khi đó <sub>2012</sub>2015 <sub>92</sub>94 <sub>4</sub> <sub>4</sub>

 



7 3 7 <i>m</i> 3 <i>n</i>  ...1  ...1 0 vậy
A có tận cùng là 0 nên chia hết cho 10 nên 1

720122015 39294

5


2


<i>A</i> 


<b>TRƯỜNG THCS LÝ NHÂN </b>
<b>TP BUÔN MÊ THUỘC </b>


<b>ĐỀ KHẢO SÁT HỌC SINH GIỎI </b>
<b>CẤP TRƯỜNG 2018-2019 </b>


<b>MƠN TỐN 6 </b>
<b>Câu 1. (1,5 điểm) </b>Thực hiện phép tính


2 3 2012
2014



3 3 3 3


3


24.47 23 <sub>7</sub> <sub>11 1001 13</sub>


) .


9 9 9 9


24 47 23 <sub>9</sub>


1001 13 7 11
1 2 2 2 ... 2


)


2 2


<i>a A</i>


<i>b M</i>


   





  <sub></sub> <sub> </sub> <sub></sub>



    






<b>Câu 2. (2,5 điểm) </b>


a) Cho <i>S</i>       5 52 53 54 55 56 ... 5 2012.Chứng tỏ S chia hết cho 65
b) Tìm số tự nhiên nhỏ nhất sao cho khi chia cho 11 dư 6, chia cho 4 dư 1 và


chia cho 19 dư 11


c) Chứng tỏ: <i>A</i>10<i>n</i> 18<i>n</i>1chia hết cho 27 (với <i>n</i>là số tự nhiên)


<b>Câu 3. (2 điểm) </b>


a) Tìm ,<i>x y</i>nguyên biết: 2<i>x</i>

3<i>y</i> 2

 

3<i>y</i>  2

55
b) Chứng minh rằng:


 

2


2 2 2


1 1 1 1 1


...


4 6 8   <sub>2</sub><i><sub>n</sub></i>  4



</div>
<span class='text_page_counter'>(55)</span><div class='page_container' data-page=55>

a) Vẽ tia OC tạo với tia <i>OA</i>một góc bằng <i>a</i>0,vẽ tia <i>OD</i> tạo với tia OC một góc
bằng

<i>a</i>10

0và với tia OB một góc

<i>a</i>20

0. Tính <i>a</i>


b) Tính góc <i>xOy</i>, biết <i>AOx</i>220và <i>BOy</i>480


c) Gọi OE là tia đối của tia <i>OD</i>,tính số đo góc kề bù với góc <i>xOD</i>khi <i>AOC</i>
bằng <i>a</i>0


<b>Câu 5. (1,5 điểm) </b>Cho <i>A</i>1020121020111020101020098
a) Chứng minh rằng <i>A</i>chia hết cho 24


b) Chứng minh rằng <i>A</i>không phải là số chính phương


<b>ĐÁP ÁN </b>
<b>Câu 1. </b>


a) Đặt <i>A</i><i>B C</i>.
24.47 23 1105


24 47 23 48


1 1 1 1


3 1


1
7 11 1001 13


1 1 1 1 3



9 1


1001 13 7 11
1105


144


<i>B</i>


<i>C</i>


<i>A</i>




 


 


 <sub> </sub> <sub></sub> <sub></sub> 


 


 


 


 <sub></sub> <sub> </sub> <sub></sub> 


 



 


 


b) Đặt <i>A</i>  1 2 22   23 .... 22012 <i>A</i> 220131


Đặt 2014

2013



2 2 2 2 1


<i>B</i>    1


2


<i>M</i>


 


<b>Câu 2. </b>






2 3 2012


2 3 4 2009 2010 2011 2012


2 3 4 2009 2 3 4



) 5 5 5 .... 5


5 5 5 5 ... 5 5 5 5


5 5 5 5 ... 5 . 5 5 5 5


<i>a S</i>     


        


        


5 5  2 53 54

780 65
Vậy S chia hết cho 65


</div>
<span class='text_page_counter'>(56)</span><div class='page_container' data-page=56>

 

 



 

 



6 33 11; 1 28 4; 11 38 19


27 11, 27 4, 27 19


<i>a</i> <i>a</i> <i>a</i>


<i>a</i> <i>a</i> <i>a</i>


     



   


Do <i>a</i>là số tự nhiên nhỏ nhất nên <i>a</i>27nhỏ nhất suy ra:




27 4,11,19 809


<i>a</i> <i>BCNN</i>  <i>a</i>


9


1


) 10 18 1 10 1 9 27


999....9 9 27


9. 11....11 27


<i>n</i> <i>n</i>


<i>n so</i>


<i>n so</i>


<i>c A</i> <i>n</i> <i>n</i> <i>n</i>


<i>n</i>
<i>n</i> <i>n</i>


      
  
 
 <sub></sub>  <sub></sub>
 


Ta biết số n và số có tổng các chữ số bằng n có cùng số dư khi chia cho 9 do đó


111...1 9
<i>n</i>


<i>n</i>


 nên


1


9. 11111...1 9
<i>n so</i>
<i>n</i>
 

 
 


  nên 9. 111...1<i>n</i> 27


<i>n</i>


 





 


  . Vậy <i>A</i> 27


<b>Câu 3. </b>
<b>a)</b>


 







2 3 2 3 2 55


55


3 2 2 1 55 2 1 (1)


3 2


<i>x</i> <i>y</i> <i>y</i>


<i>y</i> <i>x</i> <i>x</i>


<i>y</i>


    





       




Để <i>x</i>nguyên thì 3<i>y</i> 2 <i>U</i>(55)   

1; 11; 55; 5



)3 2 1 1 28


7


)3 2 5 3 7 ( )


3
13


)3 2 11 ( )


3


)3 2 55 19 1


1


)3 2 1 ( )


3


)3 2 11 3 2



53


)3 2 55 ( )


3


<i>y</i> <i>y</i> <i>x</i>


<i>y</i> <i>y</i> <i>y</i> <i>ktm</i>


<i>y</i> <i>y</i> <i>ktm</i>


<i>y</i> <i>y</i> <i>x</i>


<i>y</i> <i>y</i> <i>ktm</i>


<i>y</i> <i>y</i> <i>x</i>


<i>y</i> <i>y</i> <i>ktm</i>


      
      
    
       
     
        

     



Vậy ta có 4 cặp số x, y nguyên thỏa mãn là:


</div>
<span class='text_page_counter'>(57)</span><div class='page_container' data-page=57></div>
<span class='text_page_counter'>(58)</span><div class='page_container' data-page=58>

b) Ta có:
c)

 


     

 



2
2 2 2


2 2 2 2


2 2 2 2


1 1 1 1


...


4 6 8 <sub>2</sub>


1 1 1 1


...


2.2 2.3 2.4 2.


1 1 1 1 1 1 1 1 1


. ... . ...



4 2 3 4 4 1.2 2.3 1


1 1 1 1 1 1 1


. ...


4 1 2 2 3 1


1 1 1


. 1 ( )


4 4
<i>A</i>
<i>n</i>
<i>A</i>
<i>n</i>
<i>A</i>


<i>n</i> <i>n</i> <i>n</i>


<i>A</i>
<i>n</i> <i>n</i>
<i>A</i> <i>dfcm</i>
<i>n</i>
    
    
 
 
 <sub></sub>     <sub></sub> <sub></sub>    <sub></sub>



  <sub></sub> <sub></sub>
 
 <sub></sub>       <sub></sub>

 
 
 <sub></sub>  <sub></sub>
 
<b>Câu 4. </b>


Học sinh tự vẽ hình


a) Do <i>OC OD</i>, nằm trên cùng một nửa mặt phẳng bờ AB và


( 10 )


<i>COD COA a</i>  <i>a</i> nên tia OC nằm giữa hai tia OA và OD


 

0

0


0 0


0 0 0


10 20 180


3 30 180 50


<i>AOC</i> <i>COD</i> <i>DOB</i> <i>AOB</i>



<i>a</i> <i>a</i> <i>a</i>


<i>a</i> <i>a</i>


   


     


    


b) Ta có: <i>AOy</i>1800 <i>BOy</i>1800 480 1320 <i>AOx</i>220
Nên tia <i>Ox</i>nằm giữa hai tia <i>OA</i>và Oy


0 0 0


22 132 110


<i>AOx</i> <i>xOy</i> <i>AOy</i> <i>xOy</i> <i>xOy</i>


       


c) Vì tia <i>OC</i>nằm giữa hai tia <i>OA OD</i>, nên:


0


0 0 0 0 0


10 2 10 2.50 10 110



<i>AOC</i><i>COD</i> <i>AOD</i><i>AOD</i><i>a</i>  <i>a</i>  <i>a</i>   


Vì <i>AOx</i> <i>AOD</i>

220 1100

nên tia <i>Ox</i>nằm giữa hai tia <i>OA OD</i>,


0 0 0 0 0


22 110 110 22 88


<i>AOx</i> <i>xOD</i> <i>AOD</i> <i>xOD</i> <i>xOD</i>


         


Vậy số đo góc kề bù với góc <i>xOD</i>có số đo là: 1800 880 920


<b>Câu 5. </b>


</div>
<span class='text_page_counter'>(59)</span><div class='page_container' data-page=59>





 



3 2009 2008 2007 2006


2009 2008 2007 2006


2009 2008 2007 2006


10 10 10 10 10 8



8.125. 10 10 10 10 8


8. 125. 10 10 10 10 1 8 1


<i>A</i>    


    


 


 <sub></sub>     <sub></sub>


Ta lại có các số: 102012;102011;102010;102009có tổng các chữ số bằng 1, nên các số


2012 2011 2010 2009


10 ;10 ;10 ;10 khi chia cho 3 đều có số dư bằng 1, 8 chia 3 dư 2
Vậy <i>A</i>chia hết cho 3 (2)


 

3,8 1 (3)
Từ

     

1 , 2 , 3 <i>A</i> 24


b) Ta có các số 102012;102011;102010;102009đều có chữ số tận ùng là 0 nên


2012 2011 2010 2009


10 10 10 10 8


<i>A</i>     có chữ số tận cùng là 8



Vậy A không phải là số chính phương vì số chính phương là những số có tận cùng
là 0;1;4;5;6;9


<b>PHỊNG GIÁO DỤC VÀ ĐÀO TẠO </b>
<b>QUỲNH LƯU </b>


<b>ĐỀ THI HSG CẤP HUYỆN NĂM 2018-2019 </b>
<b>Mơn Tốn lớp 6 </b>


<b>Câu 1. (2 điểm) </b>


a) Tính nhanh: 16

27 7.6

 

 94.7 27.99



b) Tính tổng: 2 2 2 ... 2


1.4 4.7 7.10 97.100


<i>A</i>    


<b>Câu 2. (2 điểm) </b>


Cho biểu thức : <i>M</i>    5 52 53 ... 5 80. Chứng tỏ rằng:
a) M chia hết cho 6


b) <i>M</i>khơng phải là số chính phương.


<b>Câu 3. (2 điểm) </b>


a) Chứng tỏ rằng: 2 5


3


<i>n</i>


<i>n</i>
<i>n</i>


 <sub></sub>


 là phân số tối giản


b) Tìm các giá trị nguyên của n để phân số 2 5
3


<i>n</i>
<i>B</i>


<i>n</i>





 có giá trị là số nguyên.


<b>Câu 4. (1 điểm) </b>


</div>
<span class='text_page_counter'>(60)</span><div class='page_container' data-page=60>

<b>Câu 5. (2 điểm) </b>


Trên cùng nửa mặt phẳng bờ chứa tia Ox vẽ 3 tia <i>Oy Oz Ot</i>, , sao cho


0 0 0



30 , 70 , 110 .


<i>xOy</i> <i>xOz</i> <i>xOt</i> 
a) Tính <i>yOz zOt</i>,


b) Trong 3 tia <i>Oy Oz Ot</i>, , tia nào nằm giữa hai tia cịn lại ? Vì sao ?
c) Chứng minh: <i>Oz</i>là tia phân giác của <i>yOt</i>


</div>
<span class='text_page_counter'>(61)</span><div class='page_container' data-page=61>

<b>ĐÁP ÁN </b>
<b>Câu 1. </b>


 





)16 27 7.6 94.7 27.99
16 27 7.6 94.7 27.99
16 27 27.99 7.6 94.7
16 27.100 7.100


16 100. 27 7 16 100.20
2000 16 2016


2 2 2 2


) ...


1.4 4.7 7.10 97.100



2 1 1 1 1 1 1 1 1 2 1 1


. ... .


3 1 4 4 7 7 10 97 100 3 1


<i>a</i>
<i>b A</i>
   
    
    
  
    
  
    
 
 <sub></sub>         <sub></sub> 
 
33
100 50
 <sub> </sub>
 
 
<b>Câu 2. </b>

 


 




2 3 80



2 3 4 79 80


2 2 2 78 2


2 78


) 5 5 5 ... 5


5 5 5 5 .... 5 5


5 5 5 . 5 5 ... 5 . 5 5
30. 1 5 ... 5 30


<i>a M</i>     


      


      


   


b) Ta thấy : <i>M</i>    5 52 53 ... 5 80chia hết cho 5 (1)
Mặt khác, do 52  53 ... 5 80chia hết cho 5 2


2 3 80


5 5 5 ... 5


<i>M</i>



      không chia hết cho 25 (2)


Từ (1) và (2) suy ra <i>M</i>khơng là số chính phương.


<b>Câu 3. </b>


a) Gọi d là ước chung của <i>n</i>3và 2<i>n</i>5với <i>d</i>


3


<i>n</i> <i>d</i>


  và 2<i>n</i>5 <i>d</i>


 



2 <i>n</i> 3 2<i>n</i> 5 <i>d</i> 1<i>d</i> <i>d</i> 1 <i>UC n</i> 3,2<i>n</i> 5 1


          




2 5


( 3,2 5) 1


3


<i>n</i>



<i>UCLN n</i> <i>n</i> <i>n</i>


<i>n</i>




     


 là phân số tối giản.


b) Ta có: 2 5 2

3

1 2 1


3 3 3


<i>n</i>
<i>n</i>


<i>n</i> <i>n</i> <i>n</i>


 


   


</div>
<span class='text_page_counter'>(62)</span><div class='page_container' data-page=62>

Để <i>B</i>có giá trị nguyên thì 1
3


<i>n</i> nguyên


 




1 <i>n</i> 3 <i>n</i> 3 <i>U</i>(1) 1 <i>n</i> 4; 2


          


<b>Câu 4. </b>


Goi số phải tìm là <i>x</i>


Theo bài ra ta có: <i>x</i>2chia hết cho 3,4,5,6 <i>x</i> 2là bội chung của 3,4,5,6
Mà <i>BCNN</i>(3,4,5,6)60  <i>x</i> 2 60<i>n</i>


Do đó <i>z</i>60<i>n</i>2

<i>n</i>1;2;3;...



Mặt khác <i>x</i>11nên lần lượt cho <i>n</i>1;2;3;...Ta thấy <i>n</i>  7 <i>x</i> 418 11
Vậy số nhỏ nhất phải tìm là 418


<b>Câu 5. </b>


a) <i>xOy</i><i>xOz</i>

300 700

Tia Oy nằm giữa hai tia Ox, Oz


0 0 0


70 30 40


<i>yOz</i>


   


0 0




70 110


<i>xOz</i><i>xOt</i>  <i>Oz</i>nằm giữa hai tia Ox, Ot<i>zOt</i>1100700400
b) <i>xOy</i><i>xOt</i>

300 1100

Tia Oy nằm giữa hai tia


0 0 0


, 110 30 80


<i>Ox Ot</i> <i>yOt</i>  


Theo trên, <i>yOz</i>400 <i>yOz</i> <i>yOt</i>

400 800

<i>Oz</i>nằm giữa hai tia Oy, Ot


30



<i><b>O</b></i>

<i><b>x</b></i>



<i><b>y</b></i>


<i><b>z</b></i>



</div>
<span class='text_page_counter'>(63)</span><div class='page_container' data-page=63>

c) Theo trên, tia Oz nằm giữa hai tia <i>Oy Ot</i>, và <i>yOz</i>40 ,0 <i>zOt</i>400 <i>Oz</i>là tia
phân giác của <i>yOt</i>


<b>Câu 6. </b>


Ta có:


2 2 2 2


1 1 1 1 1 1 1



... ...


2 3 4 100 1.2 2.3 99.100


1 1 1 1 1 1 1


... 1 1


1 2 2 3 99 100 100


        


         


<b>SỞ GIÁO DỤC VÀ ĐÀO TẠO </b>
<b>TỈNH ĐỒNG THÁP </b>


<b>KIỂM TRA KHẢO SÁT HỌC SINH GIỎI </b>
<b>Năm học 2018-2019 </b>


<b>Mơn Tốn 6 </b>
<b>Câu 1. (4,0 điểm) </b>Thực hiện phép tính:


<sub>2</sub> <sub>2</sub>

  

9 <sub>2</sub> 6

 

<sub>2</sub> 14 <sub>4</sub>
28 18 29 18


5. 2 .3 . 2 2. 2 .3 .3
1)



5.2 .3 7.2 .3


12 12 12 5 5 5


12 5


158158158


7 289 85 13 169 91


2) 81. : .


4 4 4 6 6 6 <sub>711711711</sub>


4 6


7 289 85 13 169 91


<i>A</i>


<i>B</i>







 <sub></sub> <sub></sub> <sub></sub> <sub></sub> <sub></sub> <sub></sub> 


 



 <sub></sub> <sub></sub>


       


 


<b>Câu 2. (4,0 điểm) </b>


1) So sánh <i>P</i>và Q


Biết : 2010 2011 2012
2011 2012 2013


<i>P</i>   và 2010 2011 2012


2011 2012 2013


<i>Q</i>  


 


2) Tìm hai số tự nhiên a, b biết: <i>BCNN a b</i>( , )420;<i>UCLN a b</i>( , )21và
21


<i>a</i> <i>b</i>


<b>Câu 3. (4,0 điểm) </b>


1) Chứng minh rằng: Nếu 7<i>x</i>4 37<i>y</i> thì 13<i>x</i>18 37<i>y</i>



2) Cho


2 3 4 2012


1 3 3 3 3 3


...


2 2 2 2 2 2


<i>A</i>   <sub> </sub>  <sub> </sub>  <sub> </sub>   <sub> </sub>


        và


2013


3


: 2
2


<i>B</i>   


 


Tính <i>B A</i>
<b>Câu 4. (6,0 điểm) </b>


Cho <i>xAy</i>, trên tia <i>Ax</i>lấy điểm B sao cho <i>AB</i>6<i>cm</i>.Trên tia đối của tia <i>Ax</i>lấy


điểm D sao cho <i>AD</i>4<i>cm</i>


</div>
<span class='text_page_counter'>(64)</span><div class='page_container' data-page=64>

2) Lấy C là một điểm trên tia Ay. Biết <i>BCD</i>80 ,0 <i>BCA</i>45 .0 Tính <i>ACD</i>
3) Biết <i>AK</i> 2<i>cm K</i>

<i>BD</i>

. Tính BK


<b>Câu 5. (2,0 điểm) </b>


1) Tìm các số tự nhiên <i>x y</i>, sao cho: 3 1


9 18


<i>x</i>
<i>y</i>


 


2) Tìm số tự nhiên <i>n</i>để phân số 10 3


4 10


<i>n</i>
<i>B</i>


<i>n</i>





</div>
<span class='text_page_counter'>(65)</span><div class='page_container' data-page=65>

<b>ĐÁP ÁN </b>
<b>Câu 1. </b>



  

 







9 6 14


2 2 2 2 4 <sub>18</sub> <sub>12</sub> <sub>28</sub> <sub>14</sub> <sub>4</sub>
28 18 29 18 28 18 29 18


29 18
30 18 29 18


28 18 28 18


5. 2 .3 . 2 2. 2 .3 .3 <sub>5.2.3 .2</sub> <sub>2.2 .3 .3</sub>
)


5.2 .3 7.2 .3 5.2 .3 7.2 .3


2 .3 . 5.2 1


5.2 .3 2 .3 2.9


2
2 .3 .(5 7.2) 2 .3 . 5 14 9


1 1 1



12. 1


7 289 85
) 81.
1 1
4. 1
7 289
<i>a A</i>
<i>b B</i>
 <sub></sub>
 
 


    
  
 <sub> </sub> <sub></sub> 
 
 

  


1 1 1


5. 1


158.1001001
13 169 91



: .


1 1 1 1 711.1001001


6. 1


85 13 169 91


12 5 158 18 2 324


81. : . 81. .


4 6 711 5 9 5


  <sub></sub> <sub></sub> <sub></sub> 
 
 <sub></sub> <sub></sub>
 
   
 <sub></sub> <sub></sub> <sub></sub> <sub></sub> <sub></sub> <sub></sub> <sub></sub>
 <sub></sub> <sub></sub> <sub></sub> <sub></sub>
 
 
 <sub></sub> <sub></sub>  
 
<b>Câu 2. </b>


a) Ta có:


2010 2011 2012 2010 2011 2012



2011 2012 2013 2011 2012 2013 2011 2012 2013 2011 2012 2013


<i>Q</i>     


       


Lần lượt so sánh từng phân số của P và Q với các tử là : 2011, 2010, 2012 ta thấy


<i>P</i><i>Q</i>


b) Vì ( , ) 21 21 ,

,

1


21


<i>a</i> <i>m</i>


<i>UCLN a b</i> <i>m n</i>


<i>b</i> <i>n</i>





 <sub> </sub> 




Vì <i>BCNN a b</i>( , )420<i>BCNN</i>(21 ,21 )<i>m</i> <i>n</i> 42021.20<i>BCNN m n</i>( , )20
Vì <i>a</i>21 <i>b</i> 21<i>m</i>21 21 <i>n</i>  <i>m</i> 1 <i>n</i>(*)



Trong các trường hợp cần xét chỉ có 4, 5
2, 3
<i>m</i> <i>n</i>
<i>m</i> <i>n</i>
 

  


 là thỏa (*)


Vậy với 4, 5


2, 3
<i>m</i> <i>n</i>
<i>m</i> <i>n</i>
 

  

21.4 84
21.5 105
<i>a</i>
<i>b</i>
 

  <sub></sub> <sub></sub>

<b>Câu 3. </b>



a) Ta có


 



</div>
<span class='text_page_counter'>(66)</span><div class='page_container' data-page=66>

Vì 7<i>x</i>4 37<i>y</i> mà

4,37

 1 4 7

<i>x</i>4<i>y</i>

37


Do đó, từ (*) suy ra: 5 13

<i>x</i>18<i>y</i>

37mà

5,37

1nên:13<i>x</i>18 37<i>y</i>


b) Ta có:


 



2 3 4 2012


2 3 4 2012 2013


1 3 3 3 3 3


... (1)


2 2 2 2 2 2


3 3 3 3 3 3 3


... 2


2 4 2 2 2 2 2


<i>A</i>



<i>A</i>


       


  <sub> </sub> <sub> </sub> <sub> </sub>  <sub> </sub>


       


         


  <sub> </sub> <sub> </sub> <sub> </sub>  <sub> </sub> <sub> </sub>


         


Lấy (2) – (1) ta được:


2013 <sub>2013</sub>


2012


3 3 3 1 3 3 1


2<i>A</i> <i>A</i> 2 4 2 2 <i>A</i> 2 2


 


 <sub> </sub>      


 



Vậy


2013 2013
2014 2012


3 3 5


2 2 2


<i>B</i> <i>A</i>  


<b>Câu 4. </b>


a) Vì B thuộc tia Ax, D thuộc tia đối của tia Ax<i>A</i>nằm giữa D và B
6 4 10( )


<i>BD</i> <i>BA</i> <i>AD</i> <i>cm</i>


     


b) Vì A nằm giữa D và B nên tia CA nằm giữa hai tia <i>CB CD</i>,


<i>ACD</i> <i>ACB</i> <i>BCD</i>


   0 0 0


80 45 35


<i>ACD</i> <i>BCD</i> <i>ACB</i>



     


c) Trường hợp 1: K thuộc tia Ax


<i><b>x</b></i>


<i><b>y</b></i>



<i><b>C</b></i>



</div>
<span class='text_page_counter'>(67)</span><div class='page_container' data-page=67>

Lập luận chỉ ra được K nằm giữa A và B


Suy ra <i>AK</i><i>KB</i> <i>AB</i><i>KB</i> <i>AB</i><i>AK</i>  6 2 4(<i>cm</i>)


Trường hợp 2: K thuộc tia đối của tia Ax
-Lập luận chỉ ra được A nằm giữa K và B
Suy ra : <i>KB</i><i>KA</i><i>AB</i>  6 2 8<i>cm</i>


Vậy KB=4cm hoặc KB=8cm


<b>Câu 5. </b>


a) Từ




3 1 1 3 2 1 3


9 18 9 18 18


2 1 54 1.54 2.27 3.18 6.9



<i>x</i> <i>x</i> <i>x</i>


<i>y</i> <i>y</i> <i>y</i>


<i>x</i> <i>y</i>




      


      


Vì <i>x</i>là số tự nhiên nên 2<i>x</i>1là ước số lẻ của 54.


2<i>x</i>1 1 3 9 27


<i>x</i> 1 2 5 14


<i>y</i> <sub>54 </sub> <sub>18 </sub> <sub>6 </sub> <sub>2 </sub>


Vậy

  

<i>x y</i>;  1;54 ; 2;18 ; 5;6 ; 14;2

 

   



b) 10 3 2,5 22


4 10 4 10


<i>n</i>
<i>B</i>



<i>n</i> <i>n</i>




  


 


Vì <i>n</i> nên 2,5 22
4 10


<i>B</i>


<i>n</i>


 


 đạt GTLN khi


22


4<i>n</i>10đạt GTLN
Mà 22


4<i>n</i>10đạt GTLN khi 4<i>n</i>10là số nguyên dương nhỏ nhất
11


*)4 10 1 ( )


4



*)4 10 2 3


<i>n</i> <i>n</i> <i>ktm</i>


<i>n</i> <i>n</i>


   
   


Vậy <i>GTLN</i>của B là 13,5 khi <i>n</i>3


<b>PHÒNG GIÁO DỤC VÀ ĐÀO TẠO </b>
<b>TƯ NGHĨA </b>


<b>KỲ THI HỌC SINH GIỎI CẤP HUYỆN </b>
<b>Năm học 2018-2019 </b>


<b>Mơn thi:Tốn 6 </b>


<i><b>D</b></i>

<i><b>A</b></i>

<i><b>K</b></i>

<i><b>B</b></i>

<i><b><sub>x</sub></b></i>



</div>
<span class='text_page_counter'>(68)</span><div class='page_container' data-page=68>

<b>Câu 1. (3,0 điểm) </b>Cho 12 1


2 3


<i>n</i>
<i>A</i>


<i>n</i>






 . Tìm giá trị của <i>n</i>để:


a) A là một phân số
b) A là một số nguyên


<b>Câu 2. (4,0 điểm) </b>


a) Khơng quy đồng hãy tính tổng sau:


1 1 1 1 1 1


20 30 42 56 72 90


<i>A</i>          


b) So sánh <i>P</i>và Q biết:
2010 2011 2012
2011 2012 2013


<i>P</i>   và 2010 2011 2012


2011 2012 2013


<i>Q</i>  


 



<b>Câu 3. (3,0 điểm) </b>Tìm <i>x</i>,biết:


3 <sub>5</sub> <sub>2</sub>


) 7 11 2 .5 200


1 3


)3 16 13,25


3 4


<i>a</i> <i>x</i>


<i>b</i> <i>x</i>


  


  


<b>Câu 4. (3,0 điểm) </b>Ở lớp 6A, số học sinh giỏi học kỳ I bằng 3


7số cịn lại. Cuối năm
có thêm 4 học sinh đạt loại giỏi nên số học sinh giỏi bằng 2


3số cịn lại. Tính số học
sinh của lớp 6A.


<b>Câu 5. (2,0 điểm) </b>Cho <i>ababab</i>là số có 6 chữ số, chứng tỏ số <i>ababab</i>là bội của 3



<b>Câu 6. (5,0 điểm) </b>Cho <i>xAy</i>,trên tia <i>Ax</i>lấy điểm B sao cho <i>AB</i>5<i>cm</i>.Trên tia đối
của tia <i>Ax</i>lấy điểm D sao cho <i>AD</i>3<i>cm C</i>, là một điểm trên tia <i>Ay</i>


a) Tính <i>BD</i>.


</div>
<span class='text_page_counter'>(69)</span><div class='page_container' data-page=69>

<b>ĐÁP ÁN </b>
<b>Câu 1. </b>a) 12 1


2 3


<i>n</i>
<i>A</i>


<i>n</i>





 là phân số khi 12 1 ,2 3 ,2 3 0 1,5


<i>n</i>


<i>n</i> <i>n</i> <i>n</i>


<i>n</i>






    <sub>   </sub>


 


b) 12 1 6 17


2 3 2 3


<i>n</i>
<i>A</i>


<i>n</i> <i>n</i>




  


 


<i>A</i>là số nguyên khi 2<i>n</i> 3 <i>U</i>(17)2<i>n</i>   3

1; 17

  <i>n</i>

10; 2; 1;7 



<b>Câu 2. </b>


1 1 1 1 1 1


)


20 30 42 56 72 90



1 1 1 1 1 1


4.5 5.6 6.7 7.8 8.9 9.10


1 1 1 1 1 1 1 1


...


4 5 5 6 6 7 9 10


1 1 3


4 10 20


<i>a A</i>        


 


 <sub></sub>      <sub></sub>


 


 


 <sub></sub>         <sub></sub>


 


 



 <sub></sub>  <sub></sub> 


 


2010 2011 2012 2010 2011


)


2011 2012 2013 2011 2012 2013 2011 2012 2013
2012


2011 2012 2013


<i>b Q</i>    


     




 


Ta có: 2010 2010


2011 2012 2013   2011


2011 2011 2012 2012


;


2011 2012 2013   2012 2011 2012 2013   2013



<i>P</i> <i>Q</i>


 


</div>
<span class='text_page_counter'>(70)</span><div class='page_container' data-page=70>







3 <sub>5</sub> <sub>2</sub>


3


3 <sub>3</sub>


) 7 11 2 .5 200


7 11 800 200


7 11 1000 10


7 11 10 3


<i>a</i> <i>x</i>


<i>x</i>
<i>x</i>



<i>x</i> <i>x</i>


  


   


   


    


1 3


)3 16 13,25


3 4


10 67 53


3 4 4


10 53 67


30


3 4 4


9


<i>b</i> <i>x</i>



<i>x</i>


<i>x</i>
<i>x</i>


  




  




    


  


<b>Câu 4. </b>


Số học sinh giỏi kỳ I bằng 3


10số học sinh cả lớp


Số học sinh giỏi cuối năm bằng 2


5số học sinh cả lớp


4 học sinh là : 2 3 1


510 10(số học sinh cả lớp)


Số học sinh 6A là : 4 : 1 40


10 (học sinh)


<b>Câu 5. </b>


.10101 3


</div>
<span class='text_page_counter'>(71)</span><div class='page_container' data-page=71>

<b>Câu 6. </b>


d) Vì <i>B</i><i>Ax D</i>, tia đối tia Ax<i>A</i>nằm giữa D và B
5 3 8


<i>BD</i> <i>BA</i> <i>AD</i> <i>cm</i>


     


e) Vì A nằm giữa D và B nên tia CA nằm giữa hai tia <i>CB CD</i>,


0 0 0


85 50 35


<i>ACD</i> <i>ACB</i> <i>BCD</i> <i>ACD</i> <i>BCD</i> <i>ACB</i>


        


f) *Trường hợp 1: K thuộc tia Ax


Chứng minh được K nằm giữa A và B



5 1 4( )


<i>AK</i> <i>KB</i> <i>AB</i> <i>KB</i> <i>AB</i> <i>AK</i> <i>cm</i>


        


*Trường hợp 2: K thuộc tia đối của tia <i>Ax</i>


-Lập luận chỉ ra được A nằm giữa K và B
Suy ra : <i>KB</i><i>KA</i><i>AB</i><i>KB</i>  5 1 6<i>cm</i>


Vậy <i>KB</i>4<i>cm</i>hoặc <i>KB</i>6<i>cm</i>


<b>UBND HUYỆN VĨNH LỘC </b>
<b>PHÒNG GD VÀ ĐT </b>


<b>KỲ THI HỌC SINH GIỎI THCS </b>
<b>Mơn: Tốn 6 </b>


<i><b>y</b></i>


<i><b>C</b></i>



<i><b>D</b></i>

<i><b>A</b></i>

<i><b>B</b></i>



<i><b>x</b></i>


<i><b>B</b></i>



<i><b>D</b></i>

<i><b>A K</b></i>




<i><b>x</b></i>


<i><b>B</b></i>



</div>
<span class='text_page_counter'>(72)</span><div class='page_container' data-page=72>

<b>Năm học 2017-2018 </b>
<b>Bài 1. (4,0 điểm) </b>Thực hiện phép tính:


<sub>16</sub>

2
13 11 9


) 1.2.3....9 1.2.3....8 1.2.3...8.8
3.4.2


)


11.2 .4 16


131313 131313 131313


) 70.


565656 727272 909090


<i>a A</i>


<i>b B</i>


<i>c C</i>


  







 


 <sub></sub>   <sub></sub>


 


d) Thực hiện phép tính: 1 1 1 ... 1


4.9 9.14 14.19 64.69


<i>B</i>    


<b>Bài 2. (4,0 điểm) </b>Tìm <i>x</i>biết:




 



 



5 3


1 2 7


) 2



2 3 3


) 3 54 .8 : 4 18


) 2 15 2 15


) 1 2 ... 2013 2035147


<i>a</i> <i>x</i>


<i>b</i> <i>x</i>


<i>c</i> <i>x</i> <i>x</i>


<i>d x</i> <i>x</i> <i>x</i> <i>x</i>


  


 


 


 


  


       


<b>Bài 3. (4,0 điểm) </b>



a) Tìm số tự nhiên nhỏ nhất, biết rằng số đó khi chia cho 3, cho 4, cho 5, cho 6
đều dư là 2, cịn chia cho 7 thì dư 3.


b) Tìm ,<i>x y</i>nguyên biết: <i>x</i> <i>y</i> <i>xy</i>40


c) Khi chia một số tự nhiên <i>a</i>cho 4 ta được số dư là 3 còn khi chia <i>a</i>9 ta được


số dư là 5. Tìm số dư trong phép chia <i>a</i>cho 36


<b>Bài 4. (6,0 điểm) </b>Cho <i>xBy</i>55 .0 Trên các tia <i>Bx By</i>, lần lượt lấy các điểm <i>A C</i>, sao
cho <i>A</i><i>B C</i>, <i>B</i>.Trên đoạn thẳng <i>AC</i>lấy điểm <i>D</i>sao cho <i>ABD</i>300


a) Tính độ dài AC, biết <i>AD</i>4<i>cm CD</i>, 3<i>cm</i>
b) Tính số đo <i>DBC</i>


c) Từ <i>B</i>vẽ tia <i>Bz</i>sao cho <i>DBz</i>900. Tính số đo <i>ABz</i>


<b>Bài 5. (2,0 điểm) </b>


Cho tổng 2<sub>1</sub> 3<sub>2</sub> 4<sub>3</sub> ... 2016<sub>2015</sub> 2017<sub>2016</sub>


2 2 2 2 2


<i>T</i>      


So sánh <i>T</i>với 3


</div>
<span class='text_page_counter'>(73)</span><div class='page_container' data-page=73>








2 2


16 2 16 <sub>36</sub>


13 11 9 13 22 36 35 36
36 36


35 35


) 1.2.3...9 1.2.3..8 1.2.3...8.8
1.2.3...8. 9 1 8 0


3.4.2 3.2 .2 <sub>9.2</sub>


)


11.2 .4 16 11.2 .2 2 11.2 2


9.2 9.2


2
2 . 11 2 2 .9


131313 131313 131313


) 70. 70.



565656 727272 909090


<i>a A</i>
<i>b B</i>
<i>c C</i>
  
   
  
  
  

 
 <sub></sub>   <sub></sub>
 


13 13 13
56 72 90


1 1 1 1 1


70.13. 70.13. 39


7.8 8.9 9.10 7 10


1 1 1 1 1 1 1 1 1 1 1 1 1


) ... . ...


4.9 9.14 14.19 64.69 5 4 9 9 14 14 19 64 69



1 1 1 13


.


5 4 69 276


<i>d B</i>
 <sub></sub> <sub></sub> 
 
 
   
 <sub></sub>   <sub></sub> <sub></sub>  <sub></sub>
   
 
      <sub></sub>        <sub></sub>
 
 
 <sub></sub>  <sub></sub>
 
<b>Bài 2. </b>


1 2 1 2 1 10


) 2 4 2 4 2


2 3 2 3 2 3


1 10 17 17


2 2



2 3 6 12


1 10 23 23


2 2


2 3 6 12


<i>a</i> <i>x</i> <i>x</i> <i>x</i>


<i>x</i> <i>x</i> <i>x</i>


<i>x</i> <i>x</i> <i>x</i>


         
 
      

 
       





) 3 54 .8 : 4 18


3 54 .8 72 3 54 9


3 63 21



</div>
<span class='text_page_counter'>(74)</span><div class='page_container' data-page=74>

 


 


 





5 3
5 3
3 2
3


2 2 <sub>2</sub>


) 2 15 2 15


2 15 2 15 0


2 15 . 2 15 1 0


2 15 0 2 15 0 7,5


2 15 1 8


2 15 1 0 2 15 1


2 15 1 7


7;7,5;8



<i>c</i> <i>x</i> <i>x</i>


<i>x</i> <i>x</i>


<i>x</i> <i>x</i>


<i>x</i> <i>x</i> <i>x</i>


<i>x</i> <i>x</i>
<i>x</i> <i>x</i>
<i>x</i> <i>x</i>
<i>Vay x</i>
  
   
 
 <sub></sub>   <sub></sub>
 <sub></sub> <sub> </sub> <sub>   </sub>

<sub></sub> <sub></sub> <sub>   </sub>
      
 <sub>     </sub>



 




) 1 2 ... 2013 2035147


2014 1 2 3 ... 2013 2035147


2014 2027091 2035147


2014 8056 4


<i>d x</i> <i>x</i> <i>x</i> <i>x</i>


<i>x</i>
<i>x</i>
<i>x</i> <i>x</i>
       
     
 
  
<b>Bài 3. </b>


a) Gọi a là số tự nhiên cần tìm


Vì <i>a</i>chia cho 3,4,5,6 đều dư 2 nên <i>a</i>2chia hết ch0 3,4 ,5,6


<i>a</i> 2

<i>BC</i>

3;4;5;6



   , <i>BCNN</i>(3,4,5,6)60




2 0;60;120;180... 2;62;122;182;...


<i>a</i> <i>a</i>


    



Mà a là số nhỏ nhất và chia cho 7 dư 3  <i>a</i> 122






) 40 1 1 41 1 1 41


<i>b x</i> <i>y</i> <i>xy</i>  <i>y</i> <i>x</i>  <i>y</i>  <i>x</i> <i>y</i> 


Mà <i>x y</i>, nguyên nên <i>x</i>1,<i>y</i>1là ước của 41


Tính được

  

<i>x y</i>, 

40,0 ; 0;40 ;

 

 

 2; 42 ;

 

42; 2


c) Theo đề bài ta có: <i>a</i>4<i>p</i> 3 9<i>q</i>3( ,<i>p q</i> )




 



13 4 3 13 4 4 (1)


13 9 5 13 9 2 2


<i>a</i> <i>p</i> <i>p</i>


<i>a</i> <i>q</i> <i>q</i>


      


     



Từ (1) và (2) ta nhận thấy <i>a</i>13là bội của 4 và 9 mà

 

4,9   1 <i>a</i> 13là bội của
4.9 36


</div>
<span class='text_page_counter'>(75)</span><div class='page_container' data-page=75>

<b>Bài 4. </b>


a) Vì D thuộc đoạn AC nên D nằm giữa A và C


4 3 7( ) 7


<i>AC</i> <i>AD CD</i> <i>cm</i> <i>AC</i> <i>cm</i>


       


b) Chứng minh tia BD nằm giữa hai tia <i>BA BC</i>,
Ta có đẳng thức : <i>ABC</i><i>ABD DBC</i>


0 0 0


55 30 25


<i>DBC</i> <i>ABC</i> <i>ABD</i>


     


c) Xét hai trường hợp (học sinh vẽ hình trong 2 trường hợp)


- Trường hơp 1:Tia Bz và BA nằm trên cùng một nửa mặt phẳng có bờ là
BD


Lập luận tia BA nằm giữa hai tia Bz và BD



Tính được: 0 0 0


90 30 60


<i>ABz</i><i>DBz</i> <i>ABD</i>  


- Trường hợp 2: Tia <i>Bz</i>'và BA nằm trên hai nửa mặt phẳng đối nhau có bờ
là BD


Lập luận tia BD nằm giữa hai tia Bz và BA


Tính được: 0 0 0


90 30 120


<i>ABz</i><i>DBz</i> <i>ABD</i>  
<b>Bài 5. </b>


1 2 3 2015 2016


1 2 2014 2015


2 3 4 2016 2017


...


2 2 2 2 2


3 4 2016 2017



2 2 ...


2 2 2 2


<i>T</i>


<i>T</i>


     


     


1 1 2 2 2014 2014 2015 2015 2016


3 2 4 3 2016 2015 2017 2016 2017


2 2 ....


2 2 2 2 2 2 2 2 2


<i>T</i>   <i>T</i>         


<i><b>C</b></i>



<i><b>B</b></i>



<i><b>D</b></i>



<i><b>A</b></i>

<i><b>x</b></i>




<i><b>y</b></i>


<i><b>z</b></i>



</div>
<span class='text_page_counter'>(76)</span><div class='page_container' data-page=76>

1 2 2015 2016


1 1 1 2017


2 ...


2 2 2 2


<i>T</i>      


Đặt 1<sub>1</sub> 1<sub>2</sub> ... <sub>2015</sub>1 2 1 1<sub>1</sub> 1<sub>2</sub> ... <sub>2014</sub>1


2 2 2 2 2 2


<i>N</i>      <i>N</i>     


2015


1


2 1 1


2


<i>N</i>  <i>N</i>  <i>N</i>



Nên 2 1 2017<sub>2016</sub> 3 2017<sub>2016</sub> 3


2 2


<i>T</i>       <i>T</i>


<b>KIỂM TRA ĐỘI TUYỂN TOÁN 6 </b>
<b>NĂM HỌC 2019-2020 </b>


<b>Câu 1. </b>


a) Tính giá trị biểu thức: 20102010. 7 : 7

10 83.2422010: 22010


b) So sánh hai số: 3210và 2350


<b>Câu 2. </b>


Cho tổng <i>S</i>    1 3 5 ... 2009 2011 
a) Tính <i>S</i>


b) Chứng tỏ S là một số chính phương
c) Tìm các ước nguyên tố khác nhau của <i>S</i>


<b>Câu 3. </b>


a) Tìm giá trị <i>n</i>là số tự nhiên để <i>n</i>7chia hết cho <i>n</i>2


b) Tìm <i>x</i>là số chia trong phép chia 235cho <i>x</i>được số dư là 14


<b>Câu 4. </b>



</div>
<span class='text_page_counter'>(77)</span><div class='page_container' data-page=77>

b) Cho <i>n</i>là số tự nhiên bất kỳ


<b>Chứng</b> minh

<i>n</i>3

2<i>n</i>5

là hai số nguyên tố cùng nhau


<b>Câu 5. </b>


Trong mặt phẳng cho 6 điểm, trong đó khơng có ba điểm nào thẳng hàng
a) Vẽ được bao nhiêu đoạn thẳng đi qua các điểm đã cho


b) Vẽ được bao nhiêu tam giác có đỉnh là các điểm đã cho.


<b>ĐÁP ÁN </b>
<b>Câu 1. </b>




2010 10 8 4 2010 2010 2010


)2010 . 7 : 7 3.2 2 : 2 2010 . 49 3.16 1 0


<i>a</i>      


 

70

 

70


210 3.70 3 70 350 5.70 5 70


)3 3 3 27 ; 2 2 2 32


<i>b</i>      



Vì 2770 3270nên suy ra 32102350


<b>Câu 2. </b>


2


2011 1 2011 1


) 1 3 5 ... 2009 2011 . 1 1006 1012036


2 2


<i>a S</i>       <sub></sub>   <sub> </sub>   <sub></sub> 


   


b) <i>S</i>2 .5032 2 10062là số chính phương
c) S có hai ước nguyên tố là: 2và 503


<b>Câu 3. </b>


 

 





) 7 2 5 2 2 (5) 1; 5


3; 1; 7;3


<i>a x</i> <i>x</i> <i>x</i> <i>x</i> <i>U</i>



<i>x</i>


         


    


)235:


<i>b</i> <i>x</i>dư 14235 14 <i>x x</i>

14

221 <i>x x</i>

14

 <i>x</i>

17;221



<b>Câu 4. </b>


a) Tìm <i>x</i>:


<i>x</i>chia cho 7;8;9dư 2 và <i>x</i>có ba chữ số 

<i>x</i>2 7;8;9

và <i>x</i>có ba chữ số




2 7;8;9


<i>x</i> <i>BC</i>


   và <i>x</i>có ba chữ số  <i>x</i> 504 2 506 


b) Gọi <i>d</i> 

<i>n</i>3,2<i>n</i>5





3 ;2 5 2 3 ,2 5



<i>n</i> <i>d</i> <i>n</i> <i>d</i> <i>n</i> <i>d</i> <i>n</i> <i>d</i>


     


2<i>n</i> 6

 

2<i>n</i> 5

<i>d</i> 1 <i>d</i> <i>d</i> 1


      


Vậy <i>n</i>3và 2<i>n</i>5là hai số nguyên tố cùng nhau.


<b>Câu 5. </b>


</div>
<span class='text_page_counter'>(78)</span><div class='page_container' data-page=78>

b) Số tam giác vẽ được là:

15.4 : 3 20

 (tam giác)


<b>PHÒNG GIÁO DỤC & ĐÀO TẠO </b>
<b>DUY XUYÊN </b>


<b>ĐỀ THI HỌC SINH GIỎI NĂM HỌC 2019-2020 </b>
<b>Mơn Tốn 6 </b>


<b>Câu 1. </b>


a) Tìm các số tự nhiên <i>x y</i>, sao cho

2<i>x</i>1



<i>y</i> 5

12
b) Tìm số tự nhiên <i>n</i>sao cho 4<i>n</i>5chia hết cho 2<i>n</i>1


c) Tìm tất cả các số <i>B</i>62<i>xy</i>427, biết rằng B chia hết cho 99


<b>Câu 2. </b>Tìm <i>x</i>



2 2 3 2 2


)5<i>x</i> 125 )3 <i>x</i> 81 )5 <i>x</i> 2.5 5 .3


<i>a</i>  <i>b</i>  <i>c</i>   


<b>Câu 3. </b>


a) Chứng tỏ rằng: 12 1


30 2


<i>n</i>
<i>n</i>




 là phân số tối giản


b) Chứng minh rằng: 1<sub>2</sub> 1<sub>2</sub> .... 1 <sub>2</sub> 1
2 3  100 


<b>Câu 4. </b>


So sánh các phân số:


23 23232323 2323 232323


; ; ;



99 99999999 9999 999999


<b>Câu 5. </b>


</div>
<span class='text_page_counter'>(79)</span><div class='page_container' data-page=79>

<b>ĐÁP ÁN </b>
<b>Câu 1. </b>


a) Ta có: 2<i>x</i>1;<i>y</i> 5 <i>U</i>(12) 1.12 2.5 3.4
Do 2<i>x</i>1lẻ 2 1 1 0; 17


2 1 3 1; 9


<i>x</i> <i>x</i> <i>y</i>


<i>x</i> <i>x</i> <i>y</i>


    




      <sub></sub>


Vậy

  

<i>x y</i>;  0,17 ; 1,9

  


b) Ta có: 4<i>n</i> 5 2 2

<i>n</i> 1

3


Để 4<i>n</i>5 2<i>n</i> 1 3 2<i>n</i> 1

2<i>n</i> 1

<i>U</i>(3)

 

1;3  <i>n</i>

 

1;2
c) Ta có 999.11


99 11; 9



<i>B</i> <i>B</i> <i>B</i>


6


9 6 2 4 7 2 9 21 9


15


<i>x</i> <i>y</i>


<i>B</i> <i>x</i> <i>y</i> <i>x</i> <i>y</i>


<i>x</i> <i>y</i>


 


          <sub>   </sub>






11 7 4 6 2 2 11 13 11


<i>B</i>       <i>x</i> <i>y</i>   <i>x</i> <i>y</i>


9( ) 2


<i>x</i> <i>y</i> <i>ktm vs y</i> <i>x</i>



2& 6 2; 4


2 & 15( )


<i>y</i> <i>x</i> <i>x</i> <i>y</i> <i>x</i> <i>y</i>


<i>y</i> <i>x</i> <i>x</i> <i>y</i> <i>ktm</i>


      


   


Vậy <i>B</i>6224427


<b>Câu 2. </b>


3


2 4


2 3 2 2


2 3 2 2 3


)5 125 5 3


)3 81 3 2 4 2


)5 2.5 5 .3



5 5 .3 5 .2 5


2 3 3 2 6 3


<i>x</i>


<i>x</i>


<i>x</i>


<i>x</i>


<i>a</i> <i>x</i>


<i>b</i> <i>x</i> <i>x</i>


<i>c</i>


<i>x</i> <i>x</i> <i>x</i>





   


     


 



   


      


<b>Câu 3. </b>


a) Gọi d là ước chung của 12<i>n</i>1;30<i>n</i>2ta có:


 



5 12<i>n</i> 1 2 30<i>n</i>2 1 <i>d</i> <i>d</i> 1nên 12<i>n</i>1;30<i>n</i>2nguyên tố cùng nhau
Do đó 12 1


30 2


<i>n</i>
<i>n</i>




 là phân số tối giản.


</div>
<span class='text_page_counter'>(80)</span><div class='page_container' data-page=80>

2 2 2


2 2 2


1 1 1 1 1 1 1 1 1 1 1


.... .... 1 ...



2 3 100 1.2 2.3 99.100 2 2 3 99 100


1 1 1 1 99


.... 1 1


2 3 100 100 100


             


       


<b>Câu 4. </b>


23 23.101 2323
99 99.101 9999
23 23.10101 232323
99 99.10101 999999
23 23.1010101 23232323
99 99.1010101 99999999


23 2323 232323 23232323
99 9999 999999 99999999


 


 


 



   


<b>Câu 5. </b>


Gọi số giấy mỗi lớp thu được là (<i>x kg</i>)thì

<i>x</i>24 11;

 

<i>x</i>23 10



<i>x</i> 13 10,11


 


Do đó

<i>x</i>13

<i>BC</i>

10;11

 

 0;110;220;330;...



Mà số kg giấy nằm trong khoảng 200 đến 300   <i>x</i> 13 220 <i>x</i> 233
Số học sinh lớp 6A:

233 24 :11 1 20

  (học sinh)


Số học sinh lớp 6 : 233 23 :10 1 22<i>B</i>

  học sinh


<b>PHÒNG GIÁO DỤC VÀ ĐÀO TẠO </b>
<b>LÂM THAO </b>


<b>ĐỀ THI HỌC SINH GIỎI CẤP HUYỆN </b>
<b>Mơn Tốn 6 Năm học 2018-2019 </b>
<b>I.</b> <b>PHẦN TRẮC NGHIỆM </b>


<b>Câu 1. </b>Có bao nhiêu số tự nhiên có hai chữ số mà chữ số hàng chục lớn hơn chữ
số hàng đơn vị


A. 30 <i>B</i>.40 C. 45 D. 55


<b>Câu 2. </b>Tổng của hai số tự nhiên là 102. Nếu thêm chữ số 0 vào bên phải số bé rồi
cộng với số lớn ta được tổng mới là 417. Khi đó số lớn là:



A. 43 B. 54 C. 60 D. 67


<b>Câu 3. </b>Kết quả của phép tính 1 2 3 4 5 6 ... 99 100        là:


A. 50 B. 50 C. 100 D. 0


</div>
<span class='text_page_counter'>(81)</span><div class='page_container' data-page=81>

A.

0;1; 2; 3 

B.

 

0;1 C.

 2; 3

D.

 1; 2



<b>Câu 5. </b>Cho 7 ô liên tiếp sau
13


 <i>a</i> 27


Biết rằng tổng của ba ô bất kỳ luôn bằng 0. Khi đó giá trị của <i>a</i>là :


A. 13 B. 27 C. 13 D. 27


<b>Câu 6. </b>Cho 4 6 9 7


7.31 7.41 10.41 10.57


<i>A</i>    và 7 5 3 11


19.31 19.43 23.43 23.57


<i>B</i>   


Tỷ số <i>A</i>



<i>B</i>là:


A. 7


4 B.


7


2 C.


5


2 D.


11
4


<b>Câu 7. </b>Trung bình cộng của tử số và mẫu số của một phân số là 68. Cộng thêm
vào tử số của phân số đó 4 đơn vị thì ta được phân số mới bằng phân số 3.


2 Phân số
lúc đầu là:


A. 84


52 B.


76


60 C.



75


61 D.


80
56


<b>Câu 8. </b>Trên đường thẳng <i>a</i>lấy ba điểm <i>M N P</i>, , sao cho <i>MN</i>2<i>cm NP</i>, 5<i>cm</i>.Khi
đó độ dài đoạn thẳng <i>MP</i>bằng:


A. 3<i>cm</i> B. 7<i>cm</i> C. 3<i>cm</i>hoặc 7<i>cm</i> D. 3,5<i>cm</i>
<b>Câu 9. </b>Cho 100 điểm trong đó khơng có ba điểm nào thẳng hàng. Cứ qua hai điểm


vẽ một đường thẳng. Số đường thẳng vẽ được là:


A. 200 B. 4950 C. 5680 D. 9900


<b>Câu 10. </b>Cho <i>xOy</i>80 ,0 tia Oz nằm giữa hai tia Ox, Oy sao cho <i>xOz</i>300. Số đo
<i>yOz</i>là


A. 50 0 B. 110 0 C. 50 hoặc 0 110 0 D. 80 . 0


<b>Câu 11. </b>Cho <i>xOy</i>80 ,0 Oz là tia phân giác của góc xOy, Ot là tia phân giác của
<i>xOz</i>. Số đo của <i>yOt</i>là:


A. 20 0 B. 40 0 C. 50 0 D. 60 0


<b>Câu 12. </b>Có 9 miếng bánh chưng cần ráng vàng cả hai mặt. Thời gian ráng mỗi mặt
cần 3 phút. Nếu dùng một chiếc chảo mỗi lần chỉ ráng được nhiều nhất 6 miếng thì


cần thời gian ít nhất là bao lâu để ráng xong 9 miếng bánh chưng đó


A. 9 phút B. 12 phút C. 18 phút D. 27 phút


</div>
<span class='text_page_counter'>(82)</span><div class='page_container' data-page=82>

<b>Câu 1. (4 điểm) </b>


a) Cho biết <i>a</i>4<i>b</i>chia hết cho 13

<i>a b</i>, 

.Chứng minh rằng 10<i>b</i>13
b) Tìm số nguyên tố <i>ab a b</i>

 0

sao cho <i>ab ba</i> là số chính phương


<b>Câu 2. (4 điểm) </b>


a) Cho <i>M</i>        

<i>a b</i>

 

<i>b c a</i>

 

<i>c a</i>

.Trong đó <i>b c</i>,  cịn <i>a</i>là một số
nguyên âm. Chứng minh rằng biểu thức M luôn dương


b) Tìm tất cả các cặp số nguyên sao cho tổng của chúng bằng tích của chúng.


<b>Câu 3. (4 điểm) </b>Cho đoạn thẳng AB; điểm O thuộc tia đối của tia AB, Gọi M, N
thứ tự là trung điểm của <i>OA OB</i>, .


a) Chứng tỏ rằng <i>OA OB</i>


b) Trong ba điểm <i>O M N</i>, , điểm nào nằm giữa hai điểm còn lại.


c) Chứng tỏ rằng độ dài đoạn thẳng MN khơng phụ thuộc vào vị trí của điểm O
(O thuộc tia đối của tia AB)


<b>Câu 4. (2 điểm) </b>


Tính giá trị biểu thức sau:



1 2 3 4 2017 1 1 1 1 1


2017 ... : ...


4 5 6 7 2020 20 25 30 35 10100


<i>B</i><sub></sub>        <sub> </sub>      <sub></sub>


   


<b>ĐÁP ÁN </b>
<b>I.trắc nghiệm </b>


1C 2D 3B 4A 5B 6C 7D 8C 9B 10A 11D 12A


<b>II. tự luận </b>
<b>Câu 1. </b>




)4 13 10 40 13 10 39 13


39 13 10 13


) 43;73


<i>a b</i> <i>a</i> <i>b</i> <i>a b</i> <i>b</i>


<i>Do</i> <i>b</i> <i>a b</i>



<i>b ab</i>


    


 




<b>Câu 2. </b>


a) <i>M</i>  <i>a</i>mà a là số nguyên âm nên M luôn dương
b) <i>x</i>0,<i>y</i>0hoặc <i>x</i>2,<i>y</i>2


<b>Câu 3. </b>


a) Lập luận chứng tỏ được <i>OA OB</i>


b) Lập luận chứng tỏ <i>OM</i> <i>ON</i>nên M nằm giữa hai điểm O và N


c) .


2


<i>AB</i>


<i>MN</i>  Vì AB có độ dài khơng đổi nên <i>MN</i>có độ dài khơng đổi.


<b>Câu 4. </b><i>B</i>15


</div>
<span class='text_page_counter'>(83)</span><div class='page_container' data-page=83>

<b>Năm học : 2019-2020 </b>


<b>Mơn: Tốn 6 </b>
<b>Bài 1. (2,0 điểm) </b>


a) Rút gọn phân số:

 



3 <sub>3</sub> <sub>3</sub>


3 4


2 .3 .5 .7.8
3.5 .2 .42




b) So sánh không qua quy đồng: <sub>2005</sub>7 15<sub>2006</sub>; 15<sub>2005</sub> <sub>2006</sub>7


10 10 10 10


<i>A</i>    <i>B</i>   


<b>Bài 2. (2,0 điểm) </b>


Khơng quy đồng hãy tính hợp lý các tổng sau:


1 1 1 1 1 1


)


20 30 42 56 72 90



5 4 3 1 13


)


2.1 1.11 11.2 2.15 15.4


<i>a A</i>


<i>b B</i>


     


     


    


<b>Bài 3. (2,0 điểm) </b>


Một người bán năm giỏ xoài và cam. Mỗi giỏ chỉ đựng một loại quả với số
lượng là: 65 ,71 ,58 ,72 ,93 .<i>kg</i> <i>kg</i> <i>kg</i> <i>kg</i> <i>kg</i> Sau khi bán một giỏ cam thì số lượng xồi
còn lại gấp ba lần số lượng cam còn lại. Hãy cho biết giỏ nào đựng cam, giỏ nào
đựng xồi.


<b>Bài 4. (3,0 điểm) </b>


Cho góc AOB và góc BOC là hai góc kề bù. Biết góc BOC bằng 5 lần góc
AOB


a) Tính số đo mỗi góc



b) Gọi OD là tia phân giác của góc BOC. Tính số đo góc AOD


c) Trên cùng một nửa mặt phẳng bờ là đường thẳng AC chứa tia OB, OD, vẽ
thêm 2006 tia phân biệt (không trùng với các tia <i>OA OB OC OD</i>, , , đã cho) thì
có tất cả bao nhiêu góc ?


<b>Bài 5. (1,0 điểm) </b>


</div>
<span class='text_page_counter'>(84)</span><div class='page_container' data-page=84>

<b>ĐÁP ÁN</b>
<b>Bài 1. </b>


a) Hs tự rút gọn


2005 2006 2005 2006 2006


2005 2006 2005 2005 2006


2006 2005


7 15 7 8 7


)


10 10 10 10 10


15 7 7 8 7


10 10 10 10 10


8 8



10 10


<i>b A</i>


<i>B</i>


<i>A</i> <i>B</i>


    


    


    


    


 <sub></sub>  <sub> </sub>


<b>Bài 2. </b>


1 1 1 1 1 1 1 1


) .... ....


20 30 42 90 4.5 5.6 6.7 9.10


1 1 1 1 1 1 3


...



4 5 9 10 4 10 20


5 4 3 1 13 5 4 3 1 13


) 7.


2.1 1.11 11.2 2.15 15.4 2.7 7.11 11.14 14.15 15.28


1 1 1 1 1


7.


2 7 7 11 11


<i>a A</i>


<i>b B</i>


     


      <sub></sub>     <sub></sub>


 




   


 <sub></sub>     <sub></sub> <sub></sub>  <sub></sub>



   


 


      <sub></sub>     <sub></sub>


 


      1 1 1 1 1 7. 1 1 31


14 14 15 15 28 2 28 4


 <sub></sub> <sub></sub> <sub></sub> <sub></sub> <sub></sub>  <sub></sub> <sub></sub>


   


   


<b>Bài 3. </b>


Tổng số xoài và cam lúc đầu: 65 71 58 72 93 359(     <i>kg</i>)


Vì số xồi cịn lại gấp ba lần số cam cịn lại nên tổng số xồi và cam cịn lại chia
hết cho 4, mà 359 chia cho 4 dư 3 nên giỏ cam bán đi có khối lượng chia cho 4 dư
3.


Trong các số 65;71;58;72;93chỉ có 71 chia cho 4 dư 3
Vậy gỉ cam bán đi là giỏ 71kg



Số xồi và cam cịn lại: 359 71 288(  <i>kg</i>)
Số cam còn lại: 288: 472(<i>kg</i>)


</div>
<span class='text_page_counter'>(85)</span><div class='page_container' data-page=85>

<b>Bài 4. </b>


a) Vì góc AOB và BOC là hai góc kề bù nên: <i>AOB</i><i>BOC</i>1800mà
5


<i>BOC</i> <i>AOB</i>nên 6.<i>AOB</i>1800<i>AOB</i>30 ,0 <i>BOC</i>1500
b) Vì <i>OD</i>là tia phân giác của <i>BOC</i>nên 1 750


2


<i>BOD</i><i>DOC</i> <i>BOC</i>


Vì góc AOD và góc DOC là hai góc kề bù nên: <i>AOD DOC</i> 1800


Do đó: 0 0 0 0


180 180 75 105


<i>AOD</i> <i>DOC</i>  


c) Tất cả có 2010 tia phân biệt. Cứ 1 tia trong 2010 tia đó tạo với 2009 tia cịn
lại thành 2009 góc. Có 2010 tia tạo thành 2010.2009 góc, nhưng như thế mỗi
góc tính 2 lần. Vậy có tất cả 2010.2009 2019045


2  góc


<b>Bài 5. </b>



P có dạng 3<i>k</i>1,3<i>k</i>2

<i>k</i>



Dạng <i>p</i>3<i>k</i>  2 <i>p</i> 4là hợp số trái với đề bài


3 1 8 3 9 3 8


<i>p</i> <i>k</i> <i>p</i> <i>k</i> <i>p</i>


         là hợp số


<b>ĐỀ THI HỌC SINH GIỎI CẤP TRƯỜNG </b>
<b>Năm học 2018-2019 </b>


<b>Môn thi: TỐN 6 </b>


<i><b>A</b></i>

<i><b>C</b></i>



<i><b>B</b></i>

<i><b>D</b></i>



</div>
<span class='text_page_counter'>(86)</span><div class='page_container' data-page=86>

<b>Câu 1.</b>Tính giá trị các biểu thức sau:


       

2 3 4

   

2010 2011


) 1 . 1 . 1 . 1 ... 1 . 1


<i>a A</i>      


131313 131313 131313



) 70.


565656 727272 909090


<i>b B</i> <sub></sub>   <sub></sub>


 


2 3 4 5


)


3 4 5 2


<i>a</i> <i>b</i> <i>c</i> <i>d</i>


<i>c C</i>


<i>b</i> <i>c</i> <i>d</i> <i>a</i>


    biết2 3 4 5


3 4 5 2


<i>a</i> <i>b</i> <i>c</i> <i>d</i>


<i>b</i>  <i>c</i>  <i>d</i>  <i>a</i>


<b>Câu 2. </b>Tìm <i>x</i>là các số tự nhiên, biết:



1 8


)


2 1


2 2


0,4


1 3 <sub>9</sub> <sub>11</sub>


) : 9


8 8


2 2 <sub>1,6</sub>


9 11


<i>x</i>
<i>a</i>


<i>x</i>


<i>b x</i>


 



 


 <sub></sub> <sub></sub>


 


  <sub> </sub>


<b>Câu 3. </b>


a) Tìm tất cả các cặp số tự nhiên

 

<i>x y</i>, sao cho 34 5<i>x y</i>chia hết cho 36
b) Không quy đồng mẫu số hãy so sánh:


2010 2011 2011 2010


9 19 9 19


;


10 10 10 10


<i>A</i>    <i>B</i>   


<b>Câu 4. </b>Cho 1
4


<i>n</i>
<i>A</i>


<i>n</i>








a) Tìm <i>n</i>nguyên để <i>A</i>là một phân số
b) Tìm <i>n</i>nguyên để <i>A</i>là một số nguyên.


<b>Câu 5. </b>


Cho tam giác <i>ABC</i>có <i>ABC</i>55 ,0 trên cạnh AC lấy điểm <i>D</i>(<i>D</i>khơng trùng
với A và C)


a) Tính độ dài <i>AC</i>,biết<i>AD</i>4<i>cm CD</i>, 3<i>cm</i>
b) Tính số đo <i>DBC</i>biết <i>ABD</i>300


c) Từ B dựng tia <i>Bx</i>sao cho <i>DBx</i>90 .0 Tính số đo <i>ABx</i>


d) Trên cạnh <i>AB</i>lấy điểm <i>E</i>(E không trùng với <i>A</i>và B). Chứng minh rằng 2
đoạn thẳng BD và <i>CE</i>cắt nhau.


</div>
<span class='text_page_counter'>(87)</span><div class='page_container' data-page=87>

     



) 1.1. 1 ... 1 .1. 1 1


13 13 13 1 1 1


) 70. 70.13.



56 72 90 7.8 8.9 9.10


1 1
70.13. 39
7 10
<i>a</i>
<i>b B</i>
     
   
 <sub></sub>   <sub></sub> <sub></sub>   <sub></sub>
   
 
 <sub></sub>  <sub></sub>
 


c) Đặt 2 3 4 5


3 4 5 2


<i>a</i> <i>b</i> <i>c</i> <i>d</i>


<i>k</i>
<i>b</i>  <i>c</i>  <i>d</i>  <i>a</i> 


Ta có: 2 .3 .4 .5 4 4 1 1 4


3 4 5 2


<i>a</i> <i>b</i> <i>c</i> <i>d</i>



<i>k</i> <i>k</i> <i>k</i> <i>C</i>


<i>b</i> <i>c</i> <i>d</i> <i>a</i>         


<b>Câu 2. </b>


2

 

2


1 8


) 1 16 4


2 1


) 1 4 3


) 1 4 5( )


<i>x</i>


<i>a</i> <i>x</i>


<i>x</i>


<i>x</i> <i>x</i>


<i>x</i> <i>x</i> <i>ktm</i>


      




    
      


Vậy <i>x</i>3


2 2 2 2


0,4 0,4


1 3 <sub>9</sub> <sub>11</sub> 19 3 <sub>9</sub> <sub>11</sub>


) : 9 :


8 8 2 2


2 2 <sub>1,6</sub> 2 2


4. 0,4


9 11 9 11


1


2


8 4


<i>b x</i> <i>x</i>



<i>x</i>
<i>x</i>
   
 <sub></sub> <sub></sub> <sub></sub>  <sub></sub> <sub></sub>
    <sub></sub> <sub></sub>
  <sub> </sub>   <sub></sub> <sub> </sub> <sub></sub>
 
   
<b>Câu 3. </b>


a) Ta có: 36 9.4 mà ƯC( 4,9) 1


Vậy để 34 5<i>x y</i>chia hết cho 36 thì 34 5<i>x y</i>chia hết cho 4 và 9
34 5<i>x y</i>chia hết cho 9 khi 3 4   <i>x</i> 5 <i>y</i> 9  12 <i>x</i> <i>y</i> 9 1

 


34 5<i>x y</i>chia hết cho 4 khi 5 4<i>y</i>  <i>y</i> 2,<i>y</i>6


Với <i>y</i>2thay vào (1) 14 <i>x</i> 9 <i>x</i> 4
Với <i>y</i>6thay vào (1) 18 9 0


9
<i>x</i>
<i>x</i>
<i>x</i>


  <sub>  </sub>


</div>
<span class='text_page_counter'>(88)</span><div class='page_container' data-page=88>

2010 2011 2010 2011 2011



2011 2010 2011 2010 2010


9 19 9 10 9


10 10 10 10 10


9 19 9 10 9


10 10 10 10 10


<i>A</i>


<i>B</i>


    


    


    


    


Ta thấy 10<sub>2011</sub> 10<sub>2010</sub>


10 10 <i>A</i> <i>B</i>


 <sub></sub>  <sub> </sub>


<b>Câu 4. </b>



a) 1


4


<i>n</i>
<i>A</i>


<i>n</i>





 là phân số khi <i>n</i>    4 0 <i>n</i> 4


b) 1 4 5 1 5


4 4 4


<i>n</i> <i>n</i>


<i>A</i>


<i>n</i> <i>n</i> <i>n</i>


  


   


  



Với <i>n</i>nguyên, A nhận giá trị nguyên 5 <i>n</i>   4

<i>n</i> 4

<i>U</i>

  

5   1; 5


Lập luận tìm ra được <i>n</i>   9; 5; 3;1


<b>Câu 5. </b>


a) D nằm giữa A và C<i>AC</i> <i>AD CD</i>   4 3 7<i>cm</i>


b) Tia BD nằm giữa hai tia BA và BC nên <i>ABC</i><i>ABD DBC</i>


0 0 0


55 30 25


<i>DBC</i> <i>ABC</i> <i>ABD</i>


     


c) Xét hai trường hợp:


- Trường hợp 1: Tia <i>Bx</i>và BD nằm về hai phía nửa mặt phẳng có bờ là AB


Tính được: 0


90


<i>ABx</i> <i>ABD</i>


<i><b>A</b></i>



<i><b>B</b></i>

<i><b>C</b></i>




</div>
<span class='text_page_counter'>(89)</span><div class='page_container' data-page=89>

Mặt khác tia <i>BD</i> nằm giữa hai tia <i>BA BC</i>, nên 00 <i>ABD</i>550


0 0 0 0 0 0


90 55 <i>ABx</i> 90 0 35 <i>ABx</i> 90


       


- Trường hợp 2: Tia <i>Bx BD</i>, nằm về cùng nửa mặt phẳng có bờ là <i>AB</i>


Tính được: 0


90


<i>ABx</i>  <i>ABD</i>


Lập luận tương tự trường hợp 1 chỉ ra được: 900 <i>ABx</i>1450
Vậy 350<i>ABx</i>145 ,0 <i>ABx</i>900


d) Xét đường thẳng BD


Do <i>BD</i> cắt AC nên đường thẳng <i>BD</i>chia mặt phẳng làm hai nửa: 1 nửa mặt phẳng
có bờ <i>BD</i>chứa điểm C và nửa mặt phẳng bờ BD chứa điểm A


tia <i>BA</i>thuộc nửa mặt phẳng chứa điểm A


E thuộc đoạn AB<i>E</i>thuộc nửa mặt phẳng bờ BD chứa điểm A
,



<i>E C</i>


 ở hai nửa mặt phẳng bờ BD


đường thẳng BD cắt đoạn EC
Xét đường thẳng CE


Lập luận tương tự: ta có đường thẳng EC cắt đoạn BD
Vậy 2 đoạn thẳng <i>EC BD</i>, cắt nhau


<b>PHÒNG GIÁO DỤC VÀ ĐÀO TẠO </b>
<b>BẠCH THƠNG </b>


<b>ĐỀ CHÍNH THỨC </b>


<b>KỲ THI CHỌN HỌC SINH GIỎI THCS CẤP HUYỆN </b>
<b>NĂM HỌC 2018-2019 </b>


<b>MƠN THI: TỐN – LỚP 6 </b>
<b>Câu 1. (4,0 điểm) </b>


c) Thực hiện phép tính




10 10
8


540 : 23,7 19,7 42. 132 75 36 7317
2 .13 2 .65



2 .104


<i>A</i>


<i>B</i>


<sub></sub>  <sub></sub>   





d) Chứng minh rằng tổng của 5 số tự nhiên chẵn liên tiếp thì chia hết cho 10,
còn tổng của 5 số tự nhiên lẻ liên tiếp chia cho 10 dư 5


<b>Câu 2. (4,0 điểm) </b>


c) Tổng của hai số nguyên tố có thể bằng 2015hay khơng ? Vì sao ?
d) Tìm tất cả các số nguyên tố <i>p</i> sao cho <i>p</i>11cũng là số nguyên tố.


<b>Câu 3. (4,0 điểm) </b>


</div>
<span class='text_page_counter'>(90)</span><div class='page_container' data-page=90>

<b>Câu 4. (4,0 điểm) </b>


c) Tìm tích 1 1 1 1 1 1 ... 1 1


2 3 4 100


 <sub></sub>  <sub></sub>  <sub></sub>   <sub></sub> 



     


     


d) So sánh <i>A</i>và <i>B</i>biết: 2013.2014 1
2013.2014


<i>A</i>  và 2014.2015 1


2014.2015


<i>B</i> 


<b>Câu 5. (4,0 điểm) </b>


Cho đoạn thẳng <i>AB</i>;điểm O thuộc tia đối của tia <i>AB</i>.Gọi <i>M N</i>, thứ tự là
trung điểm của <i>OA OB</i>,


d) Chứng tỏ <i>OA OB</i>


e) Trong ba điểm <i>O M N</i>, , điểm nào nằm giữa hai điểm còn lại


</div>
<span class='text_page_counter'>(91)</span><div class='page_container' data-page=91>

<b>ĐÁP ÁN </b>
<b>Câu 1. </b>






10



10 10 10


8 8 8 3


) 540 : 4 42.171 7317
135 7182 7317 0


2 .13. 1 5


2 .13 2 .65 2 .13.6


3
2 .104 2 .8.13 2 .2 .13


<i>a A</i>
<i>A</i>
<i>B</i>


  


   





   


b) Gọi 5 số chẵn liên tiếp là:2 ;2<i>n n</i>2;2<i>n</i>4;2<i>n</i>6;2<i>n</i>8
Tính tổng ta được: 10<i>n</i>20 10



Gọi 5 số lẻ liên tiếp là: 2<i>n</i>1;2<i>n</i>3;2<i>n</i>5;2<i>n</i>7;2<i>n</i>9
Tính tổng được: 10<i>n</i>25 10

<i>n</i> 2

5chia cho 10 dư 5


<b>Câu 2. </b>


c) Tổng của hai số nguyên tố bằng 2015 là số lẻ, nên một trong hai số nguyên
tố phải là 2


Khi đó số kia là 2013, số này là hợp số


Vậy khơng tồn tại hai số ngun tố có tổng bằng 2015


d) Nếu <i>p</i>lẻ <i>p</i> 11là số chẵn lớn hơn 11 nên không là số nguyên tố
Suy ra <i>p</i>chẵn  <i>p</i> 2


<b>Câu 3. </b>


c) Ta có:


 

 



 





1 3 5 .... 99 0


1 99 .50



0
2


50 .50 0


50 0 50


<i>x</i> <i>x</i> <i>x</i> <i>x</i>


<i>x</i> <i>x</i>


<i>x</i>


<i>x</i> <i>x</i>


        


  


 


  <sub></sub>


 


    


d) Ta có: 3<i>n</i> 8 3<i>n</i>  3 5 3

<i>n</i> 1

5


Suy ra :

3<i>n</i>8

 

<i>n</i>1

khi

<i>n</i> 1

<i>U</i>(5)  

1; 5



Tìm được: <i>n</i>  

6; 2;0;4



<b>Câu 4. </b>


c) Ta có:




1 1 1 1


1 1 1 ... 1


2 3 4 100


1.2.3.4....99


1 2 3 99 1


. . ...


2 3 4 100 2.3.4....100 100


 <sub></sub>  <sub></sub>  <sub></sub>   <sub></sub> 


     


     





    


</div>
<span class='text_page_counter'>(92)</span><div class='page_container' data-page=92>

d) Ta có:


2013.2014 1 1


1


2013.2014 2013.2014


2014.2015 1 1


1


2014.2015 2014.2015


<i>A</i>


<i>B</i>




  




  


Vì 1 1



2013.2014 2014.2015nên <i>A B</i>


<b>Câu 5. </b>


d) Hai tia <i>OA OB</i>, đối nhau nên điểm <i>A</i>nằm giữa hai điểm <i>O</i>và B,
suy ra <i>OA OB</i>


e) Ta có <i>M</i>và N thứ tự là trung điểm của <i>OA OB</i>, nên ;


2 2


<i>OA</i> <i>OB</i>


<i>OM</i>  <i>ON</i> 


Vì <i>OA OB</i> <i>OM</i> <i>ON</i>


Hai điểm <i>M</i>và N thuộc tia OB mà <i>OM</i> <i>ON</i>nên điểm M nằm giữa hai điểm <i>O</i>và
N


f) Ta có: <i>OM</i> <i>MN</i> <i>ON</i><i>MN</i> <i>ON</i><i>OM</i>


Hay


2 2


<i>OB OA</i> <i>AB</i>


<i>MN</i>   



Vì <i>AB</i>có độ dài khơng đổi nên <i>MN</i>có độ dài khơng đổi.


<b>PHỊNG GIÁO DỤC VÀ ĐÀO TẠO </b>
<b>GIAO THỦY </b>


<b>ĐỀ THI HỌC SINH GIỎI NĂM HỌC 2018-2019 </b>
<b>MÔN TỐN LỚP 6 </b>


<b>Bài 1. </b>


1) Tính tổng <i>A</i>1.2 2.3 3.4 ... 98.99   
2) Cho biểu thức : 1<sub>2</sub> 1<sub>2</sub> 1<sub>2</sub> ... 1<sub>2</sub>


5 6 7 100


<i>B</i>    


Chứng tỏ rằng 1 1
6 <i>B</i> 4


<i><b>M</b></i>

<i><b><sub>N</sub></b></i>



</div>
<span class='text_page_counter'>(93)</span><div class='page_container' data-page=93>

<b>Bài 2. </b>


Tìm số nguyên <i>x</i>biết: 1 2 18


18 0


2 .2 .2<i>x</i> <i>x</i> <i>x</i> 1000....0 : 5
<i>chu so</i>



  <sub></sub>


<b>Bài 3. </b>


1) Cho <i>abc</i>deg 7.Chứng minh <i>abc</i>deg 7
2) Tìm số nguyên <i>n</i>sao cho <i>n</i>2 1 <i>n</i>1


<b>Bài 4. </b>


Cho n đường thẳng trong đó bất cứ hai đường thẳng nào cũng cắt nhau, khơng có


ba đường thẳng nào đồng quy. Biết rằng số giao điểm của các đường thẳng đó là


780. Tính n


</div>
<span class='text_page_counter'>(94)</span><div class='page_container' data-page=94>

<b>ĐÁP ÁN </b>
<b>Bài 1. </b>




2 2 2


1)3 1.2.3 2.3.3 3.4.3 ... 98.99.3


1.2.3 2.3. 4 1 3.4 . 5 2 ... 98.99. 100 97


1.2.3 2.3.4 1.2.3 3.4.5 2.3.4 .... 98.99.100 97.98.99
98.99.100 98.33.100 323400



1 1 1 1 1 1


2) .... ....


5 6 100 4.5 5.6


<i>A</i>


<i>A</i>
<i>B</i>


    


        


       


   


       


2 2 2


1 1 1 1 1 1


....


99.100 4 5 5 6 99 100


1 1 1



(1)


4 100 4


1 1 1 1 1 1 1 1 1 1 1 1


.... .... ....


5 6 100 5.6 6.7 100.101 5 6 6 7 100 101


1 1 96 96 1 1


(2)


5 101 505 576 6 6


<i>B</i>


<i>B</i>


<i>B</i> <i>B</i>


      


  


              


      



Từ (1) và (2) 1 1<sub>2</sub> 1<sub>2</sub> 1<sub>2</sub> ... 1 <sub>2</sub> 1


6 5 6 7 100 4


      


<b>Bài 2. </b>


1 2 18


18 0
3 3 18 18 18


2 .2 .2 1000....0 : 5


2 10 : 5 2 3 3 18 5


<i>x</i> <i>x</i> <i>x</i>


<i>chu so</i>
<i>x</i>


<i>x</i> <i>x</i>


 







      


<b>Bài 3. </b>


1) Ta có: <i>abc</i>deg 1000. <i>abc</i>deg


1001 1

<i>abc</i> deg 1001<i>abc abc</i> deg 1001<i>abc</i>

<i>abc</i> deg



        


Vì 1001<i>abc</i>7.143<i>abc</i>7.143.<i>abc</i> 7 (1)
deg 7


<i>abc</i> (gt) (2)


Từ (1) và (2) suy ra <i>abc</i>deg 7
2) Ta có:




2


2 1 1 3


<i>n</i>  <i>n n</i>   <sub></sub> <i>n</i> <sub></sub>
Vì <i>n n</i>

1

<i>n</i>1và  

<i>n</i> 1

<i>n</i>1
Để 2


2 1



<i>n</i>  <i>n</i> thì 3 <i>n</i>   1 <i>n</i> 1 <i>U</i>(3)     

1; 3

<i>n</i>

2;0; 4;2



</div>
<span class='text_page_counter'>(95)</span><div class='page_container' data-page=95>

Mỗi đường thẳng cắt <i>n</i>1đường thẳng cịn lại tạo nên <i>n</i>1giao điểm
Có <i>n</i>đường thẳng nên có <i>n n</i>

1

giao điểm


Nhưng mỗi giao điểm đã được tính 2 lần nên số giao điểm là

1


2


<i>n n</i>


Vậy với <i>n</i>đường thẳng, trong đó bất cứ hai đường thẳng nào cũng cắt nhau, khơng


có ba đường thẳng nào đồng quy có

1


2


<i>n n</i>


giao điểm (1)


Theo bài ra với <i>n</i>đường thẳng trong đó bất cứ hai đường thẳng nào cũng cắt nhau,


khơng có ba đường thẳng nào đồng quy. Biết rằng số giao điểm của các đường
thẳng đó là 780 (2)


Từ (1) và (2)

1

780

1

1560 39.40 40
2


<i>n n</i>



<i>n n</i> <i>n</i>




       


<b>Bài 5. </b>


Vì 7 5 1 3<i>a b</i>     7 <i>a</i> 5 <i>b</i> 1 3  <i>a b</i> 13 3  <i>a b</i> 1 3
Mà 0   <i>a b</i> 18

<i>a b</i> 

 

2;5;8;11;14;17

(1)


Vì <i>a b</i> 4chẵn nên a và b cùng lẻ hoặc cùng chẵn  <i>a b</i>chẵn (2)
Từ (1) và (2) suy ra <i>a b</i> 

2;8;14



2; 4 3, 1( )


8; 4 6; 2( )


14; 4 9; 5( )


<i>a b</i> <i>a b</i> <i>a</i> <i>b</i> <i>ktm</i>


<i>a b</i> <i>a b</i> <i>a</i> <i>b</i> <i>tm</i>


<i>a b</i> <i>a b</i> <i>a</i> <i>b</i> <i>tm</i>


       


      



      


Vậy <i>a</i>6,<i>b</i>2hoặc <i>a</i>9,<i>b</i>5


<b>PHÒNG GD & ĐT YÊN LẠC </b>


<b>Trường THCS Trung Nguyên </b> <b>ĐỀ KHẢO SÁT HỌC SINH GIỎI LỚP 6 <sub>Năm học 2018-2019 </sub></b>
<b>Mơn Tốn 6 </b>


<b>Bài 1. (2 điểm) </b>


a) Cho <i>ababab</i>là số có 6 chữ số. Chứng tỏ <i>ababab</i>là bội của 3


b) Cho <i>S</i>     5 52 53 54 ... 5 2004.Chứng minh S chia hết cho 126 và chia
hết cho 65.


</div>
<span class='text_page_counter'>(96)</span><div class='page_container' data-page=96>

Tìm số tự nhiên <i>x</i>biết:


 



) 1 2 ... 2010 2029099


)2 4 6 8 .... 2 210


<i>a x</i> <i>x</i> <i>x</i> <i>x</i>


<i>b</i> <i>x</i>


       



     


<b>Bài 3. (2,5 điểm) </b>


a) Tìm chữ số tận cùng của các số sau: 49 ,3231 2000
b) Chứng tỏ rằng: 1020118chia hết cho 72


c) So sánh các số sau: 3 và 39 1121;199 và 20 2003 15


<b>Bài 4. (1,5 điểm) </b>


Khối 6 của một trường chưa tới 400 học sinh, khi xếp hàng 10;12;15 đều dư
3 nhưng nếu xếp hàng 11 thì khơng dư. Tính số học sinh khối 6


<b>Bài 5. (1,5 điểm) </b>


Cho đoạn thẳng AB. Lấy điểm O nằm giữa A và B, lấy điểm I nằm giữa O
và B


a) Giả sử <i>AB</i>5<i>cm AO</i>, 2<i>cm BI</i>, 2<i>cm</i>.Tính OI


b) Giả sử <i>OA a BI</i> , <i>b</i>.Tìm điều kiện của a và b để <i>AI</i> <i>OB</i>


<b>Bài 6. (1 điểm) </b>


a) Vẽ 5 đoạn thẳng đôi một cắt nhau sao cho tổng số giao điểm là 10. Giải


thích vì sao số giao điểm khơng thể vượt quá 10 ?


b) Cho trước n điểm

<i>n</i> ,<i>n</i>2

. Vẽ các đoạn thẳng đi qua các cặp điểm

được tất cả 210 đoạn thẳng. Tìm n.


<b>ĐÁP ÁN </b>
<b>Bài 1. </b>


) .10101 3


<i>a ababab</i><i>ab</i> <i>ababab</i>là bội của 3
b) Chứng minh S chia hết cho 126


Có:


 

 



2 3 4 5 6 3 2 3 3 3 2 3


</div>
<span class='text_page_counter'>(97)</span><div class='page_container' data-page=97>

 





2 3 4 5 6 6 2 3 4 5 6


1998 2 3 4 5 6


5 5 5 5 5 5 5 . 5 5 5 5 5 5 ....


5 . 5 5 5 5 5 5


<i>S</i>             



     


Tổng trên có 2004:6=334 số hạng chia hết cho 126 nên <i>S</i> 126
*Chứng minh S chia hết cho 130


Có: 5 5    2 53 54

5 53

 

5 5 5 3

130 5.130    5 52 53 5 1304




2 3 4 4 2 3 4 2000 2 3 4


5 5 5 5 5 5 5 5 5 .... 5 . 5 5 5 5


<i>S</i>             


Tổng trên có 2004 :4=501 số hạng chia hết cho 130 nên <i>S</i> 130.


<b>Bài 2. </b>




<sub></sub>

<sub></sub>



)2011 1 2 ... 2010 2029099
2010.2011


2011 2029099


2



2010.2011


2029099 : 2011 4


2
)2 1 2 3 ... 210


1


2. 210 . 1 210 14.15 14


2


<i>a</i> <i>x</i>


<i>x</i>


<i>x</i>


<i>b</i> <i>x</i>


<i>x x</i>


<i>x x</i> <i>x</i>


    


  


 



 <sub></sub>  <sub></sub> 


 


    




      


<b>Bài 3. </b>


a) Do 49 có chữ số tận cùng là 9, khi đó nâng lên lũy bậc lẻ có chữ số tận cùng
là 9


Vậy 49 có chữ số tận cùng là 9 31


Ta có 322000 324.500 có chữ số tận cùng là 0 nên khi nâng lên lũy thừa 4n có tận
cùng là chữ số 6. Vậy 322000có chữ số tận cùng là 6


b) Vì 1020118có tổng các chữ số chia hết cho 9 nên tổng chia hết cho 9
Lại có 1020118có 3 chữ số tận cùng là 008 nên chia hết cho 8


Vậy 1020118chia hết cho 72.


c) Ta có: 339 340 

 

34 10 81 ;1110 211120

 

112 10 12110
Vì 12110 8110nên 1121 339


Ta có: 19920 20020 

8.25

20 2 .560 40


15


15 15 4 3 60 45


2003 2000  2 .5 2 .5
Vì 2 .560 40 2 .560 45nên 19920 200315


<b>Bài 4. </b>


</div>
<span class='text_page_counter'>(98)</span><div class='page_container' data-page=98>

Theo đề ra ta có: <i>x</i> 3 <i>BC</i>

10,12,15

và <i>x</i>11,<i>x</i>400
(10,12,15) 60


<i>BCNN</i> 


3;63;123;183;243;303;363;423;543....



<i>x</i> , mà <i>x</i>11,<i>x</i>400 <i>x</i> 363


Vậy số học sinh khối 6 là 363 em


<b>Bài 5. </b>


a) Có hình vẽ đúng


Vì I nằm giữa A và B nên <i>AB</i> <i>AI</i> <i>IB</i><i>AI</i>  <i>AB</i><i>IB</i>  5 2 3<i>cm</i>


O nằm giữa A và I nên <i>AI</i> <i>OA OI</i> <i>OI</i>  <i>AI</i>  <i>AO</i>  3 2 1<i>cm</i>


b) Vì O nằm giữa A và I nên <i>AI</i> <i>OA OI</i>



I nằm giữa O và B nên <i>OB</i><i>OI</i> <i>IB</i>


Để <i>AI</i> <i>OB</i>thì <i>OA</i><i>BI</i> <i>a</i> <i>b</i>


<b>Bài 6. </b>


a) Mỗi đoạn thẳng có số giao điểm với bốn đoạn cịn lại nhiều nhất chỉ có thể
là 4. Vậy với 5 đoạn thẳng thì số giao điểm nhiều nhất là 5.420.Nhưng
mỗi giao điểm đã được tính hai lần do đó số giao điểm nhiều nhất chỉ có


4.5: 2 10, suy ra số giao điểm không thể vượt quá 10


b) Qua mỗi cặp điểm vẽ được 1 đoạn thẳng. Có n điểm cho trước vẽ được:


1 : 2



<i>n n</i> đoạn thẳng.


Số đoạn thẳng vẽ được là : 210 đoạn thẳng nên ta có:


1 : 2

210 ( 1) 210 21.20


<i>n n</i>  <i>n n</i>  


Vậy <i>n</i>21


<b>PHÒNG GD&ĐT TAM DƯƠNG </b>
<b>ĐỀ CHÍNH THỨC </b>



<b>ĐỀ GIAO LƯU HỌC SINH GIỎI CẤP HUYỆN </b>
<b>NĂM HỌC 2017 -2018 </b>


<b>MƠN: TỐN 6 </b>
<b>Câu 1. (5,0 điểm) </b>


a) Rút gọn biểu thức: 10.11 50.55 70.77
11.12 55.60 77.84


 


 


b) Tìm số tự nhiên <i>x</i>,biết: 1 2 18


18... .. ..0


5 .5 .5<i>x</i> <i>x</i> <i>x</i> 1000...0 : 2
<i>chu so</i>


  <sub></sub>


c) Tìm hiệu <i>a b</i> ,biết rằng:
1.2 2.3 3.4 ... 98.99


<i>a</i>     và <i>b</i> 12 22  32 ... 98 2


<b>Câu 2. (3,0 điểm) </b>


a) Cho <i>A</i>  5 52 ... 5 . 100 Tìm số tự nhiên <i>n</i>,biết rằng: 4.<i>A</i> 5 5<i>n</i>


b) Tìm tất cả các số tự nhiên <i>n</i>để phân số 18 3


21 7


<i>n</i>
<i>n</i>




</div>
<span class='text_page_counter'>(99)</span><div class='page_container' data-page=99>

<b>Câu 3. (5,0 điểm) </b>


a) Tìm số tự nhiên nhỏ nhất biết rằng số đó chia cho 11 dư 6, chia cho 4dư 1
và chia cho 19 dư 11.


b) Cho <i>p</i> là số nguyên tố lớn hơn 3. Hỏi <i>p</i>20162018là số nguyên tố hay hợp
số


c) Tìm số tự nhiên có hai chữ số, biết rằng số đó gấp đơi tích các chữ số của nó


<b>Câu 4. (6,0 điểm) </b>


Cho hai góc <i>AOx</i>380và <i>BOx</i>112 .0 Biết rằng <i>AOx</i>và <i>BOx</i>không kề nhau
a) Trong 3 tia <i>OA OB Ox</i>, , tia nào nằm giữa hai tia cịn lại ? Vì sao ?


b) Tính số đo góc <i>AOB</i>


c) Vẽ tia phân giác <i>OM</i> của góc <i>AOB</i>.Tính số đo góc <i>MOx</i>


d) Nếu <i>AOx</i>

;<i>BOx</i>

,trong đó 00  

 

1800và

 

 .Tìm điều kiện
liên hệ giữa và

để tia <i>OA</i>nằm giữa hai tia <i>OB</i>và <i>Ox</i>.Tính số đo <i>MOx</i>

theo và



<b>Câu 5. (1,0 điểm) </b>


Cho 100 số tự nhiên bất kỳ. Chứng minh rằng ta có thể chọn được ít nhất 15
số mà hiệu hai số tùy ý chia hết cho 7


<b>ĐÁP ÁN </b>
<b>Câu 1. </b>


a) Ta có:





10.11. 1 5.5 7.7


10.11 50.55 70.77 5


11.12 55.60 77.84 11.12. 1 5.5 7.7 6


 


  <sub></sub> <sub></sub>


   


b) Ta có: 1 2 18 1 2 18 18


18... .. ..0



5 .5 .5<i>x</i> <i>x</i> <i>x</i> 1 000....0 : 2 5<i>x x</i> <i>x</i> 10 : 2
<i>chu so</i>


  <sub></sub> <sub></sub>     <sub></sub>


18
18


3 3 18


18


10 10


5 5 3 3 18 5


2 2


<i>x</i>


<i>x</i> <i>x</i>


  


  <sub></sub> <sub></sub>      


 


</div>
<span class='text_page_counter'>(100)</span><div class='page_container' data-page=100>

 

 










2 2 2 2


2 2 2 2


1. 1 1 2 1 2 3 1 3 ... 98. 1 98
1 1 2 2 3 3 ... 98 98


1 2 3 ... 98 1 2 3 .... 98
1 2 3 .... 98


1 98 .98: 2 4851


<i>b</i>


<i>b</i> <i>b</i>


        


        


         


     



    


Vậy <i>a b</i> 4851


<b>Câu 2. </b>


a) Ta có: 5<i>A</i>   52 53 .... 5101


2 3 101

 

2 100

101
101


5 5 5 ... 5 5 5 ... 5 5 5


4 5 5


<i>A</i> <i>A</i>
<i>A</i>


           


  


Lại có: 4<i>A</i>   5 5<i>n</i> 5<i>n</i> 5101 <i>n</i> 101


b) Giả sử 18<i>n</i>3và 21<i>n</i>7cùng chia hết cho số nguyên tố <i>d</i>


Khi đó 18<i>n</i>3<i>d</i>và 21<i>n</i>7 <i>d</i>6 21

<i>n</i> 7

 

7 18<i>n</i>3

<i>d</i>21<i>d</i>
<i>d</i>


 Ư(21)

 

3;7


+Nếu <i>d</i> 3 khơng xảy ra vì 21<i>n</i>7khơng chia hết cho 3
+Nếu <i>d</i> 7khi đó, để phân số có thể rút gọn được thì:




18<i>n</i>3 7 <i>vi</i>...21<i>n</i>7 7 18<i>n</i> 3 21 7




18 <i>n</i> 1 7


  mà

18,7

  1 <i>n</i> 1 7 <i>n</i> 7<i>k</i>1

<i>k</i>



Vậy để phân số 18 3


21 7


<i>n</i>
<i>n</i>




 có thể rút gọn được thì <i>n</i>7<i>k</i>1

<i>k</i>



<b>Câu 3. </b>


a) Gọi số cần tìm là <i>a a</i>

 *

, ta có:

<i>a</i>6 11;

 

<i>a</i>1 4

<i>a</i>11 19


Ta có:









6 33 11 27 11


1 28 4 27 4


11 38 19 27 19


<i>a</i> <i>a</i>


<i>a</i> <i>a</i>


<i>a</i> <i>a</i>


   


   


   


Do <i>a</i>là số tự nhiên nhỏ nhất nên <i>a</i>27nhỏ nhất
Suy ra : <i>a</i>27<i>BCNN</i>

4;11;19

836


</div>
<span class='text_page_counter'>(101)</span><div class='page_container' data-page=101>

b) Vì <i>p</i>là số nguyên tố lớn hơn 3 nên <i>p</i>chia cho 3 dư 1 hoặc <i>p</i>chia cho 3 dư 2


2



<i>p</i>


 chia cho 3 dư 1


Mà <i>p</i>2016 

 

<i>p</i>2 1008nên <i>p</i>2016chia cho 3 dư 1.


Mặt khác: 2018chia cho 3 dư 2, do đó:

2016



2018 3


<i>p</i> 


<i>p</i>20162018 3

<i>p</i>20162018

3nên <i>p</i>20162018là hợp số
c) Gọi số tự nhiên phải tìm là <i>ab</i>với <i>a b</i>,  ,1 <i>a</i> 9,0 <i>b</i> 9
Theo đề bài, ta có: 10<i>a b</i> 2<i>ab</i>10<i>a</i>2<i>ab b</i> 10<i>a</i><i>b</i>

2<i>a</i>1



10 2<i>a a</i> 1


  mà

<i>a a</i>;2  1

1nên 10 2<i>a</i>1
Vì 2<i>a</i>1lẻ nên 2 1 1 1 10( )


2 1 5 3 6( )


<i>a</i> <i>a</i> <i>b</i> <i>ktm</i>


<i>a</i> <i>a</i> <i>b</i> <i>tm</i>


     



      


</div>
<span class='text_page_counter'>(102)</span><div class='page_container' data-page=102>

<b>Câu 4. </b>


a) Do <i>AOx</i>và <i>BOx</i>là hai góc khơng kể nhau mà có chung cạnh Ox nên hai tia


<i>OA</i>và <i>OB</i>cùng nằm trong một nửa mặt phẳng có bờ chứa tia <i>Ox</i>


Mà <i>AOx</i><i>BOx</i>(vì 380 112 )0 nên tia <i>OA</i>nằm giữa hai tia <i>OB</i>và <i>Ox</i>


b) Do <i>OA</i>nằm giữa hai tia <i>OB</i>và <i>Ox</i>nên ta có:


0 0 0


38 112 74


<i>AOx</i><i>AOB</i><i>BOx</i> <i>AOB</i> <i>AOB</i>


c) Do <i>OM</i>là phân giác của góc <i>AOB</i>nên: 1 1.740 370


2 2


<i>AOM</i>  <i>AOB</i> 


Do tia OA nằm giữa hai tia <i>OB</i>và <i>Ox</i>;tia Om nằm giữa hai tia <i>OA</i>và <i>OB</i>(OM là
tia phân giác của <i>AOB</i>)nên tia <i>OA</i>nằm giữa hai tia <i>OM</i> và <i>Ox</i>


0 0 0



37 38 75


<i>MOx</i> <i>AOM</i> <i>AOx</i>


     


d) Có <i>OA</i>và OB nằm trong cùng một nửa mặt phẳng bờ chứa tia <i>Ox</i>nên để tia
<i>OA</i>nằm giữa hai tia <i>OB</i> và <i>Ox</i>thì

 



Thật vậy, nếu

 

 thì <i>AOx</i><i>BOx</i>tia <i>OB</i>nằm giữa hai tia <i>OA</i>và <i>Ox</i>


Nếu

 

 thì <i>AOx</i><i>BOx</i>tia OB trùng với tia OA
Với

 

 ta có:




1 1


2 2


<i>AOx</i> <i>AOB</i> <i>BOx</i> <i>AOB</i>


<i>AOB</i> <i>AOM</i> <i>AOB</i>


 


   


    



      


<i><b>x</b></i>


<i><b>O</b></i>



<i><b>B</b></i>

<i><b>M</b></i>



</div>
<span class='text_page_counter'>(103)</span><div class='page_container' data-page=103>

Vậy 1

1



2 2


<i>MOx</i> <i>AOM</i> <i>AOx</i>

  

  

 



<b>Câu 5. </b>


Ta có 100 số khi đem chia cho 7 thì các số dư nhận nhiều nhất là 7 giá trị khác
nhau


Vì 100 7.14 2  nên theo nguyên lý Dirichle ta sẽ tìm được 15 số mà khi chia cho
7 có cùng số dư


Vậy hiệu của hai số tùy ý trong 15 số này thì chia hết cho 7


<b>TRƯỜNG THCS NGUYỄN TẤT THÀNH </b>


<b>ĐỀ THI HỌC SINH GIỎI CẤP TRƯỜNG </b>
<b>NĂM HỌC 2018-2019 </b>


<b>Mơn: Tốn 6 </b>


<b>Câu 1. (2,0 điểm) </b>


Cho <i>A</i> 2 22 23 24 ... 2 20. Tìm chữ số tận cùng của A


<b>Câu 3. (1,5 điểm) </b>


Chứng minh rằng: <i>n n</i>

1 2



<i>n</i>1 3



<i>n</i>1 4



<i>n</i>1

chia hết cho 5 với mọi số tự
nhiên n


<b>Câu 4. (1,0 điểm) </b>


Tìm tất cả các số nguyên tố <i>p</i>và q sao cho các số 7<i>p</i><i>q</i>và <i>pq</i>11cũng là
các số nguyên tố.


<b>Câu 5. (1,5 điểm) </b>


c) Tìm <i>UCLN</i>(7<i>n</i>3,8<i>n</i>1)

<i>n</i> * .

Tìm điều kiện của n để hai số đó nguyên
tố cùng nhau.


d) Tìm hai số tự nhiên biết: Hiệu của chúng bằng 84, UCLN của chúng bằng 28
và các số đó khoảng từ 300 đến 400


<b>Câu 6. (1,0 điểm) </b>


</div>
<span class='text_page_counter'>(104)</span><div class='page_container' data-page=104>

<b>Câu 7. (2,0 điểm) </b>


Cho <i>xAy</i>, trên tia <i>Ax</i>lấy điểm B sao cho <i>AB</i>5<i>cm</i>.Trên tia đối của tia <i>Ax</i>


lấy điểm D sao cho <i>AD</i>3<i>cm C</i>, là một điểm trên tia Ay
d) Tính BD



</div>
<span class='text_page_counter'>(105)</span><div class='page_container' data-page=105>

<b>ĐÁP ÁN </b>
<b>Câu 1. </b>


2 3 4 20

2 3 21


21 21


.2 2 2 2 2 ... 2 .2 2 2 .... 2


2 2 2 2 2


<i>A</i>


<i>A</i> <i>A</i> <i>A</i>


         


      


Ta có: 22124.5 1 

 

24 5.2 16 .2 5


5


...16 có tận cùng là 6 nên 16 .2 có tận cùng là 2 nên 5 <i>A</i>2212có tận cùng là 0


<b>Câu 3. </b>


Với mọi số tự nhiên <i>n</i>ta có các trường hợp sau:



Th1: <i>n</i> 5thì tích chia hết cho 5
Th2:<i>n</i>chia cho 5 dư 1 thì <i>n</i>5<i>k</i>1


4<i>n</i> 1 20<i>k</i> 5


    chia hết cho 5 tích chia hết cho 5
Th3: n chia cho 5 dư 2 thì <i>n</i>5<i>k</i>2


2<i>n</i> 1 10<i>k</i> 5


    chia hết cho 5tích chia hết cho 5
Th4: n chia cho 5 dư 3 thì <i>n</i>5<i>k</i>3


3<i>n</i> 1 15<i>k</i> 10


    chia hết cho 5tích chia hết cho 5
Th5: n chia cho 5 dư 4 thì <i>n</i>5<i>k</i>4


1 5 5


<i>n</i> <i>k</i>


    chia hết cho 5tích chia hết cho 5


Vậy <i>n n</i>

1 2



<i>n</i>1 3



<i>n</i>1 4



<i>n</i>1

chia hết cho 5 với mọi số tự nhiên n


<b>Câu 4. </b>Nếu <i>pq</i>11là số ngun tố thì nó phải là số lẻ (vì <i>pq</i> 11 2)


<i>pq</i>



 là số chẵn ít nhất 1 trong 2 số phải chẵn, tức là bằng 2
+giả sử <i>p</i>2.Khi đó 7<i>p</i> <i>q</i> 14<i>q pq</i>;  11 2<i>q</i>11


</div>
<span class='text_page_counter'>(106)</span><div class='page_container' data-page=106>

Vậy 2, 3
3, 2


<i>p</i> <i>q</i>


<i>p</i> <i>q</i>


 




  


<b>Câu 5. </b>


c) Gọi <i>UCLN</i>(7<i>n</i>3,8<i>n</i> 1) <i>d</i>với <i>n</i> *
Ta có: 7<i>n</i>3 ,8<i>d n</i>1 <i>d</i>


 



8. 7<i>n</i> 3 7. 8<i>n</i> 1 <i>d</i> 31<i>d</i> <i>d</i> 1;31


      


Để hai số đó nguyên tố cùng nhau thì <i>d</i> 31
Mà 7<i>n</i>3 317<i>n</i> 3 31 317

<i>n</i>4 31




4 31


<i>n</i>


  (vì 7 và 31 nguyên tố cùng nhau) <i>n</i> 31<i>k</i>4

<i>k</i>



Do đó <i>d</i> 31 <i>n</i> 31<i>k</i>4


Vậy hai số 7<i>n</i>3,8<i>n</i>1nguyên tố cùng nhau khi <i>n</i>31<i>k</i>4

<i>k</i>



d) Gọi hai số phải tìm là <i>a b a b</i>,

,  *,<i>a</i><i>b</i>



Ta có: ( , ) 28 28

, *,

 

, 1



28


<i>a</i> <i>k</i>


<i>UCLN a b</i> <i>k q</i> <i>k q</i>


<i>b</i> <i>q</i>





 <sub> </sub>  





Ta có: <i>a b</i> 84  <i>k</i> <i>q</i> 3


Theo bài ra : 300  <i>b</i> <i>a</i> 440   10 <i>q</i> <i>k</i> 16


Chỉ có 2 số 11, 14 nguyên tố cùng nhau và có hiệu là 3 <i>q</i> 11,<i>k</i>14
28.11 308


28.14 392


<i>a</i>
<i>b</i>


 




  <sub></sub> <sub></sub>


 . Vậy hai số phải tìm là 308,392.


<b>Câu 6. </b>






6 1 2 4( , )


<i>xy</i>    <i>x</i> <i>y</i> <i>x</i> <i>y</i>   <i>x y</i>


1 1 1 2 2 4 4



2 4 4 2 2 1 1


0 2 1 3 3 5


6 2 4 0 3 1


<i>x</i>
<i>y</i>
<i>x</i>
<i>y</i>


   


   


 




</div>
<span class='text_page_counter'>(107)</span><div class='page_container' data-page=107>

g) Vì <i>B</i><i>Ax D</i>, tia đối tia Ax<i>A</i>nằm giữa D và B
5 3 8


<i>BD</i> <i>BA</i> <i>AD</i> <i>cm</i>


     


h) Vì A nằm giữa D và B nên tia CA nằm giữa hai tia <i>CB CD</i>,


0 0 0



85 50 35


<i>ACD</i> <i>ACB</i> <i>BCD</i> <i>ACD</i> <i>BCD</i> <i>ACB</i>


        


i) *Trường hợp 1: K thuộc tia Ax


Chứng minh được K nằm giữa A và B


5 1 4( )


<i>AK</i> <i>KB</i> <i>AB</i> <i>KB</i> <i>AB</i> <i>AK</i> <i>cm</i>


        


*Trường hợp 2: K thuộc tia đối của tia <i>Ax</i>


-Lập luận chỉ ra được A nằm giữa K và B
Suy ra : <i>KB</i><i>KA</i><i>AB</i><i>KB</i>  5 1 6<i>cm</i>


Vậy <i>KB</i>4<i>cm</i>hoặc <i>KB</i>6<i>cm</i>


<b>PHÒNG GD&ĐT ĐƠNG SƠN </b>


<b>TRƯỜNG THCS NGUYỄN CHÍCH </b> <b>ĐỀ GIAO LƯU HỌC SINH GIỎI NĂM HỌC 2017-2018 </b>


<i><b>y</b></i>


<i><b>C</b></i>




<i><b>D</b></i>

<i><b>A</b></i>

<i><b>B</b></i>



<i><b>x</b></i>


<i><b>B</b></i>



<i><b>D</b></i>

<i><b>A K</b></i>



<i><b>x</b></i>


<i><b>B</b></i>



</div>
<span class='text_page_counter'>(108)</span><div class='page_container' data-page=108>

<b>Mơn thi: TỐN 6 </b>
<b>Câu 1. (2,0 điểm) </b>


Tính hợp lý


2 2 2


15 9 20 9
9 19 29 6


)21.7 11.7 90.7 49.125.16
5.4 .9 4.3 .8


)


5.2 .6 7.2 .27


<i>a</i>
<i>b</i>



  





<b>Câu 2. (6,0 điểm) </b>Tìm <i>x</i>là số tự nhiên, biết:


 



2 3 2 2


2 2


0,4


1 3 <sub>9</sub> <sub>11</sub> 1 8


) : 9 )


8 8


2 2 <sub>1,6</sub> 2 1


9 11


)5 <i>x</i> 2.5 5 .3 ) 2 7 20 5. 3


<i>x</i>


<i>a x</i> <i>b</i>



<i>x</i>


<i>c</i>  <i>d</i> <i>x</i>


  <sub></sub>


 <sub></sub> <sub></sub> <sub></sub>


  <sub></sub>


  <sub> </sub>


     


<b>Câu 3. (6,0 điểm) </b>


a) Tìm số nguyên <i>x</i>và <i>y</i>,biết: <i>xy</i> <i>x</i> 2<i>y</i>3
b) Tìm các số tự nhiên <i>x y</i>, biết: 2 .3<i>x</i>1 <i>y</i> 12<i>x</i>


c) Cho số 155*710*4*16 có 12 chữ số. Chứng minh rằng nếu thay các dấu *
bởi các chữ số khác nhau trong 3 chữ số 1;2;3một cách tùy ý thì số đó ln
chia hết cho 396


d) Tìm số tự nhiên <i>n</i>để biểu thức sau là số tự nhiên:


2 2 5 17 3


2 2 2



<i>n</i> <i>n</i> <i>n</i>


<i>B</i>


<i>n</i> <i>n</i> <i>n</i>


 


  


  


<b>Câu 4. (5,0 điểm) </b>


Cho đoạn thẳng <i>AB</i>5<i>cm</i>.Lấy điểm <i>M</i>thuộc đoạn thẳng <i>AB</i>,trên tia đối
của tia <i>AB</i> lấy điểm <i>N</i>sao cho <i>AN</i> <i>AM</i>


a) Tính <i>BN</i> khi <i>BM</i> 2<i>cm</i>


b) Trên cùng một nửa mặt phẳng có bờ là đường thẳng <i>AB</i>,vẽ các tia <i>Ax</i>và


<i>Ay</i>sao cho <i>BAx</i>40 ,0 <i>BAy</i>110 .0 Tính <i>yAx NAy</i>,


c) Xác định vị trí của điểm <i>M</i>trên đoạn thẳng <i>AB</i>để đoạn thẳng <i>BN</i>có độ dài


lớn nhất.


<b>Câu 5. (1,0 điểm) </b>


</div>
<span class='text_page_counter'>(109)</span><div class='page_container' data-page=109>

<b>ĐÁP ÁN </b>


<b>Câu 1. </b>






2 2 2 2


15 9 20 9 30 18 2 20 27
9 19 29 6 9 19 19 29 18
29 18 2


28 18


)21.7 11.7 90.7 49.125.16 7 . 21 11 90 49.125.16
49.100 49.100.20 49.100 1 20 49.100.21


5.4 .9 4.3 .8 5.2 .3 2 .3 .2
)


5.2 .6 7.2 .27 5.2 .2 .3 7.2 .3
2 .3 . 5.2 3


2 .3 . 5.3 7.2


<i>a</i>
<i>b</i>
      
    
 <sub></sub> 


 

 
 2
<b>Câu 2. </b>


2 2 2 2


0,4 0,4


1 3 <sub>9</sub> <sub>11</sub> <sub>9</sub> <sub>11</sub>


) : 9 :8


8 8 2 2


2 2 <sub>1,6</sub>


4. 0,4


9 11 9 11


1


:8 2


4


<i>a x</i> <i>x</i>



<i>x</i> <i>x</i>
   
 <sub></sub> <sub></sub> <sub></sub> <sub></sub>
  <sub></sub> <sub></sub>
  <sub> </sub> <sub></sub> <sub> </sub> <sub></sub>
 
   


Vậy <i>x</i>2


2

 

2


1 8


) 1 16 4


2 1


*) 1 4 3


*) 1 4 5


<i>x</i>
<i>b</i> <i>x</i>
<i>x</i>
<i>x</i> <i>x</i>
<i>x</i> <i>x</i>
      

   


     


Do <i>x</i> nên <i>x</i>3


2 3 2 2 2 3 2 2 2 3 2


2 3 3


)5 2.5 5 .3 5 5 .3 2.5 5 5 .5


5 5 2 3 3 2 6 3


<i>x</i> <i>x</i> <i>x</i>


<i>x</i>


<i>c</i>


<i>x</i> <i>x</i> <i>x</i>


  




      


        


Vậy <i>x</i>3



 



) 2 7 20 5. 3 2 7 5 2 7 5


*)2 7 5 6


*)2 7 5 2 2 1


<i>d</i> <i>x</i> <i>x</i> <i>x</i>


<i>x</i> <i>x</i>


<i>x</i> <i>x</i> <i>x</i>


          


   


      


Vậy <i>x</i>

 

6;1


<b>Câu 3. </b>


</div>
<span class='text_page_counter'>(110)</span><div class='page_container' data-page=110>

1

 

2 1

1

1



2

1


1 1 2


*)



2 1 1


1 1 0


*)


2 1 3


<i>x y</i> <i>y</i> <i>y</i> <i>x</i>


<i>y</i> <i>y</i>
<i>x</i> <i>x</i>
<i>y</i> <i>y</i>
<i>x</i> <i>x</i>
        
  
 <sub></sub>
 <sub> </sub>  <sub> </sub>
 
   
 <sub></sub>
 <sub>  </sub>  <sub> </sub>
 


Vậy <i>x</i> 1;<i>y</i>2hoặc <i>x</i> 3;<i>y</i>0
b) 2 .3<i>x</i>1 <i>y</i> 12<i>x</i>2 .3<i>x</i>1 <i>y</i> 4 .3<i>x</i> <i>x</i> 2 .32<i>x</i> <i>x</i>


2
1
1


2 3
2 3
2 3
<i>x</i> <i>y</i>


<i>x</i> <i>y x</i>


<i>x</i> <i>x</i>


 




   


Nhận thấy:

 

2;3         1 <i>x</i> 1 <i>y</i> <i>x</i> 0 <i>x</i> <i>y</i> 1


c) Ta thấy, vị trí của các chữ số thay thế ba dấu sao trong số trên đều ở hàng
chẵn vì ba chữ số đó đơi một khác nhau, lấy từ tập hợp

1;2;3 nên tổng của


chúng luôn bằng 1 2 3 6  


Mặt khác 3964.9.11, trong đó 4;9;11đôi một nguyên tố cùng nhau nên ta
cần chứng minh <i>A</i>155*710*4*16chia hết cho 4;9;11


Thật vậy:


*)<i>A</i> 4 vì số tạo bởi hai chữ số tận cùng của <i>A</i>là 16 chia hết cho 4
*)<i>A</i> 9vì tổng các chữ số chia hết cho 9:





1 5 5 7 1 4 1 6          * * * 30 6 36 9 


*)<i>A</i>11vì hiệu số giữa tổng các chữ số hàng chẵn và tổng các chữ số hàng lẻ là 0,
chia hết cho 11


1 5 7 4 1             

 

5 1 6 * * *

18 12 6 0
Vậy <i>A</i> 396




2 2 5 17 3 2 2 5 17 3 4 19


)


2 2 2 2 2


4 2 11


4 19 11


4


2 2 2


<i>n</i> <i>n</i> <i>n</i> <i>n</i> <i>n</i> <i>n</i> <i>n</i>


<i>d B</i>


<i>n</i> <i>n</i> <i>n</i> <i>n</i> <i>n</i>



<i>n</i>
<i>n</i>


<i>B</i>


<i>n</i> <i>n</i> <i>n</i>


      
    
    
 

   
  


Để <i>B</i>là số tự nhiên thì 11
2


<i>n</i> là số tự nhiên




11 <i>n</i> 2 <i>n</i> 2 <i>U</i>(11) 1; 11


       


</div>
<span class='text_page_counter'>(111)</span><div class='page_container' data-page=111>

<b>Câu 4. </b>


a) Vì <i>M</i>thuộc <i>AB</i>nên <i>AM</i> <i>MB</i> <i>AB</i><i>AM</i>   2 5 <i>AM</i> 3<i>cm</i>



Có <i>AN</i> <i>AM</i> <i>AN</i>3<i>cm</i>


Do N thuộc tia đối của tia <i>AB</i>nên điểm <i>A</i>nằm giữa N và B
5 3 8


<i>BN</i> <i>AB</i><i>AN</i>    <i>cm</i>. Vậy <i>BN</i> 8<i>cm</i>


b) +Trên cùng một nửa mặt phẳng có bờ chứa tia <i>AB</i>có:


0 0



40 110


<i>BAx</i><i>BAy</i>  Tia <i>Ax</i>nằm giữa hai tia <i>AB</i>và <i>Ay</i>nên ta có:
<i>BAx</i><i>xAy</i><i>BAy</i>hay 400 <i>xAy</i>1100 <i>xAy</i>700


Trên cùng một nửa mặt phẳng có bờ <i>AB</i>,ta có <i>BAy</i>và <i>NAy</i>là hai góc kề bù
Hay <i>BAy</i><i>NAy</i>1800hay 1100 <i>NAy</i>1800 <i>NAy</i>1800 1100 700


c) Vì <i>BN</i> <i>AB</i> <i>AN</i> 5 <i>AN</i><i>BN</i>có độ dài lớn nhất khi <i>AN</i>có độ dài lớn


nhất


Mà <i>AN</i>  <i>AM</i> <i>BN</i>có độ dài lớn nhất khi <i>AM</i>có độ dài lớn nhất


Có <i>AM</i> <i>AB</i><i>AM</i>lớn nhất khi <i>AM</i> <i>AB</i>khi đó điểm M trùng với điểm <i>B</i>
Vậy khi điểm M trùng với điểm B thì <i>BN</i>có độ dài lớn nhất.


<b>Câu 5. </b>



Dãy số 1;2;...;<i>n</i>có n số hạng 1 2 ....

1 .


2


<i>n</i> <i>n</i>


<i>n</i> 


    


Mà 1 2 3 ....    <i>n aaa</i>


Suy ra

1 .

.111 .3.37

1

2.3.37.
2


<i>n</i> <i>n</i>


<i>aaa</i> <i>a</i> <i>a</i> <i>n n</i> <i>a</i>




     


Vì tích <i>n n</i>

1

chia hết cho số nguyên tố 37 nên <i>n</i> 37hoặc <i>n</i>1 37


<i><b>y</b></i>



<i><b>x</b></i>



<i><b>M</b></i>



<i><b>A</b></i>



</div>
<span class='text_page_counter'>(112)</span><div class='page_container' data-page=112>

Vì số

1


2


<i>n</i> <i>n</i>


có 3 chữ số   <i>n</i> 1 74 <i>n</i> 37hoặc <i>n</i> 1 37
Với <i>n</i>37thì 37.38 703


2  (loại)
Với 1 37 36.37 666


2


<i>n</i>    (thỏa mãn)


</div>

<!--links-->
Tài liệu Đề thi học sinh giỏi toán lớp 4 cấp tỉnh docx
  • 12
  • 15
  • 282
  • ×